Osce Basil.

February 12, 2017 | Author: AsuganyadeviBala | Category: N/A
Share Embed Donate


Short Description

Download Osce Basil....

Description

Content Medicine CVS

Diabetes

Neurology

Ankle swelling Calf pain Hypertension MI Palpitation Syncope

DKA Tiredness

Approach to Headache, acute Headache, environmental Headache, primary Headache, subacute Headache, template Multiple Sclerosis Neuropathic pain Weakness in arm

RS Asthma Cough, Acute Cough, Chronic Hemoptysis SOB SOB – Post Surgery

Hematology High MCV Wrong blood transfusion

Ophthalmology Loss of vision

GI Abnormal LFT Acute abdomen Diarrhea Dysphagia GERD Mesenteric Iscemia

GU

Otolaryngology Allergic rhinitis Lump in neck

Introduction Dark urine Geriatric wets himself Difficulty to pass urine

HIV Breaking news – HIV Fever and tiredness Headache – HIV complications HIV test request Needle stick injury Tiredness for 6 weeks

Please note that this is only a draft version based on several sources, including: Dr. Basel Mohasen’s lectures, Toronto Notes, Therapeutic Choices and others. Edited and organized for the sake of all attendances of the Canadian Osce Exams: NAC OSCE and MCCQE2. by: Dr. Merlyn D’Souza and Dr. Zeev Gross, Spring 2011. Page 1

Counselling

Paediatrics

Abortion Ante-natal counselling Breast Feeding Diabetic daughter Domestic violance Endometrial cancer Epilepsy Fall Febrile seizure HRT Lump in breast Obesity OCP Pregnancy Smoking Cessation Warfarin Counseling Pap Smear OCP 3 Breast feeding 5 Ante-natal 7 HIV 9 Needle stick 10 IUGR 11 Abortion 12 Sexual abuse 14 Epilepsy 15 Drug seeker 17 Smoking cessation 18 Alcohol cessation 20 Imptence 21 HRT 22 Obesity 24

Introduction Anaphylactic Shock Cough Cry Diarrhea Diabetic daughter Febrile seizure Fever Marijuana counseling Osgood Schlatter Pale Phone cases Post-concussion Vomiting Yellow discoloration Immunization IUGR Child abuse Enuresis

FTT 1 Child abuse 2 Speech delay 3 Vomiting in newborn 4 Enuresis 5 Febrile seizures 6 Jaundice child 7 Joint pain 8 Child with ADHD 9 Child with chronic cough 10 Child with abdominal pain 11 Well-baby visit 12 Child with fever 14 Yellow baby 16 Vomiting baby 17 Baby with diarrhea 18 Pallor baby 19 Child with chronic cough 20 Child with abdominal pain 21 Child abuse 22 Enuresis 26 Hyperactive child 27 History taking format 28 Vomiting child 30 Anemia in a child 32 Diarrhea in a child 34 Enuresis 36 Breast feeding 37 Delayed speech 39 ADHD 40 FTT 41 Child abuse 43 Anorexia nervosa 44 Vaccination 46

Please note that this is only a draft version based on several sources, including: Dr. Basel Mohasen’s lectures, Toronto Notes, Therapeutic Choices and others. Edited and organized for the sake of all attendances of the Canadian Osce Exams: NAC OSCE and MCCQE2. by: Dr. Merlyn D’Souza and Dr. Zeev Gross, Spring 2011. Page 2

OBGYN

Psychiatry

Physical Exam / Manage

Introduction Amenorrhea Ask for file after C/S HRT Infertility Pre-eclampsia Request for C/S Vaginal bleeding Vaginal discharge First trimester bleeding 1 Third trimester bleed 2 Pre-eclampsia 3 High risk pregnancy 4 Counseling breast feed 5 Vaginal bleeding 6 Vaginal discharge 7 Amenorrhea 8 OCPs 9 Counseling HRT 10 Screen for Breast Ca 11 Pap smear 12

Psychiatry Introduction Anorexia Nervosa Bipolar Disorder Delusions – contamination Delusions – persecutory Depression Forms Insomnia Mania Marijuana in the bag MMSE MMSE-Delirium MSE-Psychosis Panic Attack Personality disorders Psychiatric assessment Req. admit (Delusion) Req. Admit (Borderline) Request to stop Lithium Somatization disorder Suicide Attempt Depression 21 Manic episode 24 Sleep hygiene 26 Delusional disorder 35 Schizophrenia 36 Panic disorder 39 MMSE 42 MSE 43 Dementia 44 Delirium 46 Suicidal attempt 48 Competency=Capacity 50 Alcohol abuse 55 Borderline personality D. 57

Introduction to Physical Exam Acute and acute on chronic abdomen Back Blood transfusion Cranial Nerves CVS Diabetic Foot ER: Trauma and Non-Trauma Hand-Laceration Hand - CTS Hematemesis Hip Knee Neck PVD Respiratory system Secondary Hypertension Shoulder Unconscious Patient Volume Status MI GI Bleeding 3 DKA 5 Asthmatic attack 7 TCA Overdose 8 Seizure 9 Anaphylaxis 11 Acute aortic dissection 12 Subarachnoid hemorrhage 14 Violent patient 17

Ethics Decision to forgo treat 3 Delivering bad news 4 Woman abuse 5 Telling the truth 6 Death before arrival 7 Pharmacist refusal 8 Organ retrieval 9 Confidentiality 10 Decision maker 11 Substitute Decision M 12 (Admition) notes 13

Please note that this is only a draft version based on several sources, including: Dr. Basel Mohasen’s lectures, Toronto Notes, Therapeutic Choices and others. Edited and organized for the sake of all attendances of the Canadian Osce Exams: NAC OSCE and MCCQE2. by: Dr. Merlyn D’Souza and Dr. Zeev Gross, Spring 2011. Page 3

Tool

Setting

Meaning

OCD

Any S&S

Onset, Course, Duration

PQRSTUV

Pain

Position, Quality, Radiation, Scale, Time/Triggers, How it affects YOUR life? Deja Vu – is it the first time?

COCA-B

Secretion

Colour, Odour, Consistency, Amount, Blood

MOAPS

Psychiatric screening

Mood, Organic, Anxiety, Psychosis, Suicide/Homicide/Self care

HEAADDDSSSS

Teen ager

Home, Education, Alcohol, Activities (hobies), Diet, Dating, Drugs, Sexual activity, Stress, Sleep, Suicide,

MGOS

OBGYN questions in Hx

Menstrual, Gynecological, Obstetrics, Sexual

MI PASS ECG

R/O Depression

Mood, Interest, Psychomotor retardation, Appetite, Sleep, Suicide, Energy, Concentration, Guilt

SAD PERSONS

Risk of suicide

Sex: male; Age: >60; Depressed; Previus attempts; Ethanol/drugs; Rational thinking loss; Suicide in family; Organized plan; No support; Serious illness/pain

DIG FAST

R/O Mania

Distractability, Impulsivity, Grandiosity, Flight of ideas, Goal directed Activity, Sleep, Talkative

ABCD

HRT or OCP C/I

Active liver disease, Blood diathesis, Cance (breast, endometrial), DVT

ABCDE

Benefits of OCPs

Anemia and Acne – reduced; Benign breast disease decreased; Cancer (ovarian) decreased, Cycles regulated, Cervical mucous increased (reduces STIs), Dysmenorrhea decreased, Ectopic pregnancy/ pregnancy reduced.

SPIKE

Bad news

Setting, Perception, Invitation, Knowledge, Empathy

BINDE

Child

Birth, Immunization, Nutrition, Development, Environment

AMPLE

Trauma

Allergies, Medications, PMHx, Last menstruation/meal/tetanus shot, Event

SEADS

Joint P/E

Swelling, Erythema, Atrophy, Deformity, and Scars

Please note that this is only a draft version based on several sources, including: Dr. Basel Mohasen’s lectures, Toronto Notes, Therapeutic Choices and others. Edited and organized for the sake of all attendances of the Canadian Osce Exams: NAC OSCE and MCCQE2. by: Dr. Merlyn D’Souza and Dr. Zeev Gross, Spring 2011. Page 4

CAGE

Screening for alcoholism

Ever felt the need to Cut down on drinking? Ever felt Annoyed at criticism of your drinking? Ever feel Guilty about your drinking? Ever need a drink first thing in the morning (Eye opener)

MUD PILES

Causes to high AG Met. Ac.

Methanol; Uremia; DKA; Paraldehyde; Isopropyl; Lactate; Ethanol; Salicylates

HARD UP

Causes of Non AG Met. Ac.

Hyperalimintation; Acetazolamide; RTA; Diarrhea; Ureteroentric fistula; Pancreaticodudenal fistula

ABCDEFGHI-M

Causes of AST and ALT raise

Autoimmune hepatitis; Hepattis B; Hepatitis C; Drugs or toxins; Ethanol; Fatty liver; Growths (i.e. tumors); Hemodynamic disorders (CHF); Iron (hemochromatosis), copper (Wilson’s disease) or alph1antitrypsin deficiency; Muscle injury

Please note that this is only a draft version based on several sources, including: Dr. Basel Mohasen’s lectures, Toronto Notes, Therapeutic Choices and others. Edited and organized for the sake of all attendances of the Canadian Osce Exams: NAC OSCE and MCCQE2. by: Dr. Merlyn D’Souza and Dr. Zeev Gross, Spring 2011. Page 5

WARFARIN COUNSELLING 40 M came to clinic as he was informed by clinic to come as his INR result was 1. Next 10 mins take history & provide counselling Divide into 2 parts: 1. History ------> 5 mins 2. Counselling ------> 5mins General scheme: 1. Event 2. Symptoms at the time of prescription 3. Compliance 4. Risk of bleeding from other sites 5. R/O relapse of DVT 6. Drugs and diet that interfere with warfarrin: Grapefruit, Antibiotics, NSAIDs, Antifungals, Restart INR (fresh person) X3 dose of 10mg/d  change INR every day (change the warfarrin accordingly) Check three time a week the INR if three consecutive are in the goal range  once aweek  than for every two weeks check once  than every month (or depend on the condition) If INR>10 or patient is bleeding  Vit. K (if active bleeding give FFP according to setting) Greeting: Good afternoon Mr.Hendricks,I’m Dr.X with you & will be your physician for today. As I understand, you’re here to discuss your blood reports. 1. Why was the blood test done? [Pt had DVT x 5 wks ago,& was having regular checkups till last week when he decided to stop as he’d read some alarming information on the internet & did not like warfarin (or other scenario,his friend who was on warfarin had a stroke)] 2. When was DVT Diagnosed? 3. How was it diagnosed? 4. What was done?? 5. WAS HE TREATED AS AN OUTPATIENT OR WAS HE ADMITTED?/If Yes: How many days? 6. What were the symptoms at that time? 7. Was there pain & swelling? 8. Was there SOB (lung involvement) DO NOT LOOK FOR FACTORS THAT CAUSE DVT 9. Which medicines were you treated with? -----> Blopd thinners/Warfarin? 10. Is INR done on a regular basis? 11. What was the last time it ws done? 12. What was the target? 13. What was the level?

Please note that this is only a draft version based on several sources, including: Dr. Basel Mohasen’s lectures, Toronto Notes, Therapeutic Choices and others. Edited and organized for the sake of all attendances of the Canadian Osce Exams: NAC OSCE and MCCQE2. by: Dr. Merlyn D’Souza and Dr. Zeev Gross, Spring 2011. Page 6

NOW BREAK THE NEWS Your INR is ONE; do you know the reason why? IF Pt vague, give him options: 1...Do you take your medications on a regular basis? 1. Do you take your meds by yourself or do you need help? 2. Any chance you were skipping a dose? 3. Did you start any new medications or antibiotics? 4. Diet: Are you eating a lot of spinach? 5. Any Vit K supplements? [If Pt expresses concern about bleeding S/e:Adress it & say it is a reasonable enough concern. I’ve to ensure that you do not have any bleeding at that time. Did you notice any blood from your gums,nose,bruises in body,coughing up blood? Neuro Sx:..... Since you stopped the meds, I want to ensure that there is no Relapse of your DVT: Do you have: Swelling/Calf pain/SOB/Heart racing/Chest tightness? 2 Qns about PMH: H/o long term illness or surgery FH COUNSELLING: What is your understanding of DVT?....clot Why did it occur? ....... The concern about this clot is that if not treated, there is a chance of relapse, or it may recur & this chance is: 8% To decrease this chance to 0.8% we use warfarin If DVT occurs more than twice – take life time medication. If not convinced: In addition to local recurrence there is damage to veins in the legs& valves& if this happens more than once warfarin has to be taken for a longer period In addition these clots formed in your legs may dislodge & travel all the way to your heart,& This is serious. If large, can cause, sudden death. Can travel to lungs & can cause a condition called PE which again is a very serious condition Of course the main side effect is bleeding which is very rare if properly monitored. As long as INR is in normal limits chances of bleeding are minimal i.e: 1% We’ve to restart with Heparin & warfarin & monitor INR on a daily basis

Back to Content

Please note that this is only a draft version based on several sources, including: Dr. Basel Mohasen’s lectures, Toronto Notes, Therapeutic Choices and others. Edited and organized for the sake of all attendances of the Canadian Osce Exams: NAC OSCE and MCCQE2. by: Dr. Merlyn D’Souza and Dr. Zeev Gross, Spring 2011. Page 7

EPILEPSY COUNSELLING Young 16 yr old male for driving License counsel Always ask Qn as to why he wants a driver’s license. Usually a Dr does not give such a note unless there is an underlying condition. General scheme: 1. Intro 2. Event – before in and after the attack. When was the drug level checked? Any other medications that might interfere with epileptic drugs (e.g. OCPs). 3. If it is only seizure  go to secondary causes of seizures  refer to neurology 4. Which medication, and compliance 5. HEADDSSS – 6. Triggers – sleep deprivation 7. MOAPS 8. Counsel: needed to be seizure free for one year. Invite him again for f/u after one year. Risk behaviour: drivint, swimming, hicking, bath door open and don’t take bath but can take shower, no heavy machines You have to take it for your whole life – if you have any attack let me know and we’ll discuss it. Talk with your physician about any new medication you want to take. Valproic acid 500mg. OCD: 1. Age of onset 2. When was the Ds 3. What was the Ds 4. How long does each attack last? 5. How frequently do the attacks occur? 6. +/- LOC 7. Aura prior to attack 8. How does she feel after the attack? 9. What meds is she on/Is she compliant/Were the drug levels checked? 10. Any other meds (if female ask about OCs) 11. When was last attack? 12. What happens during an attack? Does she shake/All over/Partly/roll up her eyes/bite her tongue? I’ve to ask questions to R/o any new pathology:

Please note that this is only a draft version based on several sources, including: Dr. Basel Mohasen’s lectures, Toronto Notes, Therapeutic Choices and others. Edited and organized for the sake of all attendances of the Canadian Osce Exams: NAC OSCE and MCCQE2. by: Dr. Merlyn D’Souza and Dr. Zeev Gross, Spring 2011. Page 8

System review: CNS: Head trauma/HA/Vi CSx: MOOD: Any chance you may hurt yourself? PMH: h/o Dm HEAADDS HOME: With whom do you live/How is your relationship with parents/siblings EDUCATION: How is school? How’re your grades? Any recent change in grades? ACTIVITIES: what are your hobbies? ALCOHOL: Sometimes kids your age might smoke or take alcohol & experiment with drugs, any of your friends do it? How about you? If YES: How much/How often? DIET: How is your diet? DATING: Are you dating? Are you sexually active? STIMULANT USE: STRESS: SLEEP: Do you have enough sleep?

MAKE SURE that he knows what a seizure is What do you know about epilepsy? It is a common condition due to increased electrical activity in the brain, some people lose consciousness, and some do not. It does not cause learning disability or damage the brain In those who have seizure attacks: If lasts for a few minutes there is no brain damage If lasts for > 30 minutes, will cause brain damage People with epilepsy should AVOID dangerous activities such as: 1. Driving 2. Mountain climbing 3. Swimming 4. Operate machinery 5. Boating 6. Chewing gum 7. Tub baths (have a shower bath & never lock bathroom door) You can have a driver’s licence only after you’ve been seizure free for 1 year It is my duty as your physician to inform the Ministry of transportation

Please note that this is only a draft version based on several sources, including: Dr. Basel Mohasen’s lectures, Toronto Notes, Therapeutic Choices and others. Edited and organized for the sake of all attendances of the Canadian Osce Exams: NAC OSCE and MCCQE2. by: Dr. Merlyn D’Souza and Dr. Zeev Gross, Spring 2011. Page 9

Mention TRIGGERS  If you drink alcohol, it decreases the point at which ea seizure occurs and can cause an attack  Sleep deprivation also can cause it  So also flashing lights If you want to take any other meds, speak to your Dr I will check the blood levels of your medications to see if it is at the therapeutic level If you want to get pregnant consult your Dr There are support groups Regular F/u Any Concerns?

Back to Content

Please note that this is only a draft version based on several sources, including: Dr. Basel Mohasen’s lectures, Toronto Notes, Therapeutic Choices and others. Edited and organized for the sake of all attendances of the Canadian Osce Exams: NAC OSCE and MCCQE2. by: Dr. Merlyn D’Souza and Dr. Zeev Gross, Spring 2011. Page 10

OCP COUNSELLING 21 F for OCPs Counsell x 10 mins General scheme: Intro: Good morning xxx,I’m Dr...... As I understand,you’re here today because you want a prescription for Birth Control pills. During the next few minutes, I will ask some questions that will help me 2 Questions here: 1. Have you ever used any form of contraception before ? 2. Why do you want to use it? 2.1. If in stable relationship 2.2. If sexually active 2.3. Do you practise safe sex? 2.4. How do you feel about this relationship? 2.5. Prior to this were you in any other relationship? 2.6. Whose idea was it/ Yours or His? MGOS MENSES: MENSES Use the word ―period‖ 1. When was your last period? 2. Are your periods regular / not 3. How often? 4. How many days or How long does it last? 5. How many pads do you use/change? 6. Are the pads full? 7. Are they heavy? 8. Do you see clots? 9. Between periods do you have spotting? 10. From your last menstrual period was your period different from the current one? 11. At what age did you start your periods? 12. Were they regular/irregular? 13. When did it become regular? 14. Are your periods painful / painless? 15. If irregular from beginning? 16. Discharge – ask if pregnant and when LMP GYENECOLOGY 1. Any history of Gyn. Disease – polyps or cysts 2. History of pelvic surgery (if yes – when?) 3. Have you used any birth control? 4. When/type/any complications Please note that this is only a draft version based on several sources, including: Dr. Basel Mohasen’s lectures, Toronto Notes, Therapeutic Choices and others. Edited and organized for the sake of all attendances of the Canadian Osce Exams: NAC OSCE and MCCQE2. by: Dr. Merlyn D’Souza and Dr. Zeev Gross, Spring 2011. Page 11

5. Pap’s smear OBSTETRICS: Have you ever been pregnant? Have you ever had an abortion or miscarriage? SEXUAL HISTORY: Any STIs? Any PIDs? Any partner with STI? CONTRAINDICATIONS: To find out if you’re a suitable candidate,I need to ask a few more questions: ABCD (Active liver disease, Bleeding, Cancer, DVT) 1. Any abnormal vaginal bleeding? 2. Any active liver disease: (Ac & Ch) 3. CVS:Have you ever had clots in your calves/DVT/Very High blood pressure 4. H/o Migraine headaches? 5. FH of Ca breast/Uterine or Liver AGREE to give if No CI EXPLAIN what are OCs 1. Combination of hormones Estrogen & Progesterone or sometimes only a progesterone 2. These come in packs of 21 or 28 pills 3. They prevent pregnancy by interfering with hormone signals in our body & prevent ovulation 4. Also make the inner lining of the womb & makes it hostile for conception 5. Thickens secretions at the mouth of the uterus & prevents conception MISSED PILL; To be taken at same time every day, so chances of forgetting is less & constant blood levels 1St pill on 1st Sunday of period, or 1st day of periods 1st month use back up method of Cx like a condom

In first 2 weeks: If miss one pill: Take 2 pills next day & use condom x 1 week If miss 2 pills: Take 2 pills same day + 2 pills day after + Condom x 7 days If miss 3 pills: Stop,use condom & restart new pack

In 3rd week: If you miss any pills restart new pack Explain BENEFITS: 1. Help regulate cycle if periods are irregular Please note that this is only a draft version based on several sources, including: Dr. Basel Mohasen’s lectures, Toronto Notes, Therapeutic Choices and others. Edited and organized for the sake of all attendances of the Canadian Osce Exams: NAC OSCE and MCCQE2. by: Dr. Merlyn D’Souza and Dr. Zeev Gross, Spring 2011. Page 12

2. Will eliminate pain 3. Less blood loss during periods 4. Less chances of benign breast disease & ovarian Ca But like any other medications, there are also the SIDE EFFECTS:  MILD N/V,Wt gain (5lbs)breast heaviness,mood changes,Spotting may occur in the initial months If these occur,you can change brand  SEVERE; Severe Ha/SOB Chest pain -----> If these occur STOP the pill & sek urgent medical attentiomn DRUG INTERACTIONS: If takes any other medications,let her Dr know she s on the pill SAFE Sex: PAP’s Smear If Teenager: HEAADDS Last any Concerns or qns?

Back to Content

Please note that this is only a draft version based on several sources, including: Dr. Basel Mohasen’s lectures, Toronto Notes, Therapeutic Choices and others. Edited and organized for the sake of all attendances of the Canadian Osce Exams: NAC OSCE and MCCQE2. by: Dr. Merlyn D’Souza and Dr. Zeev Gross, Spring 2011. Page 13

ABORTION 1. Young woman 19 yrs asking for abortion x 10 mins counsel 2. Can be a teenager with a vague complain a. Read body language & assure Confidentiality b. When did sexual contact occur? c. Who was the partner? d. Was she raped or was it against her will? e. Is she being regularly abused? f. Do her parents know? M (Signs of pregnancy: engorgement of breast, urine frequency, n&v) O G S PMH SHx: HEADDSSS: Home enviorenment & parental attitude MOOD & Interest SUICIDAL IDEATION NOW? When pt tells outright she wants an abortion: How do you feel about having an abortion? If she says she feels there is no other option: Explain that there are other options, Do you want to know them? When did she find out she was pregnant? Here be sensitive if she found out last night, she is probably in a panic, but if she has known it for a week, she has thought about it well, & is more decisive. Before it can be done, I need to ask you some qns 1. How did she find out she was pregnant? 2. Was she using any contraception? 3. Has she spoken to her partner/family/friend? 4. Would she like to talk about it? LMP: How was her LMP, was it similar to her previous LMP? Or was it shorter, lighter? Is there Nx/V,visiting washroom more frequently? Breast engorgement? O Have you been pregnant before? Have you ever had an abortion/miscarriage? G Has she ever used any kind of birth control before? Any Pelvic surgeries Any Pap smear (depending on age) H/O STIs Since it is the first time I’m seeing you, I need to ask about PMH; Please note that this is only a draft version based on several sources, including: Dr. Basel Mohasen’s lectures, Toronto Notes, Therapeutic Choices and others. Edited and organized for the sake of all attendances of the Canadian Osce Exams: NAC OSCE and MCCQE2. by: Dr. Merlyn D’Souza and Dr. Zeev Gross, Spring 2011. Page 14

Any H/o HTN/Liver disease/DM? Surgeries/Anaesthesia complications Blood Group Any Medications/Allergies SOCIAL Hx: With whom do you live? How do you support yourself financially? If young teen: HEAADDS Whatever you choose to decide, I will support you. Is she decides to go in for an abortion: I will refer you to an abortion clinic However it is difficult to get an obstetrician who will do it after 20 weeks She has to make a decision fast Also here ask about her own support system (family/boyfriend) I will also get you connected with a support group, who are women who’ve had abortions before & will help you cope with it.

Now in addition to abortion there are OTHER OPTIONS: Would you like me to tell you about them? 1. If your concern is financial, you can carry on this pregnancy & there are a lot of support groups as well as the government who will help you. 2. You can carry on this pregnancy & give up the baby for adoption, a lot of people are looking for a child also nowadays you can have visitation rights in certain cases. If Pt is still going for an abortion: For now, I will do 1. PAPs test 2. Blood tests: Sr B HCG & Blood group & Rh typing 3. USound Once your pregnancy has been confirmed by us, I will send you to the abortion clinic From now, until the time you’ve the abortion, you’ve to; Quit Smoking/Alcohol/Drugs If you happen to see any dr during this time period, you’ve to inform him you’re pregnant

Back to Content

Please note that this is only a draft version based on several sources, including: Dr. Basel Mohasen’s lectures, Toronto Notes, Therapeutic Choices and others. Edited and organized for the sake of all attendances of the Canadian Osce Exams: NAC OSCE and MCCQE2. by: Dr. Merlyn D’Souza and Dr. Zeev Gross, Spring 2011. Page 15

Smoking Cessation According to the type of patients we will allocate the time: Neutral: Hx (4m), Why (3m), How (3m) Willing: Hx (4m), Why (1m), How (5m) Unwilling: Hx (4m), Why (5m), How (1m) General scheme: 1. Intro (“Very good decision”) 2. Hx 2.1. Impact: breathing, coughing, phlegm 2.2. RF: HTN, DM, Hyperlipidemia, Questions about target organs: heart, lung 2.3. Gain from quitting: what do you think you are going to gain from quitting 2.4. Hx from previous quit – what support do you need? What led to relapse? Withdrawal symptoms? What is the longest time you quitted? 2.5. In which situations you need to smoke? 2.6. SHx: do you smoke in front of your children? 3. Counseling What is your motivation to stop smoking (scale 1 to 10) Different people from different reasons…what is the reason for your smoking? 3.1. Why 3.1.1. Effects of the smoking on different of the body 3.1.2. Reduced risk for diseases – time frame 3.1.3. Influence on other household 3.1.4. Economical effect 3.2. How 3.2.1. Set a quit date within 2 weeks – reduce gradually within 2 weeks 3.2.2. The support you’ll need – tell your family. Found someone who wants to quit. 3.2.3. Diary 3.2.4. Exercise, healthy diet 3.2.5. Things you can do instead of smoking 3.2.6. If taking nicoting replacement – stop smoking. 3.2.7. Medication: Ziban (bupropion) 150mg (only in the morning for three days and than increase to bid to 7-12 weekly up to 6months) S/E Insomnia and dry mouth C/I seizure, eating disorder, MAOI; Varenicline (Champex) S/E nausea C/I Previous psychiatric conditions

Please note that this is only a draft version based on several sources, including: Dr. Basel Mohasen’s lectures, Toronto Notes, Therapeutic Choices and others. Edited and organized for the sake of all attendances of the Canadian Osce Exams: NAC OSCE and MCCQE2. by: Dr. Merlyn D’Souza and Dr. Zeev Gross, Spring 2011. Page 16

Intro Hello Mr./Ms. ….. As I understand you are here today to seek help to quit smoking. I am really happy to hear that – can you tell me what made you come to that decision? What are your expectations from this visit? Motivation can be assessed by asking the following two questions: 1. “Given everything going on in your life right now, on a scale of 1 to 10, where 10 is the most important thing to do right now, how important is it for you to quit smoking altogether?” 2. “Given everything going on in your life right now, on a scale of 1 to 10, where 10 is the most confident you have felt about anything, how confident do you feel you will be able to quit smoking altogether?” Ask about the smoking now – how long, how much, since when Impact of smoking of his life: breathing and coughing, weakness, relationship and sex, CSx RF: HTn, DM, Cholesterol, FHx of CAD and Cancers, In your opinion - what are the good things you will gain from quitiing? Have you tried to quit before? What stopped you from quitting before? What support will you need in order to quit? What are the situations in which you usually smoke?

Please note that this is only a draft version based on several sources, including: Dr. Basel Mohasen’s lectures, Toronto Notes, Therapeutic Choices and others. Edited and organized for the sake of all attendances of the Canadian Osce Exams: NAC OSCE and MCCQE2. by: Dr. Merlyn D’Souza and Dr. Zeev Gross, Spring 2011. Page 17

Please note that this is only a draft version based on several sources, including: Dr. Basel Mohasen’s lectures, Toronto Notes, Therapeutic Choices and others. Edited and organized for the sake of all attendances of the Canadian Osce Exams: NAC OSCE and MCCQE2. by: Dr. Merlyn D’Souza and Dr. Zeev Gross, Spring 2011. Page 18

Smoking Hx What is the reason that made you decide to smoke? How much you smoke, how long (More than 10pk/y -

Withdrwal symptoms: Heart racing, sweating, shakiness, Irritable What is the longest time you quit smoking? (every time you quit you have a better chance of success.)

Sleeping pills; Please note that this is only a draft version based on several sources, including: Dr. Basel Mohasen’s lectures, Toronto Notes, Therapeutic Choices and others. Edited and organized for the sake of all attendances of the Canadian Osce Exams: NAC OSCE and MCCQE2. by: Dr. Merlyn D’Souza and Dr. Zeev Gross, Spring 2011. Page 19

Because you have been smoking for long time I’d like to see how this smoking has affected your health: Target organs: Heart, Lung, GI, PVD, Sexual, CHx MOAPS FHx (also addiction, cancer, suicide, depression) SHx Do you smoke in front of your children?

Counselling Why Different people smoke from different reasons - what is the reason you smoke? In your opinion – what are the advantages of smoking? Do you know what the active components in cigarettes are? (It is Nicotine, and when you smoke it you have a sense of well being. To maintain the same effect you keep increasing the number of cigarettes and by that tolerance develops. So, when you stop you get withdrawal symptoms, and therefore it is habit forming and difficult to quit. In addition to the effect on our brain it causes narrowing of our blood vessels all over the body. In the heart it causes heart attacks which are leading cause of death in our society. In the brain it causes stroke which is the third leading cause of death. In the GI it causes peptic ulcers. It can cause erectile dysfunction. In addition to nicotine, cigarettes can contain few thousands of other substances – some of these affect the lungs and cause COPD which is an irreversible condition which there is no treatment. On top of that, smoking is associate with cancer in a lot of different organs of which lung cancer is the leading cause of death from cancer worldwide. In addition to medical impact it affects also family members and expose them to most of the harmful effects mentioned previously. It is expensive, staining and smelling, increases hazards (fire). If you quit smoking you are taking the right step and can expect to gain the following:    

After 1 year, the risk of coronary heart disease is cut in half After 5 years, the risk of stroke falls to the same as a non-smoker After 10 years, the risk of lung cancer is cut in half and the risk of other cancers decreases significantly After 15 years, the risk of coronary heart disease drops, usually to the level of a nonsmoker

Please note that this is only a draft version based on several sources, including: Dr. Basel Mohasen’s lectures, Toronto Notes, Therapeutic Choices and others. Edited and organized for the sake of all attendances of the Canadian Osce Exams: NAC OSCE and MCCQE2. by: Dr. Merlyn D’Souza and Dr. Zeev Gross, Spring 2011. Page 20

How 1. Set a quit date. It should be within the next two weeks. Avoid a time when you will be under stress. 2. Think about why you want to quit and all the good things that you expect as a result of quitting. 3. If you have tried to quit before, you have probably learned some valuable tips of what not to do this time. Think about what was most difficult last time and why you gave up trying. Think about the things you need to avoid this time. 4. Decide what kind of support will be most helpful over the next six months to a year. For example, you can join a smoking cessation group or plan to meet regularly with a health professional (such as a pharmacist, nurse or doctor). 5. Tell your family and friends that you are quitting. Ask them to help you to stick to your plan. If they smoke, ask them to respect your decision to quit and to not smoke in front of you. Think of things you can do to avoid smoking while with them. 6. Find someone you know who does not smoke and ask them to help you to quit. 7. Make a diary for a few days to keep track of when and why you smoke. 8. Think of ways to avoid situations when you usually smoke. 9. Buy a brand you don’t like. Buy one pack at a time. Increase the time of lighting it, and smoke only part of it. 10. Think of things you can do instead of smoking (for example, chewing gum, sipping water, holding a fake cigarette). 11. Most people gain weight while quitting. You can avoid this by healthy eating and increased activity. Keep healthy snacks around for times when you get the urge to nibble. 12. Keep busy with healthy activities like walking or an exercise program. Starting a new activity will help to break old habits connected with smoking. 13. If you are taking medication to help you to quit, be sure to follow the instructions carefully. 14. If you are taking nicotine replacement therapy, do not continue smoking, as this is very dangerous to your health.

Please note that this is only a draft version based on several sources, including: Dr. Basel Mohasen’s lectures, Toronto Notes, Therapeutic Choices and others. Edited and organized for the sake of all attendances of the Canadian Osce Exams: NAC OSCE and MCCQE2. by: Dr. Merlyn D’Souza and Dr. Zeev Gross, Spring 2011. Page 21

Back to Content

Please note that this is only a draft version based on several sources, including: Dr. Basel Mohasen’s lectures, Toronto Notes, Therapeutic Choices and others. Edited and organized for the sake of all attendances of the Canadian Osce Exams: NAC OSCE and MCCQE2. by: Dr. Merlyn D’Souza and Dr. Zeev Gross, Spring 2011. Page 22

OBESITY COUNSELLING 38 YOM 6 ft height,weight=260Ib 10 min.HX. and Education Keys:  Explore motivations for wt.loss.  Provide information about the consequences of obesity (health and psychological wellbeing) and nutrition.  Set realistic goals, target BMI  Offer support/reinforcement throughout the weight-losing process.

Hx DIET & EXERCISE: Motivation for losing wt. now? Self –image, Health concerns? ……… Can you tell me more. Good decision to come today, I am glad you came. If patient request first surgery, tell him that sound reasonable however, surgery is not the first line can be done based on BMI if >40 If you like I can calculate it for you; kg/m2. I need > information about your condition: WEIGHT: 1. What is your weight today? 2. Highest weight. 3. When started to gain wt? 4. When started to be concerned? 5. Have you tried any wt- loss programs? 6. Which one? 7. How long? 8. Did you lose wt? 9. Why did you stop? I am going to ask you Qs to see WHY YOU’RE GAINING WEIGHT: Do you calculate your calorie intake? How many meals do you take/day including snacks? Tell me more about your DIET:  What do you eat?  How much fat, fruit, veg bread? Eat while watching TV, Before bed, Please note that this is only a draft version based on several sources, including: Dr. Basel Mohasen’s lectures, Toronto Notes, Therapeutic Choices and others. Edited and organized for the sake of all attendances of the Canadian Osce Exams: NAC OSCE and MCCQE2. by: Dr. Merlyn D’Souza and Dr. Zeev Gross, Spring 2011. Page 23

Breakfast daily, Ever eat to relax or when stressed? Binge eating? Do you feel guilty about your eating? Do you induce vomiting/purging? ALCOHOL How about your ACTIVITY, Do you exercise? IMPACT: I am going to ask you how this Affects your life? 1. Difficulty sleeping, 2. Tiredness, 3. Heart burn, 4. Nausea, vomiting, 5. GB stones, bowel motion, 6. Back pain, 7. Jt pain. PMH: HTN, DM, Medications: anti-psychotic, OTC, steroids, thyroid disease, OCP. Social Hx: With whom do you live? Any change in sexual desire? How it affects your self esteem,mood and interest? Do you smoke? Drink? Recreational drugs? FHx : obesity Education: There are some genetic factors that influence wt. We can’t modify these but we can modify our diet and exercise. In some people, diseases are the underlying cause for obesity. Give patient their ideal wt. for ht. >20% ideal wt is obesity. Being overwt increases the risk of Hypertension,CVD,CAD,GB disease,DM,fatty liver,cancers(breast,bowel),OA,sleep apnea,spinal dysfunction.  We recommend to lose 10% of your body wt.over 6 months (gradually).guidance is BMI There are 2 methods to lose wt: Decrease intake or Exercising more. If you like I can refer you to a dietician. We also recommend dividing your meals into 3 small and in between snacks ( carrot, veg.or fruit) 55% CHO, 15% protein,30% fat Avoid saturated fat, cheese, alcohol Give patient a target caloric intake: to lose 1 Ib/week,should take 300-500 kcal less 1g fat-9kcal, 1g CHO-4kcal, 1g protein-4kcal Do not recommend diet medications and fad diet, these may be harmful and are of no long-term benefit. If BMI>27 + RF (DM, Htn...) or BMI > 30 start pharmacotherapy: Xenical=increase bulk of stools, leakage, decrease absorption of fat sol.Vit. Meridia (sibutramine) = suppress appetite, cause heart racing, hypertension. IF BMI>35 + RF or BMI > 40 recommend Baratric srgery Please note that this is only a draft version based on several sources, including: Dr. Basel Mohasen’s lectures, Toronto Notes, Therapeutic Choices and others. Edited and organized for the sake of all attendances of the Canadian Osce Exams: NAC OSCE and MCCQE2. by: Dr. Merlyn D’Souza and Dr. Zeev Gross, Spring 2011. Page 24

Exercise:begin with walking,regularly 30 min,4-5 times per week Reach 60-80% maximum heart rate (220-age) Self-monitoring, group support Follow-up: advise patient to come back in a week with food intake diary etting sick

Please note that this is only a draft version based on several sources, including: Dr. Basel Mohasen’s lectures, Toronto Notes, Therapeutic Choices and others. Edited and organized for the sake of all attendances of the Canadian Osce Exams: NAC OSCE and MCCQE2. by: Dr. Merlyn D’Souza and Dr. Zeev Gross, Spring 2011. Page 25

Back to Content

70 yr old female with H/o fall at home .Brought in by ambulance personell to the ER. She is medicaly cleared; In the next 20 minutes take history & Counsel; Diff/Diag (Dd): 1.Poly pharmacy 2. Recent hypovolemeia  Diarrhea/Vomiting  Lack of intake  Recent bleeding 3.Orthostatic hypotension 4.Hypoglycemia 5.Elder abuse FALL: 1. When did the fall occur? 2. Where did it occur? 3. Were you alone? 4. Could you get up by yourself or did you need help? 5. How long before you got help? 6. Did you trip or just feel your legs give way? 7. If there was a witness around ask permission to speak to witness after you finish talking to Pt to obtain collateral history 3 parts of history relating to the EVENT: A.Before fall B.During the Fall C.After the fall

Events assoc with the Fall: A.Before: 1. 2. 3. 4. 5. 6. 7. 8.

Did you feel Light headed/ Spinning/ Hungry/ Heart racing & Sweating --- HYPOGLYCEMIA Chest pain/ Palpitations/ Shortness of breath----CVS Lights flashing/ Strange smell/ Strange feeling in body--Seizure Weakness/Numbness/Dificulty finding words/Visual disturbances --CNS/STROKE Was the lighting good? How is your vision Is your footwear comfortable?

Please note that this is only a draft version based on several sources, including: Dr. Basel Mohasen’s lectures, Toronto Notes, Therapeutic Choices and others. Edited and organized for the sake of all attendances of the Canadian Osce Exams: NAC OSCE and MCCQE2. by: Dr. Merlyn D’Souza and Dr. Zeev Gross, Spring 2011. Page 26

B.During the Fall: 1. 2. 3. 4.

Did your wife mention that you were shaking or making jerky movts? Did you wet yourself? Turn blue & were stiff? Bite your tongue?

C.After the fall: 1. Nausea/Vomiting 2. Weaknes 3. Difficulty finding words 4. Any vision difficulties 5. Loss of sensation in the arms or legs 6. Ringing in ears Has this ever happened before 1. When & where 2. Did you seek medical help then 3. What were you advised? CONSTITUTIONAL SYMPTOMS:  Fever & Chills & Night sweats  Wt loss & Loss of appetite  Lumps & Bumps Sx related to CVS: Chest pain/SOB/Palpitations Sx of CNS: Weakness/Numbness/Loss of vision/LOC Past Medical History; 1. Are you taking any medications? 2. Can you take them by yourself or does your caregiver give them to you? 3. Do you take them regularly as prescribed? 4. Can I see them please? Please see the meds Was there a recent change in the meds 5. Besides these do you take any additional OTC products or herbal medications? 6. Do you take alcohol? ..............  How much do you take regularly?  Did you take alcohol prior to the fall? 7. Do you have high blood pressure?  When was it last checked? Please note that this is only a draft version based on several sources, including: Dr. Basel Mohasen’s lectures, Toronto Notes, Therapeutic Choices and others. Edited and organized for the sake of all attendances of the Canadian Osce Exams: NAC OSCE and MCCQE2. by: Dr. Merlyn D’Souza and Dr. Zeev Gross, Spring 2011. Page 27

 What did your doctor have to say about it? 8. Do you have high blood sugar or Diabetes?  When was it last checked?  What did your doctor have to say about it? 9. Did you ever have a stroke or heart attack? 10. Were you ever diagnosed with Cancer 11. Were you hospitalized at any time in your life? I need to ask a few more questions concerning your lifestyle that will aid me to help you. It is all confidential & my duty is to help you (When you suspect Elder abuse) 1. With whom do you live? 2. Are you happy living with XXXXX 3. Who prepares your meals? 4. Do you do your own shopping? 5. Do you manage your own finances? 6. Do you go out of the house & meet up with friends & have your own social life? 7. Do you get into arguments with XXXX? 8. Have you ever been hit or yelled at or threatened by XXXX? COUNSELLING FOR POLYPHARMACY (Orthostatic Hypotension) Based on what you’ve told me most likely the reason of your fall is a condition called “Orthoststic Hypotension”.Have you ever heard about it?.......... When you change position from lying to sitting or standing blood pools to the legs & Bld vessels narrow to maintain BP. In pts with OH because of Age,Medications,DM or a combination of these condts body might fail to react,& blood pools in the legs & thus BP drops & there is not enough bld reaching the brain. There is a possibility that this might happen again & from now on whenever you change your posn from lying get up slowly,sit at edge of bed & slowly get up. I need to get in touch with your doctor & modify the dosage of your meds or change them. Is it alright with you? I need to talk now to your wife & do an ECG to check your heart

Back to Content

Please note that this is only a draft version based on several sources, including: Dr. Basel Mohasen’s lectures, Toronto Notes, Therapeutic Choices and others. Edited and organized for the sake of all attendances of the Canadian Osce Exams: NAC OSCE and MCCQE2. by: Dr. Merlyn D’Souza and Dr. Zeev Gross, Spring 2011. Page 28

HA DOMESTIC VIOLENCE Domestic Violence common presentations: 1. HA 2. Abd Pain 3. Ac Abd 4. Insomnia 5. Sleeping pills 6. Vaginal Bleeding Sx 1. No good eye contact 2. Vague complaints 3. Non communicative OCD/PQRST CONSTITUTIONAL Sx: R/o Migraine & Tension HA RISK FCTS: Smoke/Alcohol/Recreational drugs PMH:Are you on any meds/OTC/Herbal meds? Were you hospitalized at any time? FH: SOCIAL HISTORY: Important** All information you give here is entirely confidential & will not be released unless you authorize it Who lives with you? Any recent changes/Stress in your relationship SCREEN FOR DOMESTIC VILOENCE: Does your Partner: 1. Hit you? 2. What happens during an argument? 3. when he is angry,does he : 4. Shout/Swear & call you names or demean you? 5. Has your partner ever ridiculed you or cut you off from other relationships with friends/family? 6. Have you ever sought help from others in health care? ABUSE RISK FCTS: 1. Drink alcohol,drink more now than before? 2. Does he have access to firearms? 3. Does he ever get angry to the point where he gets physical & hits you?Did you ever have to go to the ER? Was there a serious consequence? 4. Are you having more arguments now Please note that this is only a draft version based on several sources, including: Dr. Basel Mohasen’s lectures, Toronto Notes, Therapeutic Choices and others. Edited and organized for the sake of all attendances of the Canadian Osce Exams: NAC OSCE and MCCQE2. by: Dr. Merlyn D’Souza and Dr. Zeev Gross, Spring 2011. Page 29

5. 6. 7. 8. 9.

Does he get more angry now,& How has all this affected your self esteem? How does it make you feel? Does he ever force you to have sex against your will? Who controls the finances & spending? Has he ever mistreated you in front of the children?- If yes: it is emotional abuseto children & has to be reported to CAS 10. Has he ever misRxed th children? 11. Have you ever thought of putting an end to your life or his life? Have you spoken to anybody abt this? Do you have some support? COUNSELLING: Empowering & Education 3 kinds of Pt: 1.She wants Help 2.She might Consider getting help 3.She does not want to get help & thinks he is right I’d like you to know that what you’re experiencing is called “Domestic Violence “or Spousal abuse. It is a crime against the law & not acceptable. It is not your fault & you should not accept it & feel guilty It can get out of hand & you can get harmed seriously Call Police (Never Call Police from your office) Contact Social worker, who will help you with housing, finding a job & finances & child support If she is considering Escape Plan Keep a bag with important documents,change of clothes & hide it DOCUMENT Fup x 3 days

Back to Content

Please note that this is only a draft version based on several sources, including: Dr. Basel Mohasen’s lectures, Toronto Notes, Therapeutic Choices and others. Edited and organized for the sake of all attendances of the Canadian Osce Exams: NAC OSCE and MCCQE2. by: Dr. Merlyn D’Souza and Dr. Zeev Gross, Spring 2011. Page 30

Diabetic Daughter 2y, Counsel Either she is not doing well in school Not seeing well Not playing well, tired DKA Is it regular f/u or something special you wanted to discuss? When was the last f/u? How was she diagnosed? What happen then? What were the symptoms? Is there any pain / vomiting? Are you feeling eating/drinking/peeing more? Is there any weight loss or blurred vision? From the last f/u till now have you had DKA? How about before? Have you had low blood sugar? Talk with the father: which medication does she take? How does she take? When was the last time? Do you take insulin or somebody else gave it to you? Do you take it all the time? DO you skip dose? Does she need any help to take insulin? DO you measure blood sugar regularly? When was the last time? Do you record them in the machine? (The glucometer should be used by only one patient). There is a blood work called ―Hemoglobin A1C‖ it is done every three month – did you do it? Did you start new medication? How about your diet? DO you have your log book? What do you eat? Have you ever seen by a dietician? PMHx FHx Counseling A lot of people have diabetes and she is not the only one. What’s your understanding of diabetes? Whenever we eat food contains sugar it is absorbed in our stomach and goes to the blood and from there to different parts of our body. Sugar act in our body like a fuel, in order for our body to use this energy it needs insulin. Patients having diabetes have not enough insulin. Sugar will be built up in your blood. The body tries to get rid or it, by peeing extra sugar – this will lead to thirsty and tiredness. This can be avoided by controlling the blood sugar. If you control your blood sugar you’ll be able to play again. If not controlled – may end in DKA, hypoglycaemia and serious consequences. Always be aware of hypoglycaemic symptoms: loss of conscious, sweating, heart racing, hungry. Since you might lose conscious it is important to carry MedAlert Caed or Bracelet which will clarify your situation. Please note that this is only a draft version based on several sources, including: Dr. Basel Mohasen’s lectures, Toronto Notes, Therapeutic Choices and others. Edited and organized for the sake of all attendances of the Canadian Osce Exams: NAC OSCE and MCCQE2. by: Dr. Merlyn D’Souza and Dr. Zeev Gross, Spring 2011. Page 31

Medical Error, Wrong blood transfused When there is a mistake, always there is a kind of unintentional medical error. (to the nurse) when informed about wrong blood – ask: ―did you stop the blood?‖ say: ―Well done!‖ If she asks not to tell the patient...ask her what her believe she may loose her job, and it is too early to determine who is responsible. Errors take place in medical practice. We don’t know what exactly happened. We will stabilize patient and ensure he’s fine and later deal with this issue. (to the patient) Intro: I am the doctor in charge, and it looks like it was an unintentional medical error took place. We need to make sure you are stable. We don’t know who is responsible, there are at least 15 steps and in each step could have been an error. We will fill an incident report and as soon as we get result we will inform you. You can sue, it is your right at the moment it is my priority to stabilize you. ABCD A – Open your mouth (check for anaphylaxis, no swelling in mouth, ask for any itchiness, or difficulty breathing), Oxygen saturation. Normal air entry. Normal S1, S2 Vitals again Remove blood unit and keep cannula. Start new IV line. Once new line, don’t give fluids if stable. Send blood: CBC, Lytes, INR, PTT, LFT, Cr, BUN, FDP, Haptoglobulin, Direct coombs test; Urinalysis: hemoglobulinuria Unit to be sent to blood bank for cross matching. Ask nurse to call the blood bank and keep original blood. D D1 – I’d like to shine a light in your eyes. Pupils are round, active, and symmetrical. Squeeze my finger, wriggle...wriggle... D2 – (if febrile) give tylanol Please prepare for me benedril (Diphenhydramine) 50mg. Steroids (Hydrocortisone) and Epinephrine Secondary survey Hx (two parts:) condition (how is he feeling now) and the other is: ―Why blood was given?‖ Condition: Do you feel warm? Chills? Itchiness? Tinglings? Diffculty breathing? Wheezing? Swelling in lips / fingers? Hives? Before transfusion did you have fever? Check for haemolytic reaction – any back or flank pain? P/E – no oozing at IV line Then press on flank and back – no pain for haemolytic reaction. Is it the first time? Why did you receive blood? If received blood before – was there any complications? Any long term diseases? Please note that this is only a draft version based on several sources, including: Dr. Basel Mohasen’s lectures, Toronto Notes, Therapeutic Choices and others. Edited and organized for the sake of all attendances of the Canadian Osce Exams: NAC OSCE and MCCQE2. by: Dr. Merlyn D’Souza and Dr. Zeev Gross, Spring 2011. Page 32

Counseling Mr. X what do you know about blood transfusion? It is a life saving measure, and a lot of measures are taken to make sure it is safe. However, like any other medication with blood transfusion there could be side effects, and these side effects could be serious. The most common side effect is febrile reaction (3%), usually it is self limited and can happen again. Next time you receive blood we will give you tylanol. Anaphylactic reaction. It is a severe allergic reaction, and it is very serious and we cannot predict it. However, we have good measures to deal with it, and your symptoms make it less likely that you have had an anaphylactic reaction. The yhird reaction reaction is more serious and called haemolytic reaction. Usually happens when patients receive blood belonging to another blood group. The fact that this blood is same as your blood group, and the symptoms are not consistent with haemolytic anemia make it less likely that this is not the case here. The blood is sent to the blood bank and once results are back we will get final confirmation, we will able to reassure you.

Back to Content

Please note that this is only a draft version based on several sources, including: Dr. Basel Mohasen’s lectures, Toronto Notes, Therapeutic Choices and others. Edited and organized for the sake of all attendances of the Canadian Osce Exams: NAC OSCE and MCCQE2. by: Dr. Merlyn D’Souza and Dr. Zeev Gross, Spring 2011. Page 33

Febrile Seizure A child brought to the ER because of febrile seizure. Next 10m counsel him. He is stable. During the next few minutes I’ll ask you few questions, and after that I’ll go with you to see him. You should r/o meningitis. Educate, and what to do next time. Did you see him? (Started to shake. All over his body? Bite his tongue / roling up his etes / wet himself). After the seizure does he have any neurologic deficits. How long did it last, or did you come on your by his own or medcial staff. Did he stop seizing on his own or after medical interv. Is it the 1st time? Ask about fever? (if it started a week ago – did you seek medical assistance? Discharge? Did they give you any treatment? Did they give it to him or no?) Why! Some studies show you can treat OM without antibiotics. You should look for the reason not to give the antibiotics (negligence?). Is he having any vomiting? Skin rash? Coughing? Head to toe... If you find nothing – ask when he got his last shot? (up to 72 hours he can have fever). R/O meningitis, pneumonia. Any family history of febrile seizures, epilepsy BINDE (especially immunization). Counseling: Your child has condition called febrile seizure (FS). It is a condition that might happen from 6m to 60m. We don’t know exactly why – we believe it is a sudden change in the temp. This might lead to the seizure. This condition might happen again. Any time your child has fever – seek medical admition. Give tylanol and sponge to decrease his temp. Most of the children will outgrow this condition by the 6th year. They don’t recommend Diazepam because it might make him drowsy. If it stopped less than 5m or more than 5m including neurological symptoms seek ER immediately. Brochure.

“This is Dr. ... (immediately should introduce yourself). I am the Dr. In charge in the ER. I am calling that your child swallow medication. I know you are stressed, I need to take your phone number and address, and how far it is from the hospital.” Stay calm. Your son needs you, I am going to give you some instructions and you need to follow them. Is your son is alert or not? Is he conscious? Can he talk to you? Can he recognize you? If he doesn’t – do you know how to do CPR and start with that. He’s crying. What is colour? Pink. Hold him and try to calm and sooth him. If he his conscious – try to hold him and check his mouth. Is he breathing? We’ll send the ambulance for you. When did it happen? How long was he alone? Which medications did he take? Do you have the container? (don’t go to the next room to bring them). Do you know what condition your father have (was it vitamins, sleeping pills, or any other?) how much the amount? Don’t use any ippecak? Is it happened before? What is the weight of the child? BINDE (was it full date, did he needed special attention after term, does he have any special conditions). Weight for two reasons – antidote and estimate neglect. Please note that this is only a draft version based on several sources, including: Dr. Basel Mohasen’s lectures, Toronto Notes, Therapeutic Choices and others. Edited and organized for the sake of all attendances of the Canadian Osce Exams: NAC OSCE and MCCQE2. by: Dr. Merlyn D’Souza and Dr. Zeev Gross, Spring 2011. Page 34

Post encounter Q: what are the first four steps you do when he arrives? (ABC, Monitor vitals, IV line, NG, Foley as needed, Blood works – CBC, Lytes, BUN, Cr, Osmolality, Coagul, LFTs, Tox screen – blood and urine). List three risk factors for this child. What is the antidote for betablocker (glucagon) and for CaChannel is (Calcium gluconate). CAS and Poisoning centre. Second scenario – while he is seizing just put him on the side, and not start any CPR. Before I proceed I’d like to take your phone number and address. Is it the same time or happened before. If it is the second time – more than 15m he needs intervention. Is he seizing right now? Try to put him on the floor on the left side (the right bronchus is shorter than the lt.). Observe him. What is his colour? Is he still shaking? You send the ambulance. Can you tap on his shoulder? If he is not responding – can you do CPR? Can you feel his pulse? Is he alert? Can he talk to you? Can he move his legs? Was he shaking? Does your child have fever? Did you seek medical attention? What prevented you from giving the medication? Post Concussion. 2 scenarios (Osgood schlatter and Post-concussion) Decision will based whether the child can tolerate pain or not? #1 About to see the father of 14yom with Osgood Schlatter. Make sure that the child best interest are preserved. What was done to diagnose the child? OCD PQRST compare to the other knee, is the first time or not, was any trauma. What is the child wish? (Don’t go for HEADDSSS since it is the father). Counseling What is your understanding of OS. Let me explain to you what is the mechanism for OS. Avoid him from playing, especially jumping. But he can continue with ice presses and pain killers. The rule is that he can continue up to his limit of his pain.

Back to Content

Please note that this is only a draft version based on several sources, including: Dr. Basel Mohasen’s lectures, Toronto Notes, Therapeutic Choices and others. Edited and organized for the sake of all attendances of the Canadian Osce Exams: NAC OSCE and MCCQE2. by: Dr. Merlyn D’Souza and Dr. Zeev Gross, Spring 2011. Page 35

54 year old female comes to clinic concerning about using HRT. . When a patient has concern about any subject, address it very soon. Don't wait to the end. Dr: As far as I understand you're here as you have concern about using HRT. Patient: yes Dr. I feel I am confused about using HRT. Always ask what do you mean by HRT. So the patient will tell you how much they known about HRT. Dr: I'm glad you're here so we can discuss about it and address your concerns and hopefully by the end of the session you can make a decision regarding using HRT. Or hopefully by doing this discussion you will have a better understanding of HRT. Or you can say: I agree with you as there are a lot of confusion about HRT and the reason for this confusion is that in the past because it was used to be given routinely to all women who reach a certain age, however 10 years ago there was study called " women health initiative" in which the authors found that the numbers of the patients with serious side effects are very high. However those ladies used HRT for a long time.

Serious side effects are Cancer, Heart attacks and Strokes. For that reason the routine use of HRT was stopped. Nowadays we have a better understanding and have better guidelines. Not only that we do it on the individualized basis. We use it only for short time, they don't exceed five years. So using HRT within five years is safe. So I would take some information from you and we will discuss about the risk factors and if you are a good candidates we can make a decision to prescribe it or not. Dr: What makes you interested in HRT? Patient: because of hot flushes.

At this stage if the patient gives you the symptom, it is your chief complaint. But if patient doesn't give you any symptoms, you should start with her LMP If she starts with the symptom of hot flushes, ask the patient 1. When did hot flushes start, 2. Is it all the time, 3. On & off or continues, 4. How many attacks, 5. Day or night, 6. How do you feel that you have it. 7. Night episodes, you have any night sweating, does it wake you up. Asked patient if the hot flushes wake her up during the night and if she needs to change her gown of night’s sweats. 1. 2. 3. 4.

Affect your sleep and how does it affect your concentration. Change in your mood, anybody has told you that your short tempered, and if you feel tired. Some women with the same symptoms may notice some change in their sexual life. a) So the doctor should ask with whom do you live? b) Are you sexually active? c) Any dryness or pain during the intercourse? 5. Any change in your urination? 6. Have you ever lost control? 7. Last period? Please note that this is only a draft version based on several sources, including: Dr. Basel Mohasen’s lectures, Toronto Notes, Therapeutic Choices and others. Edited and organized for the sake of all attendances of the Canadian Osce Exams: NAC OSCE and MCCQE2. by: Dr. Merlyn D’Souza and Dr. Zeev Gross, Spring 2011. Page 36

 Are you periods regular or not?  If it's irregular, when did it start to become irregular?  Are your periods heavy or not?  Any clots?  *Any bleeding or spotting between periods? This is a very important point. 8. Bone pain? Any fractures? Any family history of osteoporosis? If yes, tell the patient that you will discuss this in another meeting. Because that's another session to discuss about using steroids, smoking, alcohol, caffeine, warfarin and diet. If she takes calcium supplements.

MGOS for GYN cases: Menstural, Gynecologic diseases, Obstetrics, Sextually transmitted disease Dr: any history of gynecological disease like polyps, cysts, any pelvic intervention/instrumentation, surgeries. Dr: did you use to take any oral contraception? If yes, which one and did you have any side effects? Also you should ask about her last smear. Because she is 50+ you should ask about her mammogram. At any age you ask about Pap smear, once you reach 50 to ask about mammogram and when the patient pass 65 you should add bone density. You can ask about her obstetrics history, like have you ever been pregnant if yes how many times you have been pregnant? Now use the transition... Because this is the first time I met you, I would like to ask you about your past medical and social history. Is there any long-term disease, hospitalization before, any surgery, diabetes, or hypertension. Any history of allergy, and the medication she takes. ABCD: Active liver disease, vaginal Bleeding, Cancer, DVT For A you ask about any history of Active liver disease. Have you ever been yellowish? Any dark urine or pay stool? For B you should ask about any vaginal Bleeding? ... You have already asked these question before For C you should check about Cancer. I would like to ask about constitutional symptoms here to see if there is any endometrial cancer. Fever, chills, weight loss, appetite, lumps & bumps. A history of cancer in yourself or family (breast cancer, endometrial cancer,and colon cancer). For D you should ask about any history of swelling in the legs (DVT), any history of heart attacks, pulmonary embolism or stroke. Social history: smoking, taking alcohol, recreational drugs, how does she support financially herself, how does this affect her life and ask about osteoporosis. Usually in this set of scenario, you tell her on the basis of the history you are good candidates for HRT. However as I told you it is an important information to tell you to make your decision. As we go through different stages of life usually for ladies, we go to the stage called menopause which is vary between person to person. At this stage there is hormonal changes and ovaries start to produce less hormones specialty estrogen and progesterone and that changes affect the whole body. It can explain about dryness, decreasing or absence of periods. And that's why we try to replace those decreased hormones by HRT. They are the same hormones but we give it through external sources either tablets or skin patches. Please note that this is only a draft version based on several sources, including: Dr. Basel Mohasen’s lectures, Toronto Notes, Therapeutic Choices and others. Edited and organized for the sake of all attendances of the Canadian Osce Exams: NAC OSCE and MCCQE2. by: Dr. Merlyn D’Souza and Dr. Zeev Gross, Spring 2011. Page 37

As I told you before there is a balance it's your decision to make. And the balance is to use it up to five years. Using more than five years would increase the risk of stroke, heart attack or some cancers depending on what we call it estrogen dependent that includes breast and endometrial cancer. And some studies showed that it might increase the risk of Alzheimer's disease. So the risk of use for less than five years is not significant and still acceptable. So if you want to use it the shorter the better. To get rid off the hot flushes that are other measures like exercise or herbal supplements that you can try to improve the symptoms.

The HRTs are the same as OCP's but in this smaller doses and you can take one tablet a day. They have a few side effects like weight gain, bloating, nausea, abdominal distention and pain but they improve by time. This serious side effects are headaches, swelling of the legs or chest tightness which whenever happen you should go to emergency room. By using these HRT's your periods may stop or you may see spottings. If the patient had hysterectomy before you only give estrogen without progesterone, otherwise you should give both. Because you take it regular shootout regular ultrasound scans to check the thickness of the endometrium

Back to Content

Please note that this is only a draft version based on several sources, including: Dr. Basel Mohasen’s lectures, Toronto Notes, Therapeutic Choices and others. Edited and organized for the sake of all attendances of the Canadian Osce Exams: NAC OSCE and MCCQE2. by: Dr. Merlyn D’Souza and Dr. Zeev Gross, Spring 2011. Page 38

CARDIO 1.45 yr old Chest Pain x 45 mins ER History x 5 mins (MI) Onset: 1. When did it start? 2. What were you doing at that time? 3. How did you get here today? 4. If you came in by Ambulance, did the paramedics give you a tablet to be kept under your tongue? Course: Was it sudden or gradual? Position: Where exactly is it hurting you the most? Quality: Can you describe the pain? Is it crushing? Knifelike?....... Radiation: 1. Does it move anywhere else in your body 2. Does it move to the back? Severity: On a scale of 1 to 10 where one is minimum & 10 is highest, where would you place this pain? Associated symptoms: CVS:  N/V,Sweating?  Heart racing?  SOB/Orthopnoea/PND?  Have you been under stress recently?  Cough with blood tainted sputum? GI  Acid taste in mouth?  Heart burn?  Dysphagia  Pud? MSK  Have you had any trauma to the chest  Are there any blisters on chest? RS  Did you have any flu recently?  Cough with Phglem? CONSTITUNIOL Sx  Do you have night sweats  Loss of appetite & Loss of wt? Please note that this is only a draft version based on several sources, including: Dr. Basel Mohasen’s lectures, Toronto Notes, Therapeutic Choices and others. Edited and organized for the sake of all attendances of the Canadian Osce Exams: NAC OSCE and MCCQE2. by: Dr. Merlyn D’Souza and Dr. Zeev Gross, Spring 2011. Page 39

Alleviating Fcts: What makes you feel better? 1. Rest? 2. GTN? 3. Antacid? 4. Sitting forward? Aggravating Factors: 1. Exercise/exertion? 2. Movements? 3. Deep inspiration? 4. Lying down? 5. Eating? RISK FCTS: I need to know additional information that could be related to your pain right now, and need to ask some further questions...... 1. Do you have a high Blood pressure?  When,& what did your doctor have to say about it?  Were you put on medicatn? 2. Were you diagnosed at any time with an elevated Blood Sugar or were told you had diabetes?  When,& what did your doctor have to say about it?  Were you put on medication? 3. Have you ever had your cholesterol checked? If yes:  When,& what did your doctor have to say about it?  Were you put on medication? 4. Do you smoke? If Yes; How many & Since how long? 5. Do you take alcohol 6. Have you used recreational drugs? Cocaine? 7. .Do you find time for regular physical activity? 8. Do you eat a lot fast food? 9. In your family has anyone had a heart attack under the age of 50? CAUSES/Consequences & Symptoms: 1. Did you ever have a stroke? Please note that this is only a draft version based on several sources, including: Dr. Basel Mohasen’s lectures, Toronto Notes, Therapeutic Choices and others. Edited and organized for the sake of all attendances of the Canadian Osce Exams: NAC OSCE and MCCQE2. by: Dr. Merlyn D’Souza and Dr. Zeev Gross, Spring 2011. Page 40

2. Did you have a weakness or numbness? Past Medical History: 1. Do you take any medications?/OTC or herbal products? 2. Are you allergic to anything? 3. Were you hospitalized at any time? 4. Did you ever undergo any surgery? 5. Were you ever diagnosed with Cancer? 6. Do you have a bleeding disorder? 7. Did you have any head/facial trauma since last 3 mo? I need some more information about your family HTN/DM/MI/Stroke Social History: 1. With whom do you live? 2. How do you support yourself? MANAGEMENT: 

Rapid, targeted history and physical examination, with particular attention to onset of symptoms, contraindications to use of thrombolytic agents

Absolute contraindications: 1. 2. 3. 4. 5. 6. 7.

Previous intracranial hemorrhage; Known malignant intracranial neoplasm, Known cerebral vascular lesion, Ischemic stroke within 3 mo EXCEPT acute stroke within 3 h; Suspected aortic dissection; Active bleeding or bleeding diathesis (excluding menses); Significant closed head or facial trauma within 3 mo.

Relative contraindications: 1. 2. 3. 4. 5. 6. 7. 8. 9.

History of chronic severe, poorly controlled HTN, Severe uncontrolled HTN (BP > 180/110 mm Hg)c; Prior CVA greater than 3 mo or known intracerebral pathology not covered above; Traumatic or prolonged (> 10 min) CPR or Major surgery (< 3 wk); Noncompressible venous punctures; recent (2–4 wk) internal bleeding; pregnancy; active peptic ulcer; current use of anticoagulants.

Please note that this is only a draft version based on several sources, including: Dr. Basel Mohasen’s lectures, Toronto Notes, Therapeutic Choices and others. Edited and organized for the sake of all attendances of the Canadian Osce Exams: NAC OSCE and MCCQE2. by: Dr. Merlyn D’Souza and Dr. Zeev Gross, Spring 2011. Page 41

and evidence of high-risk features (tachycardia, hypotension, congestive heart failure)

Management 1. ECG STAT, then every 8 hours for the first 24 hours, then daily for 3 days. a. In addition, repeat the ECG with each recurrence of chest pain 2. Baseline troponin STAT, (creatine kinase if troponin is unavailable) and then every 8 hours until enzymatic confirmation of the diagnosis 3. CBC to rule out the presence of anemia, 4. Baseline electrolytes, 5. Creatinine, 6. Fasting lipid profile (within 24 hours of presentation) 7. Liver function tests 8. Portable chest x-ray (CXR) STAT 9. Echocardiography to assess LV function after stabilization and treatment. Echocardiography is also used emergently when there is suspicion of acute mechanical complications post-MI

Therapeutic Tips    



   

The goal for thrombolytic treatment is a door-to-needle time of 30 minutes or less. The goal for primary PCI is a door-to-dilatation time of 90 minutes or less. Careful attention to maximum pain relief is important. In patients with right ventricular infarcts: o avoid nitrates and diuretics o use fluids and inotropes to treat hypotension Administer beta-blockers early to all patients without contraindications. Increase the dose every 12 hours (every 24 hours for once-daily beta-blockers), if tolerated (monitor blood pressure and heart rate), until the patient has reached adequate beta-blockade (HR ≤ 5565 BPM). Start ACE inhibitors early. The choice of agent can depend on practitioner preference, hospital formulary or financial constraints for the individual patient. In smokers, the need to quit smoking should be reinforced early (within 24 hours) and frequently. Stool softeners are often used in the immediate post-MI period to prevent straining with bowel movements. Anxiolytics are often used on an as-needed basis in the immediate post-MI period.

Early Management of STEMI

Please note that this is only a draft version based on several sources, including: Dr. Basel Mohasen’s lectures, Toronto Notes, Therapeutic Choices and others. Edited and organized for the sake of all attendances of the Canadian Osce Exams: NAC OSCE and MCCQE2. by: Dr. Merlyn D’Souza and Dr. Zeev Gross, Spring 2011. Page 42

Please note that this is only a draft version based on several sources, including: Dr. Basel Mohasen’s lectures, Toronto Notes, Therapeutic Choices and others. Edited and organized for the sake of all attendances of the Canadian Osce Exams: NAC OSCE and MCCQE2. by: Dr. Merlyn D’Souza and Dr. Zeev Gross, Spring 2011. Page 43

Hx OCD PQRST (if it is suspected to be ACS - stop at R and start primary survey) Primary Survey (If patient talks – Airway preserved, take Oxygen saturation and start Oxygen Stat – 4L/m through nasal prongs) Vitals Auscultation: normal air entry and normal S1, S2 IV lines (normal NaCl 50ml/hr to keep line open, from the other side take blood for: Troponin, CK-MB, Cr, BUN, Lytes, CBC, INR, PTT, LFT, Toxic., Alcohol, Lipids; and finger prick for Glucose) ECG 12 leads and continue monitoring Ask about Allergy and Viagra (if negative) Give ASA chewable (325mg) Non-ST elevation: give Nitro x3 (S.L) if there is no benefit – give Morphine. Contin Please note that this is only a draft version based on several sources, including: Dr. Basel Mohasen’s lectures, Toronto Notes, Therapeutic Choices and others. Edited and organized for the sake of all attendances of the Canadian Osce Exams: NAC OSCE and MCCQE2. by: Dr. Merlyn D’Souza and Dr. Zeev Gross, Spring 2011. Page 44

PQRST AA&A How do you feel now? Ask Hx on CVS and GI (especially peptic ulcer) CSx RS DVT ST Elevation: do not go for DDx, Vitals (again) RF Nitro (2nd dose) Examination: JVP Listen to heart Base of lung Compare BP in both Upper extremities to r/o coarctation of Ao CXR Once there is no Aortic Dissection  Thrombolytics (should be clear to r/o: Peptic ulcer, recent surgery, pericarditis, aortic dissection, brain tumor, and stroke) Based on ECG – counselling Counseling Based on your ECG it is most likely you are having an heart attack. If stable – BP and HR are stable, but it is a serious condition, however it is treatable. Heart attack means that greater than one blood vessel supporting your heart is blocked by a clot that has to be reimoved. The medications are called clot busters. Based on ECG and no sign of pericarditis or signs of aortic dissection you are a good candidate for treatment. It is an effective medication, needs consent. 1% chance of stroke and we can start heparin.

Back to Content

Please note that this is only a draft version based on several sources, including: Dr. Basel Mohasen’s lectures, Toronto Notes, Therapeutic Choices and others. Edited and organized for the sake of all attendances of the Canadian Osce Exams: NAC OSCE and MCCQE2. by: Dr. Merlyn D’Souza and Dr. Zeev Gross, Spring 2011. Page 45

PALPITATIONS A.37 M/6wks [H&PE] B.30F/4wks[H&C] Dd: VITAMINS C VASCULAR: SVT,Rapid atrial fibrillation,& V Tach METABOLIC:Fever,Anemia,Hyperthyroidism,Acromegaly NEOPLASTIC: Pheochromocytoma SUBSTANCE ABUSE & PSYCHIATRIC DRUG INGESTION (sympathomimetic) Drug Withdrawl,Anxiety CONGENITAL:WPW Syndrome >48 hrs not panic attack OCD: O: When did it start? How did it Start? Sudden/Gradual What were you doing at that time? C: Does it come in bouts or Continous? How often does it occur? What was the duration of the attack? D: How long since you’ve had these palpitations? How long does each episode last? / ? > 48 hrs? PQRSTUV Q: Ask Pt to tap with his fingers the heart beat. Does it Miss abeat/Racing/Slowing of heart beat? S: On a scale of 1 – 10 How has it affected the quality of your life? T: Does it occur even at night?Is it the first time or has it happened before? Emphatize: I know it can be a fairly scary feeling AA&A A: What makes it worse:Coffee/Recreational drugs/Stress/Smoke(extrasystoles)Choclates/Alcohol A: Anything makes it better? How was your health prior to the palpitations? ASOC Sx: Please note that this is only a draft version based on several sources, including: Dr. Basel Mohasen’s lectures, Toronto Notes, Therapeutic Choices and others. Edited and organized for the sake of all attendances of the Canadian Osce Exams: NAC OSCE and MCCQE2. by: Dr. Merlyn D’Souza and Dr. Zeev Gross, Spring 2011. Page 46

CVS: Chest Pain/SOB/Orthopnea/PND/Dizziness/Sweating/N/VSwelling of feet/Cough CNS: Weakness/Vision loss/Difficulty in finding words/Numbness or loss of sensation THYROID: Do you feel hot/cold Do you have wt loss inspite of increased appetite?/Tremors?Shakiness?Sweaty palms & moist skin? PHEOCHROMOCYTOMA: Repeated headaches,with increased sweating CARCINOID:Flushing/Diarrhoea CNS:In last few weeks did you notice any difficulty in Walking,numbness,finding words? (Look for Sx/o Embolism) CONSTITUTIONAL Sx: Fever/Chills/Loss of appetite/Lumps & Bumps RISK FCTS: Smoking/Alcohol (Holiday Heart)/Recreational drugs PMH:    

Are you on any medications? Have you seen a psychiatrist? Any OTC/Herbal products/Cold meds/Asthma meds Are you allergic to anything? If Allergic to Penicillin: 1. When did you find out? 2. Where did you take it? 3. Why did you take it? Do you have any Heart disease/HTN/DM/Stroke/Ca/MI H/O Rheumatic fever as a child? Did you get repeated sore throat infections as a child? Did you receive Penicillin injections regularly as a child?

Back to Content

Please note that this is only a draft version based on several sources, including: Dr. Basel Mohasen’s lectures, Toronto Notes, Therapeutic Choices and others. Edited and organized for the sake of all attendances of the Canadian Osce Exams: NAC OSCE and MCCQE2. by: Dr. Merlyn D’Souza and Dr. Zeev Gross, Spring 2011. Page 47

HTN(GAO Pg:95) History: OCD o * * o * * o

o o

O: When did you notice your BP was high? When was your last (N) BP Duration of hypertension, Usual level of blood pressure and Any sudden change in severity of hypertension History of antihypertensive drug use, * Reason for changing therapy, * effectiveness, * side effects and intolerance (IMPOTENCE) Drugs that may cause hypertension drugs that may interact with antihypertensive drugs (those that induce or inhibit metabolism) Adherence with lifestyle recommendations and drug therapy HOME MONITORING

END ORGAN DAMAGE: 1. Angina/Mi: Chest pain/SOB 2. TIA/Stroke:LOC/Vision changes/weakness/Numbness 3. Peripheral Vascular disease/Leg pain/ED/ 4. Kidney disease SX of SECONDARY HTN, , 1. Pheochromocytoma (hyperadrenergic symptoms)Do you have episodes of palpitations/HA/Sweating? 2. Hyper- and hypothyroidism: Feel Hot/Cold/Tremors 3. Cushing’s syndrome: Bruising of skin/Wt gain 4. Renal/urinary symptoms or a past history of renal disease RISK FCTS: 1. Cigarette and alcohol use, 2. Usual physical activity 3. Usual diet and sodium intake, 4. Current weight and recent weight change, waist circumference, 5. Diabetes 6. Dyslipidemia PAST MED HISTORY 1. Medications Pt is on/OTC/Herbal 2. Allergies 3. Hospitalizations/Surgery FAMILY HISTORY  Hypertension, Please note that this is only a draft version based on several sources, including: Dr. Basel Mohasen’s lectures, Toronto Notes, Therapeutic Choices and others. Edited and organized for the sake of all attendances of the Canadian Osce Exams: NAC OSCE and MCCQE2. by: Dr. Merlyn D’Souza and Dr. Zeev Gross, Spring 2011. Page 48

 

Cardiovascular risk factors Premature cardiovascular disease SOCIAL HISTORY

Nonpharmacologic Choices Effect of Lifestyle Changes on Blood Pressure in Adults with Hypertension Change in Blood Pressure (systolic/diastolic) mm Hg

Intervention

↓ by 1800 mg (78 mmol) per day

−5.8/−2.5

2. Weight loss

4.5 kg

−7.2/−5.9

3. Reduction in alcohol

↓ by 2.7 drinks/day

−4.6/−2.3

4. Exercise

3 times/week

−10.3/−7.5

5. Dietary

DASH dieta

−11.4/−5.5

1. Reduction in sodium intake

intake

recommendations 

Weight loss of 4 kg or more if overweight (target body mass index: 18.5 to 24.9 kg/m 2; waist circumference 50 years old to screen for carotid sinus hypersensitivity (do not perform in patients with carotid bruits) o tilt table testing might be useful in diagnosing vasovagal syncope in patients with atypical symptoms After potentially fatal causes are eliminated and reversible causes are removed, most patients will have one of several syndromes of orthostatic intolerance:3 o reflex syncope syndromes  vasovagal syncope  carotid sinus hypersensitivity in the elderly o pure autonomic failure syndromes o multiple system atrophy syndromes The orthostatic intolerance syndromes can be distinguished based on history and a simple stand test in the office.

To perform the stand test, first measure blood pressure and heart rate after the patient has been supine for 5 minutes, then after 2 and 4 minutes of standing. These responses are seen: Normal and vasovagal syncope: modest rises in heart rate (about 10 BPM) and blood pressure (about 10 mm Hg). Autonomic failure: progressive fall in blood pressure of ≥20 mm Hg systolic or ≥10 mm Hg diastolic with development of presyncope; often no increase in heart rate.

Please note that this is only a draft version based on several sources, including: Dr. Basel Mohasen’s lectures, Toronto Notes, Therapeutic Choices and others. Edited and organized for the sake of all attendances of the Canadian Osce Exams: NAC OSCE and MCCQE2. by: Dr. Merlyn D’Souza and Dr. Zeev Gross, Spring 2011. Page 51

Therapeutic Choices Treatment is directed at the cause of syncope. Treat any reversible causes. Refer patients with syncope secondary to bradycardia (asystole or complete heart block) for a permanent pacemaker. Refer patients with suspected or diagnosed ventricular tachycardia, and all patients with structural heart disease to a cardiologist, preferably an electrophysiologist. The following addresses treatment of syndromes of orthostatic intolerance.

Nonpharmacologic Choices 





Reassure the patient that this syndrome is not life threatening and that it is a physical problem, not a psychiatric disorder. Encourage increased dietary salt intake of about 3–5 g daily, in the absence of contraindications such as hypertension or heart failure.5 , 8 Teach the patient to use physical counterpressure manoeuvres at the onset of presyncope.9 , 10 These include squatting, crossing the legs with isometric contraction if standing, and vigorous hand clenching with upper girdle isometric contraction. All should be tried. The evidence is based on a good physiologic study10 and an open label randomized clinical trial.9 Pacemaker therapy is no longer indicated, based on the results of an adequately powered randomized placebo-controlled trial.11 The occasional patient with asystole documented during vasovagal syncope might benefit, and these uncommon patients should be assessed at a tertiary referral cl

Back to Content

Please note that this is only a draft version based on several sources, including: Dr. Basel Mohasen’s lectures, Toronto Notes, Therapeutic Choices and others. Edited and organized for the sake of all attendances of the Canadian Osce Exams: NAC OSCE and MCCQE2. by: Dr. Merlyn D’Souza and Dr. Zeev Gross, Spring 2011. Page 52

65/F Calf Pain x 10 weeks How many blocks you could go? How many now? How fast the pain disappears after resting? Is the pain alleviated by bending forward or extending backward? Is it awakening you at night? Ddx: 1. 2. 3. 4. 5. 6.

Spinal Stenosis (Pain disappears about 15min after resting, alleviate by leaning forward) Disc herniation (Pain disappears about 15 min after resting, alleviated by extending) Intermittent Claudication (After resting – pain disappears by few minutes) PE Cellulitis Ruptured Baker’s cyst

CC: Unilateral Vs (B)

O: Can you tell me when it all started? Sudden Vs Gradual What were you doing at that time? What made you come in today? C: Is it Increasing,decreasing or same? Has the intensity increased?

*Does it awaken you up at night? D: How often does each episode occur? How long does it last? PQRSTUV: R:Does it move anywhere else in the body? Buttock/Toes/Feet/Thigh T:When does it come on? When you walk? How many blocks can you walk when it comes? ------ >Now & at the beginning? Reproducible pain U V:  Has it happened before?  If Yes When?  How Often? AAA: AGGRAVATING: Stand/Sit? Please note that this is only a draft version based on several sources, including: Dr. Basel Mohasen’s lectures, Toronto Notes, Therapeutic Choices and others. Edited and organized for the sake of all attendances of the Canadian Osce Exams: NAC OSCE and MCCQE2. by: Dr. Merlyn D’Souza and Dr. Zeev Gross, Spring 2011. Page 53

When you walk uphill or downhill? When you raise your leg? ASSOC. SX: CONSTITIONAL Sx: Fever/ /night sweats/Loss of appetite & los of Wt/Lumps & bumps. Local Sx: 1. Swelling/Raised temp 2. Back Pain or Trauma to back or knee/Morning stiffness 3. Numbness/Tingling/Weakness/Burning sensation 4. Change in nails/Hair loss/Skin is it shiny/any Ulcers? 5. Are your feet cold? CARDIAC: Chest pain/Palpitations/SOB *How is your sex life? Desire & Erection?........ How has this affected your life? RS: Cough RISK FCTS: I need to ask you some more qns that will help me arrive at a diagnosis of your pain: 1. 2. 3. 4.

Do you Smoke? Drink Alcohol? Have you recently travelled a long distance in an airplane? Were you at any time Diagnosed as HTN,Is it Rxed & Under control? When was the last time you saw your Dr.? 5. Were you at any time Diagnosedwith high blood sugar?Is it Rxed & Under control? When was the last time you saw your Dr.? 6. Have you checked your cholesterol? ....... 7. Did you have recent surgery?

PMH: I need some information about your health in general:  Are you on any meds?/OTC/Herbal products?  Allergies?  Were you ever hospitalized? FHx SOCIAL:  Who lives with you?  How do you support yourself financially?

Back to Content

Please note that this is only a draft version based on several sources, including: Dr. Basel Mohasen’s lectures, Toronto Notes, Therapeutic Choices and others. Edited and organized for the sake of all attendances of the Canadian Osce Exams: NAC OSCE and MCCQE2. by: Dr. Merlyn D’Souza and Dr. Zeev Gross, Spring 2011. Page 54

ANKLE SWELLING 30 M x 10 days A. Gout B. CHF Dd: A. 1. 2. 3. 4.

UNILATERAL Trauma Arthritis: Gout/SepticA Cellulitis Varicosities

B. 1. 2. 3. 4. 5.

BILATERAL CHF Nephrotic Liver failure Myxoedema Protein losing enteropathy

First Qn: Unilateral or Bilateral OCD: O; Sudden/Gradual Off & On/Continous Everyday/Certain time of day Related to activity/Standing PQRSTUV: P:Above kne/Below knee Posture AA&A What makes it worse: Activity/Alcohol & diet(Gout)/Standing Alleviating fcts:Rest with elevation of feet.... ASSOC Sx: LOCAL Sx: In addition to the swelling did you notice any: Pain/Stiffness/Fullness/Redness Did you notice swelling anywhere else? Face/Eyes particularly did you feel your eyes were puffy in the morning/Increasing waist size/Rings are tighter? CONSTITUTIONAL Sx: Fever/Wt loss/Night sweats/lumps & bumps CARDIAC Sx: Chest Pain/SOB/Heart racing GI/LIVER Disease:Yellow discoloration of skin/Pale stools/Dark urine KIDNEY DISEASE:Change in the amount of: Urine/Color/Consistency (Frothy/Cloudy)/Odour Please note that this is only a draft version based on several sources, including: Dr. Basel Mohasen’s lectures, Toronto Notes, Therapeutic Choices and others. Edited and organized for the sake of all attendances of the Canadian Osce Exams: NAC OSCE and MCCQE2. by: Dr. Merlyn D’Souza and Dr. Zeev Gross, Spring 2011. Page 55

RISK FCTS: I need to ask you some more details to get more insight into your condt: Do you Smoke,Take Alcohol or Recreational drugs? What is your diet like? Do you eat a lot of red meats? (GOUT) PMH:

*

Any meds you’re currently taking?( Aspirin & Thiazides for gout)/OTC/Herbal products? Have you ever been diagnosed with HTN/DM/MI/Stroke/Ca? Were you ever hospitalized or undergone any surgical procedure? FH: Does anyone in the family have a similar condt? DM/HTN/MI/Stroke? SOCIAL H: Who lives with you How do you support yourself financially?

Back to Content

Please note that this is only a draft version based on several sources, including: Dr. Basel Mohasen’s lectures, Toronto Notes, Therapeutic Choices and others. Edited and organized for the sake of all attendances of the Canadian Osce Exams: NAC OSCE and MCCQE2. by: Dr. Merlyn D’Souza and Dr. Zeev Gross, Spring 2011. Page 56

24 female, acute asthma in ER, 3 dasys ago,asthmatic for the last 3yr comes to you at clinic for F up As I understand you were in ER 3 days ago with an attack of Ac asthma. HOW DO YOU FEEL RIGHT NOW? Event – Before Event After OCD Can you tell me what happened at that time? SX: 1. Sudden/Gradual 2. Was there Wheezing? 3. Chest tightness? 4. Were you able to talk? 5. How many times did you use the puffer? 6. How did you get to the ER? 7. What did they do in the ER? 8. Were you intubated? 9. What medicines did they give you?

Asthma history Let us talk about Asthma history: 1. When were you diagnosed? 2. How were you diagnosed? 3. Are you on regular f/Up? 4. When was the last time you were seen at F/up? 5. Have you visited the ER before? 6. Did you notice any increase in nos of attacks? 7. Do you have attacks at rest? Attacks at Night? 8. Did you dr adjust your meds at that time? 9. Which meds do you use? 10. How often do you have to use your medicines? 11. Are you using the meds more frequently? 12. Triggers 13. Do you suffer from heart burn or condt called GERD? 14. Do you user a peak flow meter? I need to ask more qns which will help me to clarify as to why you had an attack recently? 1. Have you had recently any flu/infection? (any chest infection upto 10 wks post infection hyperreactive airways) Please note that this is only a draft version based on several sources, including: Dr. Basel Mohasen’s lectures, Toronto Notes, Therapeutic Choices and others. Edited and organized for the sake of all attendances of the Canadian Osce Exams: NAC OSCE and MCCQE2. by: Dr. Merlyn D’Souza and Dr. Zeev Gross, Spring 2011. Page 57

2. Can you show me how you take medication? (Shake it, put it in your mouth, take deep breath when puffing). 3. Do you make sure your medication are not expired and stored expired? 4. Did you started any new medication that might interfere (beta blocker / aspirin)with your asthma? 5. Outdoor – cold weather, pollens, exercise, construction, dust 6. Indoor – a. Do you smoke or anyone around you, b. Pets or people around, c. New curtain, indoor plants, carpets, curtains, pillows. d. Basement – mould, renovations, paintings; 7. Relation to any type of food; 8. Strong odour Important to classify – mild / moderate / severe I want to ask you… What do you do for living? PAST H: Any HTN/DM Hospitalizations/Surgery FAMILY H: Same Condt DM/HTN/Stroke SOCIAL H Do you take alcohol/recreational drugs? How do you support yourself financially?

Please note that this is only a draft version based on several sources, including: Dr. Basel Mohasen’s lectures, Toronto Notes, Therapeutic Choices and others. Edited and organized for the sake of all attendances of the Canadian Osce Exams: NAC OSCE and MCCQE2. by: Dr. Merlyn D’Souza and Dr. Zeev Gross, Spring 2011. Page 58

ER Rx of Ac Asthma

Please note that this is only a draft version based on several sources, including: Dr. Basel Mohasen’s lectures, Toronto Notes, Therapeutic Choices and others. Edited and organized for the sake of all attendances of the Canadian Osce Exams: NAC OSCE and MCCQE2. by: Dr. Merlyn D’Souza and Dr. Zeev Gross, Spring 2011. Page 59

Asthma Treatment Very mild, intermittent asthma may be treated with fast-acting beta2-agonists taken as needed. Inhaled corticosteroids (ICS) should be introduced early as the initial maintenance treatment for asthma, even in individuals who report asthma symptoms less than 3 times a week. Leukotriene receptor antagonists (LTRAs) are second-line monotherapy for mild asthma. If asthma is not adequately controlled by low doses of ICS, additional therapy should be considered. A long-acting beta2-agonist (LABA) should be considered first as add-on therapy only in combination with an ICS. Increasing to a moderate dose of ICS or addition of an LTRA are third-line options. Theophylline may be considered as a fourth-line agent in adults. Severely uncontrolled asthma may require additional treatment with prednisone. Omalizumab may be considered in individuals 12 years of age and over with poorly controlled atopic asthma despite high doses of ICS and appropriate add-on therapy, with or without prednisone. Asthma symptom control and lung function tests, inhaler technique, adherence to asthma treatment, exposure to asthma triggers in the environment and the presence of comorbidities should be reassessed at each visit and before altering the maintenance therapy. After achieving proper asthma control for at least a few weeks to months, the medication should be reduced to the minimum necessary to maintain adequate asthma control.

Short-acting Inhaled Beta2-agonists (SABAs) Salbutamol and terbutaline are selective beta2-agonists that are agents of first choice for treatment of acute exacerbations and for prevention of exercise-induced asthma. They are best used as required rather than on a fixed schedule. Although potent bronchodilators, they have little effect on the late (inflammatory) phase of an exacerbation. If patients use a short-acting beta2-agonist more than 4 times per week (including any doses used to prevent or treat exercise-induced symptoms), initiate therapy with 4 an anti-inflammatory agent. Isoproterenol and epinephrine are not recommended for the treatment of asthma because of lack of beta2-selectivity and potential for excessive cardiac stimulation, especially at high doses.

Long-acting Inhaled Beta2-agonists (LABAs) Salmeterol and formoterol are long-acting beta2-agonists for regular twice daily treatment of asthma. Salmeterol has a slow onset of action and should not be used for immediate relief of bronchospasm. Formoterol is rapid acting and can be utilized for rescue therapy. These drugs help to prevent exercise5 induced bronchospasm. Both should be used only in patients already taking inhaled corticosteroids and may be particularly useful for the prevention of nocturnal symptoms. Adding long-acting beta2-agonists to 6 inhaled corticosteroids may permit decreasing the latter’s dose.

This information was originally published in Canadian Respiratory Journal 2010;17(1):15-24.

Back to Content

Please note that this is only a draft version based on several sources, including: Dr. Basel Mohasen’s lectures, Toronto Notes, Therapeutic Choices and others. Edited and organized for the sake of all attendances of the Canadian Osce Exams: NAC OSCE and MCCQE2. by: Dr. Merlyn D’Souza and Dr. Zeev Gross, Spring 2011. Page 60

ACUTE COUGH Cough for the last 5d Local Cause Community acquired Pn HIV In young Pt can ask directly H/o HIV Status In HIV(Pneumocytis Jevorici there is (B) chest pain & night sweats OCD:UV O How did it start: Sudden/Gradual? C: Is it first time or have you had it before? Is it increasing/Decreasing or same intensity now as it was in the beginning? Does it wake you up from sleep? EMPATHY if awakens him up NATURE: Dry/Wet If Wet: COCA Color Odour Consistency Amount Blood CHEST PAIN: PQRST Constitutional symptoms RESPIRATORY Sx: 1. Shortness of breath, 2. Tightness, 3. Wheezing, 4. Ear pain 5. Sore throath CARDIAC Sx: SOB/ Heart racing Chest ain S/o Meningitis: RISK FCTS: 1. Recent contact with sick people, 2. Vaccination for flu 3. TRAVEL H 4. Smoking 5. Alcohol 6. Drug abuse Past Med H Any meds/OTC/Allergies Asthma/DM/Similar condt Please note that this is only a draft version based on several sources, including: Dr. Basel Mohasen’s lectures, Toronto Notes, Therapeutic Choices and others. Edited and organized for the sake of all attendances of the Canadian Osce Exams: NAC OSCE and MCCQE2. by: Dr. Merlyn D’Souza and Dr. Zeev Gross, Spring 2011. Page 61

Hospitalizations/Surgery FAMILY H SOCIAL H: Habits IF HIV + PT: DO you know your HIV diagnoses? * When? * Where? * Regular follow up? * Medication? Taking/or not? * Last CD4? * H/O Thrush? PAST MED H Allergy COUNSELLING: PE/ CXR & Bl work Admit today

Back to Content

Please note that this is only a draft version based on several sources, including: Dr. Basel Mohasen’s lectures, Toronto Notes, Therapeutic Choices and others. Edited and organized for the sake of all attendances of the Canadian Osce Exams: NAC OSCE and MCCQE2. by: Dr. Merlyn D’Souza and Dr. Zeev Gross, Spring 2011. Page 62

Cough for the last 6w Increasing, not improving DID you seek medical attention?, what make you come today? Is it the first time? OCD

O How did it start: Sudden/Gradual? C: Is it first time or have you had it before? Is it increasing/Decreasing or same intensity now as it was in the beginning? Does it wake you up from sleep? EMPATHY if awakens him up What made you come in today/ NATURE: Dry/Wet If wet first & then dry 2 elements: 1. Previous episode Sx: When productive:Fever/chills/Night sweats/ muscle pain & joint ache/COCA 2. Dry cough Sx: From that time till now,do you have fever?chills,muscle pains? Sx of infection? PATTERN OF COUGH: 1. Whole day? 2. How often? 3. How may attacks? 4. How long each attack? 5. Any particular time of the day?or 6. Do you wake up in morning with cough?(NIGHT COUGH: GERD/Asthma/CHF) (MORNING COUGH: PND/GERD) 7. When you cough do you cough to the extent that you’ve: a. SOB b. Difficulty talking c. Wheezing d. Chest pain e. Sweating f. (in children----> vomit) RISK FCTS: I need to ask some qns now that could lead me to the cause:  H/o repeated sinusitis  Facial pain  Ned to clear throath  Runny nose  GERD: H Burn Acid taste in mlouth Please note that this is only a draft version based on several sources, including: Dr. Basel Mohasen’s lectures, Toronto Notes, Therapeutic Choices and others. Edited and organized for the sake of all attendances of the Canadian Osce Exams: NAC OSCE and MCCQE2. by: Dr. Merlyn D’Souza and Dr. Zeev Gross, Spring 2011. Page 63

Relation to lying down/bending forward How many pillows do you need at night Do you get up in night gasping for breath/  ASTHMA; h/o Asthma Relation between cough & outdoors or indoors  SMOKING H: Self: How many/ How long Those around you  Pets What do you do for a living/ Any exposure to dust? Any perfumes? D/D Other causes of Ch Cough 1. Medications: a. HTN; ACE/Aspirin/NSAIDS 2. Swelling in legs 3. Rcent travel 4. Contact with Tb 5. H/o Lung Ca CHEST PAIN: PQRST Constitutional symptoms RESPIRATORY Sx: 1. Shortness of breath, 2. Tightness, 3. Wheezing, 4. Ear pain 5. Sore throath CARDIAC Sx: SOB/ Heart racing Chest ain S/o Meningitis:

Please note that this is only a draft version based on several sources, including: Dr. Basel Mohasen’s lectures, Toronto Notes, Therapeutic Choices and others. Edited and organized for the sake of all attendances of the Canadian Osce Exams: NAC OSCE and MCCQE2. by: Dr. Merlyn D’Souza and Dr. Zeev Gross, Spring 2011. Page 64

Back to Content

Please note that this is only a draft version based on several sources, including: Dr. Basel Mohasen’s lectures, Toronto Notes, Therapeutic Choices and others. Edited and organized for the sake of all attendances of the Canadian Osce Exams: NAC OSCE and MCCQE2. by: Dr. Merlyn D’Souza and Dr. Zeev Gross, Spring 2011. Page 65

67Male with Hemoptysis: Same check list as H Uria look for pulmonary Sx Intro: As I understand you’re here because you’ve cough x 1 week? Any chance you may be vomiting? OCD Duration: Night? COCA + Bl Sx of Hypovolemeia: Dizziness/Faint/Tiredness/Loc A&A ASx: RS CSx: Hoarse voice Risk Fcts; Smoking Contact with TB/Screened for TB Travel outside Canada Exposed to asbestos H/O Dvt,Calf pain,redness,swelling of calves H/O Hd: PND/Orthopnea H/o Blood thinners Bleeding from any parts of body CNS: PMH

Back to Content

Please note that this is only a draft version based on several sources, including: Dr. Basel Mohasen’s lectures, Toronto Notes, Therapeutic Choices and others. Edited and organized for the sake of all attendances of the Canadian Osce Exams: NAC OSCE and MCCQE2. by: Dr. Merlyn D’Souza and Dr. Zeev Gross, Spring 2011. Page 66

SOB/67 F x 6wks [5min/H] HF Dd: VITAMINS VASCULAR:CHF,ACS,PE Precipatants of CHF:  Meds: * Stopped * NSAIDS  Increased Na intake INFECTIONS:Pneumonia TRAUMATIC:Pneumothorax METABOLIC:DKA IDIPATHIC/IATROGENIC:COPD/Asthma/Massive atelactasis NEOPLASTIC: Large pleural effusion FIRST evaluate:ABC,ask pt if she is comfortable....... OCD: O:Sudden/Gradual What were you doing when you had this SOB? ---Exertion/Lying down? C: *SOB first always ask if difficulty in breathing is for: Breathing IN or Breathing OUT Does it occur all the time or only now Is it related to activity or does it occur even at rest If brought on by walking? How many blocks can you walk now as compared in the beginning? If at Rest? Do you sleep well? How many pillows do you need? Do you wake up at night gasping for breath? D: How long? If assoc Leg swelling,---- How long since leg swelling? PQRSTUV: S: On a scale of 1 – 10? T: Has it ever happened to you before?If so,how often? When was the last time you had SOB? How did you cope? Please note that this is only a draft version based on several sources, including: Dr. Basel Mohasen’s lectures, Toronto Notes, Therapeutic Choices and others. Edited and organized for the sake of all attendances of the Canadian Osce Exams: NAC OSCE and MCCQE2. by: Dr. Merlyn D’Souza and Dr. Zeev Gross, Spring 2011. Page 67

U: How has it affected your life & how do you manage? Do you have someone to help you? [EMPATHY] AAA AGGRAVATING FctS: Exercise POsition Exposure to cold air? Infection? Allergies? ALLEVIATING FctS; ASSOCIATED SYMPTOMS: CONSTITUTIONAL Sx; Fever/Chills/Wt loss/Lumps/Bumps RS: Cough/Sputum GI: Dec appetite (Liver & GI congestion) Increase in waist size (Ascitis) CVS: Wt gain/weakness/Fatigue (Decreased cardiac Output) Chest pain/Sweating/N/V/Heart racing/dizziness/Nocturia Leg Pain/Leg Swelling/Wt gain THYROID: Do you feel cold/Hot /tremors RISK FCTS: I need some more details about you to get a better understanding abt your condt & hence need to ask you a few more qns Do you smoke? Take alcohol? * Take your meds regularly? * Any change in your diet recently? --- Are you eating more canned foods or have you been taking salted nuts * Do you measure the Na in your diet? Did you notice you’re pale?

PMH: Were you ever diagnosed with HTN? What meds do you take? Have you taken your meds regularly? Were you ever diagnosed with a heart condition? Which HD?When were you diagnosed? How were you diagnosed? Do you have regular follow ups? Please note that this is only a draft version based on several sources, including: Dr. Basel Mohasen’s lectures, Toronto Notes, Therapeutic Choices and others. Edited and organized for the sake of all attendances of the Canadian Osce Exams: NAC OSCE and MCCQE2. by: Dr. Merlyn D’Souza and Dr. Zeev Gross, Spring 2011. Page 68

When was your last F up? During your last visit, did your Dr add or remove any medications? How were you doing? Were your symptoms controlled at that time? Particularly ask about Dixogin: How much/How long/Dose/did you have your levels checked/any chance that you might’ve missed a dose? Particularly ask about Water pills: NSAIDS DM/MI/Cholesterol/Stroke/Ca/Hospitalization/Surgery FH: FH of premature deaths

Back to Content

Please note that this is only a draft version based on several sources, including: Dr. Basel Mohasen’s lectures, Toronto Notes, Therapeutic Choices and others. Edited and organized for the sake of all attendances of the Canadian Osce Exams: NAC OSCE and MCCQE2. by: Dr. Merlyn D’Souza and Dr. Zeev Gross, Spring 2011. Page 69

SHORTNESS OF BREATH – POST SURGICAL Surgery 3 days ago: SOB x 45 mins D/D: 1. Volume status (low & High) 2. Atelectasis 3. Pneumonia 4. Heart Failure 5. Embolism 6. Fat embolism INTRO: As I understand, you’d surgery 3 days ago & I’ve to do a PE on you VITLAS please I would like to R/O orthostatic hypotension Respiratory Rate G/E: Orientation:  Time  Place  Person Head: Sclera & Pallor Mouth: S/o dehydration No Central cyanosis No nasal flaring or pursed lips No S/o Respiratory distress HANDS: Capillary refill Clubbing Cyanosis Skin: Hot/Cold Pulse: Rate & Volume LEGS: Dorsalis Pedis Temperature Pedal oedema Feel for DVT Measure diameter of (B) calves Homann’s sign NECK: JVP Trachea S/o respiratory distress & use of accessory muscles of respiration L Nodes for pneumonia Please note that this is only a draft version based on several sources, including: Dr. Basel Mohasen’s lectures, Toronto Notes, Therapeutic Choices and others. Edited and organized for the sake of all attendances of the Canadian Osce Exams: NAC OSCE and MCCQE2. by: Dr. Merlyn D’Souza and Dr. Zeev Gross, Spring 2011. Page 70

CHEST: Inspection: Symmetrical No IC retraction No accessory muscles No obvious pulsations No PMI seen FEEL Apex beat

Back to Content

Please note that this is only a draft version based on several sources, including: Dr. Basel Mohasen’s lectures, Toronto Notes, Therapeutic Choices and others. Edited and organized for the sake of all attendances of the Canadian Osce Exams: NAC OSCE and MCCQE2. by: Dr. Merlyn D’Souza and Dr. Zeev Gross, Spring 2011. Page 71

LUMP In Breast/Neck 40/F h/o lump in Breast x 8 wks INTRO: As I understand you’ve a lump in your breast since 8 weeks, can you tell me more about it since it all started? OCD: O: HOW did you notice it? Routine examination or Accidentally? C: From that time to now, is it increasing, decreasing or remaining the same? V: Is it the first time, or have you noticed it before? Any relation to periods? Did you notice it on the upper or lower half of breast or inner or outer side? Do you feel it reaching into the arm? SIZE: How do you estimate the size? Chickpea/Olive/egg/Orange? Hard/soft/rubbery? Pain+/Skin: slides or fixed? Changes in skin above: redness/ulcers Lumps in other breast Nipples: Dischareg/changes/ulcer? CSX: TRAUMA to breast? METASTASIS: Ha/Nx/Vx/Back pain/Cough/numbness in hand/Tired/pruritus? I’m going to ask you qns that may explain this: RISK FCTS: 1. H/o Ca in breast or other breast? 2. If any biopsy was performed on the breast? 3. FH of Ca breast 4. LMP 5. Have you ever been pregnant & at what age your first pregnancy? 6. Have you breast fed? 7. Any OCs & for how long? 8. Do you smoke/alcohol? 9. H/o Ovarian or uterine Ca 10. Fatty Diet?

Back to Content Please note that this is only a draft version based on several sources, including: Dr. Basel Mohasen’s lectures, Toronto Notes, Therapeutic Choices and others. Edited and organized for the sake of all attendances of the Canadian Osce Exams: NAC OSCE and MCCQE2. by: Dr. Merlyn D’Souza and Dr. Zeev Gross, Spring 2011. Page 72

LUMP IN NECK:16/F X 2 WKS D/d: Reactive Adenitis --------> Recent Flu Tooth problems ---------> Lymphoma ----------> Hard IM HIV INTRO: WHERE? HOW : OCD: SIZE: FEEL: Pain +/SKIN changes Any other lumps? CSx: Sore throat / dyspahgia? Hx/Nx/Vx? Skin rash? Ear discharge? Sinusitis/cough/Pglem? H/o Ca or malignancies? HEADSS Abd pain Vaginal discharge Urine changes PE: Vitals: Look & asses the lump Look for any other lumps: Cervical LN/Supraclavicular/axillary Mouth PA: Liver & spleen Groin LN Popliteal fossa Pelvic & Vaginal exam Rectal exam

Back to Content

Please note that this is only a draft version based on several sources, including: Dr. Basel Mohasen’s lectures, Toronto Notes, Therapeutic Choices and others. Edited and organized for the sake of all attendances of the Canadian Osce Exams: NAC OSCE and MCCQE2. by: Dr. Merlyn D’Souza and Dr. Zeev Gross, Spring 2011. Page 73

TIREDNESS 45M with tiredness x 6 weeks As I understand you’re having Tiredness since 6 weeks, can you tell me more about it since it all started? Pt says he is concerned. STOP & ask about his concern. He says he is Air traffic controller & his vision has been blurry Pt says, he is also never been so tired before. Clarify: What do you mean about Tiredness?  Sometimes I do not feel refreshed after sleep.  Do you feel lack of energy? Like you cannot move your arm above your head. The Statement: NOT REFRESHED ANYMORE points to an organic cause

OCD + Relation to sleep +/- Mood If Mood Sx + ------ MOAPS If Organic cause ---- Red Flags OCD At onset you can ask if there were any flu-like Sx initially (Thyroiditis)  Do you sleep more  When do you go to bed?  Do you wake up in the middle of the night  Which time of day/Night do you feel most?  Ask Nature of work; If shift  With whom do you sleep?  Does your partner C/o you snoring or jerky movts of limbs? (Restless Leg)  Do you feel better in morning or evening(if tired in morning-->Depression If evening-------->Organic cause) 1. Ask Constitutional Sx first: Fever/Wt loss/Night sweats/Chills/Lumps & Bumps Then quick review of Sx: 2. Cardiac: Chest Pain/SOB/Palpitations 3. Pulmonary: Cough/Wheezing/Phglem 4. GI: N/V Abnormal bowel movts/Diarrhoea/Malena/ 5. Liver: Dark urine/Yellow sclera/Abd pain/Loss of aooetite/Pale stool/Itchy skin 6. GU: Change in color of urine/Amt of urine/Cloudy/Frothy urine/Dysuria/Facial swelling 7. Anemia: Bleeding gums/Easy bruising/Malena/Haematuria/Female: Meorrhagia & LMP Please note that this is only a draft version based on several sources, including: Dr. Basel Mohasen’s lectures, Toronto Notes, Therapeutic Choices and others. Edited and organized for the sake of all attendances of the Canadian Osce Exams: NAC OSCE and MCCQE2. by: Dr. Merlyn D’Souza and Dr. Zeev Gross, Spring 2011. Page 74

8. Autoimmune Disease: Joint Pains/Skin rashes/Oral Ulcers 9. Endocrine: Thyroid: Feel hot/Cold Skin Moist/Dry/Tremors/Wt loss 10. DM: Risk fcts: FH & Lifestyle Once Pt has DM in history, GO over ALL Sx & Sy:

A. Fluctuating 6 Sx 3 High: 1. 2. 3. 3: 4. 5. 6.

Increased eating Inc Drinking Inc urination Tired Wt loss Blurry vision

B. Complications:  1. 2. 3. 4.

Micro:

Retinopathy: Black spots Neuropathy: Tingling/ numbness Nephropathy: Inc Urine Impotence: Sometimes people with this condition have marital problems; Do you have changes in desire or difficulty in having or maintaining an erection? 5. Autonomic Dysfn:  Orthostatic hypotension  Gastroparesis  Diarrhea  Voiding difficulties  HTN

 Macro: 1. Coronary Artery Disease: Chest Pain/SOB/ 2. CVA: Weakness/numbness/difficulty finding words/Sudden visual loss 3. Peripheral Vascular disease: Pain in calves/Cramps/Cold feet Skin infections Candidial infections in women RISK FCTS: 1. Diet 2. Exercise 3. FH Please note that this is only a draft version based on several sources, including: Dr. Basel Mohasen’s lectures, Toronto Notes, Therapeutic Choices and others. Edited and organized for the sake of all attendances of the Canadian Osce Exams: NAC OSCE and MCCQE2. by: Dr. Merlyn D’Souza and Dr. Zeev Gross, Spring 2011. Page 75

4. Smoking 5. Alcohol 6. Recreational drugs PAST MEDICAL HISTORY: Any medications/OTC/Herbal/LMP Surgery/Hospitalization FAMILY HISTORY: DM/HTN/Stroke/MI SOCIAL H: Habits With whom do you live? How do you support yourself financially?

Please note that this is only a draft version based on several sources, including: Dr. Basel Mohasen’s lectures, Toronto Notes, Therapeutic Choices and others. Edited and organized for the sake of all attendances of the Canadian Osce Exams: NAC OSCE and MCCQE2. by: Dr. Merlyn D’Souza and Dr. Zeev Gross, Spring 2011. Page 76

Here Counselling should be short Focus on Blurry vision in relation to DM & why he needs to correct it. As I told you DM is a lifelong disease & it is imp to have the BSL controlled. Without proper BSL control, the increasing Blood sugar damages the blood vessels in our body, & leads to heart attacks, Strokes, Kidney failure. Also visual loss & feet ulcers I will have to refer you to a diabetic clinic However I will do basic blood inv & ECG first

Back to Content Please note that this is only a draft version based on several sources, including: Dr. Basel Mohasen’s lectures, Toronto Notes, Therapeutic Choices and others. Edited and organized for the sake of all attendances of the Canadian Osce Exams: NAC OSCE and MCCQE2. by: Dr. Merlyn D’Souza and Dr. Zeev Gross, Spring 2011. Page 77

35 M in hospital setting had a DKA 3 days ago x 5 mins H He is diabetic since 25 years Here we’ve to look for Rf & see that are not repeated As I understand you were seen in the hospital by my colleagues & Rxed for a condt called DKA 3 days ago. Can you tell me what exactly happened to you at the time? Xxxxx 1. Was there abdominal pain 2. Vomiting? 3. Were you drowsy? 4. Was there loss of consciousness? 5. How did you get to the ER? 6. 7. 8. 9.

When were you Diagnosed ad Diabetes? How were you Dsed? What type of Dm? Which medications do you take/

10. Insulin? 10.1 How much? 10.2 When was the last time your insulin was adjusted? 10.2 What was the wt at that time/ 11. Any loss or gain of wt? 12. How often do you monitor your bl Glucose? 13. When was the last HbA1c 14. Did you have DKA before? 15. When was the last time you had it/Are you under regular F/U? 16. How were you doing for the last few mths? Ask about Sx Blurry vision Inc thirst Inc Peeing Wt loss

control:

Any idea why Sx are not controlled? DIET What about diet? How often do you party? Do you modify your diet when you party? Do you adjust insulin when you party/ If you take alcohol do you count it as a part of your calories Please note that this is only a draft version based on several sources, including: Dr. Basel Mohasen’s lectures, Toronto Notes, Therapeutic Choices and others. Edited and organized for the sake of all attendances of the Canadian Osce Exams: NAC OSCE and MCCQE2. by: Dr. Merlyn D’Souza and Dr. Zeev Gross, Spring 2011. Page 78

EXERCISE: Any RECENT INFECTION/FEVER? LONG TERM COMPLICATIONS: Have you ever had an eye check/ When was the last time you saw an ophthalmologist? Any retinal complications/ Any H/o heart attack? How are your feet/ Any feet ulcers? How is your urine,is it frothy/Cloudy/ Do you have tingling/numbness Thank you for the information,we will have to do further assessment

38M requesting a note to say he is well & fit to work Cannot see at work, works as an accountant.

Back to Content

Please note that this is only a draft version based on several sources, including: Dr. Basel Mohasen’s lectures, Toronto Notes, Therapeutic Choices and others. Edited and organized for the sake of all attendances of the Canadian Osce Exams: NAC OSCE and MCCQE2. by: Dr. Merlyn D’Souza and Dr. Zeev Gross, Spring 2011. Page 79

ABDOMINAL PAIN/ACUTE ABDOMEN 1. 2. 3. a. b. 4. a. b. c.

Acute abdomen x 24 hrs 45/M Er x 5min H Dysphagia x 6 wks 55M Inc LFT: 35 M ALT>>AST 55M AST > ALT Diarrhoea: Ch Diarrhoea x 6 wks H & PE Ch Diarrhoea x 6 wks H & C Ac Diarrhoea x 3 days H x 5mins

AC ABD: Acute abdomen x 24 hrs 45/M Er x 5min H As I understand you’ve abdominal pain since 24 hrs. Are you comfortable sitting down? Please bear with me for 5 mins till I find the cause of this pain & then I can give you something for relief OCD: O: Gradual/Sudden C: From the onset till now,is it: Inc/Dec Off & On/Ct-ous How was it at first? When change in course,QUALITY & SEVERITY is significant P Q: Ask nature from beginning R S T: Any relation with time? Before eating/After eating U: How was your sleep? V: Alleviating: Eating/Antacids/Defecation/Meds? Aggravating Fcts: Food/Posn/Alcohol/Meds?

Assoc Sx:   * * * *

Nausea

Vomiting: If +ve: Color Odour Consistency Amount

Please note that this is only a draft version based on several sources, including: Dr. Basel Mohasen’s lectures, Toronto Notes, Therapeutic Choices and others. Edited and organized for the sake of all attendances of the Canadian Osce Exams: NAC OSCE and MCCQE2. by: Dr. Merlyn D’Souza and Dr. Zeev Gross, Spring 2011. Page 80

* Forceful * Does vomiting relieve pain? * What started first: Vomiting or pain? (If V 1st --Infection/ If Pain 1st Surgical) BOWEL CHANGES: * When was the last bowel movt? * Any abdominal distension? * Is there any blood in stool/Dark stool? * Are you passing any gases? APPETITE CHANGES: Sx Dehydration: * Dizziness * Dark Urine LAST MEAL: * Did you eat alone? * Was there anything new? * Did others have same Sx? RISK FCTS: * Any Past H/o Abdominal Surgeries * Crohn’s disease? * Groin surgery(hernia repairs) * Gall Bladder stones? * Pancreatic stones * Diverticulitis? * Smoking * Alcohol CONSTITUNIOL Sx: Fever/Chills/wt loss/Lumps 7 bumps CVS: Chest Pain/Hd/HTN FH; Ca Colon/Polyposis PAST Medical h: Meds: Aspirin/OTC/Herbals/LMP

Back to Content

Please note that this is only a draft version based on several sources, including: Dr. Basel Mohasen’s lectures, Toronto Notes, Therapeutic Choices and others. Edited and organized for the sake of all attendances of the Canadian Osce Exams: NAC OSCE and MCCQE2. by: Dr. Merlyn D’Souza and Dr. Zeev Gross, Spring 2011. Page 81

71F/Abd pain after meals x 4 weeks [10 min H] (ischemic) Dd: 1. 2. 3. 4. 5. 6.

GERD PUD Gastric Ca Ischemic Mesentry Pancreatic failure Ischemic colitis Here Pt was concerned that spouse died of Ca Stomach.Reassuare her that unlike Flu Ca Stomach is not contagious, but because they were married for so long there might be a chance that they were exposed to a risk fct.But you will conduct a thorough History & Inv

OCD PQRSTUV A&A Assoc Sx:  Constitutional Sx:  GI Sx: * * * *

N/V/D Malena Bowel movts Abdominal bloating

 Cardiac Sx: * Chest Pain/Palpitations/SOB RISK FCTS: HTN/DM/Smoking/FH of HD/Cholesterol?HD If +ve for HD  

Ascertain type & when diagnosed,enquire abt F ups When/what & if intervention

PMH:  

Any meds/OTC/Herbal Hospitalization & Surgery

FH Social H: Here since Pt had Cardiac Sx.Your cardiac history is of concern to me & I’ve to do a PE & perform some blood inv & an ECG & then ask for a Surgical Consult

Back to Content Please note that this is only a draft version based on several sources, including: Dr. Basel Mohasen’s lectures, Toronto Notes, Therapeutic Choices and others. Edited and organized for the sake of all attendances of the Canadian Osce Exams: NAC OSCE and MCCQE2. by: Dr. Merlyn D’Souza and Dr. Zeev Gross, Spring 2011. Page 82

DIARRHOEA Bloody Diarrhoea: Awakens at night? Any constant pain? Think Inflammatory bowel disease Always ask about antibiotic use: if yes: 1. 2. 3. 4.

Why Which Ab For how long When

In acute Diarrhoea look for dehydration In ch diarrhoea look for Wt loss & then dehydration Bad Diarrhoea if: +ve for fat droplets Floats Undigested food Bulky After intro:

OCD Frequency COCA + Blood Impact PAIN ASx D/d: 1. GE: if fever/N/V 2. Travel 3. New restaurant 4. Antibiotic 5. Osmotic PMH Fh Sh Please note that this is only a draft version based on several sources, including: Dr. Basel Mohasen’s lectures, Toronto Notes, Therapeutic Choices and others. Edited and organized for the sake of all attendances of the Canadian Osce Exams: NAC OSCE and MCCQE2. by: Dr. Merlyn D’Souza and Dr. Zeev Gross, Spring 2011. Page 83

24 yr old diarrhoea x 3 days Introduction: Tell me more about is since the moment it all started? O C: How often do you go to the wash room? Estimated amount of stool passed? Is it Tarry? Mixed with blood? Any undigested food? Bulky? Is it offensive? Does it float in toilet bowl? Is it difficult to flush? A&A Did you try any meds/did they help ASx: Do you feel dizzy/Thirsty? Do you drink enough fluid? Have you lost wt? Do you have any additional Sx like abdominal pain? Does bowel movt relieve your pain? N/V CSx: Did you eat in a new place? (raw food----Shigella) Did you recently take any antibiotics? Anybody else in the family has diarrhoea? Any FH of Bowel disease? Or condt called Crohn’s Disease? FH SH

Please note that this is only a draft version based on several sources, including: Dr. Basel Mohasen’s lectures, Toronto Notes, Therapeutic Choices and others. Edited and organized for the sake of all attendances of the Canadian Osce Exams: NAC OSCE and MCCQE2. by: Dr. Merlyn D’Souza and Dr. Zeev Gross, Spring 2011. Page 84

24 yr M Diarrhoea x 6 weeks (Crohn’s) Can you tell me about it? O C: All the day?/At night/ D COCA + Bl Bulky + Fat droplets Floats & difficult to flush Undigested food PAIN Relieved by diarrhoea or not Distension/gases U: How has it affected your life How do you deal with it? V: CSx: Extraintestinal Sx: Skin rashes/nail changes/Joint pains/back pain/Hx of psoriasis/Back pain/red eyes/Morning urethral discharge/ D/D: Thyroid :tremors/sweating Relation of diarrhoea to food like dairy products Travel or camping history Sx of Liver disease FH of Ulcerative colitis/Crohn’s SH

Back to Content

Please note that this is only a draft version based on several sources, including: Dr. Basel Mohasen’s lectures, Toronto Notes, Therapeutic Choices and others. Edited and organized for the sake of all attendances of the Canadian Osce Exams: NAC OSCE and MCCQE2. by: Dr. Merlyn D’Souza and Dr. Zeev Gross, Spring 2011. Page 85

DIARRHOEA x 6 wks (Irritable bowel disease) Dd: 1. IBD 2. Infection: camping/travel 3. Hyperthyroidism 4. IBS 5. Lactose intolerance 6. Coeliac Disease 7. HIV 8. Ch Liver & pancreratic failure OCD COCA + Blood + mucus PAIN:if +ve which started first Pain or diarrhoea Does bowel movement relieve the pain? Does it awaken you at night? Do you feel you want to go back to wash room again? Distension/gases U What made you come in today? V: DIET: dairy products C Sx: In IBS R/o Organic cause: Stress? What type; Family/Job How do you cope with it? How is your mood/Interest? FH: of Ca Colon at young age SH

Back to Content

Please note that this is only a draft version based on several sources, including: Dr. Basel Mohasen’s lectures, Toronto Notes, Therapeutic Choices and others. Edited and organized for the sake of all attendances of the Canadian Osce Exams: NAC OSCE and MCCQE2. by: Dr. Merlyn D’Souza and Dr. Zeev Gross, Spring 2011. Page 86

42 M with Bld work LFT INCREASED: ALT:300 AST:100 Cc: Bld works Intro Talk about blood works: 1. WHY? 2. Is it 1st time 3. When 4. Who 5. If done before: what were results then? SPIKE EXPLAIN Results CONSEQUENCES: Sx. Ac & Ch CAUSES OF LIVER DISEASE: PMH FH SH Intro: As I understand you’re here today to discuss some of your blood tests results Is it 1st time/ Why/who/When EXPLAIN: Results show that there is an increase in the markers to measure the function of the liver, called liver enzymes, this indicates that there is an injury to the liver cells There are different causes, but before coming to the causes, I want to know if there are any Symptoms of liver disease Ac Sx; 1. Yellow discoloration of skin/eyes 2. Pale stools/Dark urine 3. Itchy skin 4. Loss of appetite/nausea/distaste for cigarettes 5. Flu like Sx few weeks ago Ch Sx; CAUSES OF LIVER DISEASE 1. Have you ever been diagnosed as liver disease before? 2. Have you ever ben screened for liver disease before? 3. Have you ever been vaccinated for Hepatitis A or Hepatitis B before? TRANSITION Please note that this is only a draft version based on several sources, including: Dr. Basel Mohasen’s lectures, Toronto Notes, Therapeutic Choices and others. Edited and organized for the sake of all attendances of the Canadian Osce Exams: NAC OSCE and MCCQE2. by: Dr. Merlyn D’Souza and Dr. Zeev Gross, Spring 2011. Page 87

I want to ask qns now as to whether you were exposed to liver disease: 1. Do you smoke? Take alcohol/Recreational drugs? Past use IV drugs 2. Tattoos 3. Any past surgeries/hospitalizations 4. FH of liver disease 5. Long term disease in past 6. Any H/o bleeding disease Thank you for all this info Need to do some further assessment

Back to Content

Please note that this is only a draft version based on several sources, including: Dr. Basel Mohasen’s lectures, Toronto Notes, Therapeutic Choices and others. Edited and organized for the sake of all attendances of the Canadian Osce Exams: NAC OSCE and MCCQE2. by: Dr. Merlyn D’Souza and Dr. Zeev Gross, Spring 2011. Page 88

52 M/Dysphagia x 6 wks ----5 mins H D/d: 1. 2. 3. 4. 5. 6. 7.

Oesophageal Ca Scleroderma O.Stricture O. DES web/Ring HIV

FIRST CLARIFY: If difficulty to initiate swallowing or food coming out from nose ---- Neuro Sx/Stroke/MS Pain on swallowing ------ AIDS/CMV/Ca/Decreased immunity/Leukemia If food gets stuck -> Can you show where it gets stuck? ONSET:How did it start? COURSE: Intermittent /progressive Intermittent for both solids & liqds ----> SPASM Intermittent for only SOLIDS---------------> Oesophageal web/ring check by endoscopy Progressive: Starts with fluid & Solids later ------>Achalasia & Scleroderma Progressive: Starts with solids & later liquids ------> M/Canical obstruction: Ca Here initially you could flush it down with water, but unable to do so now ASSOC Sx: N/Vx Repeated chest infections Chest pain/Tightness in chest Do you bring up undigested food? Cough Change in voice GI Sx: Abd pain/Abd distension Change in bowel pattern Any blood in stool or vomit? CONST Sx: Fever/ Night sweats/Change in appetite/Chills /Lumps & bumps If wt loss,how much Wt loss over how long? Please note that this is only a draft version based on several sources, including: Dr. Basel Mohasen’s lectures, Toronto Notes, Therapeutic Choices and others. Edited and organized for the sake of all attendances of the Canadian Osce Exams: NAC OSCE and MCCQE2. by: Dr. Merlyn D’Souza and Dr. Zeev Gross, Spring 2011. Page 89

Tiredness *Mets: Liver: Yellow discoloration of skin/Dark urine & pale stools RISK FCTS: I’m going to ask you a few qns to reach the diagnosis: 1. Any H/o heart burn? 2. Have you ever been diagnosed with a condt called GERD?----If Yes: How long ago? Did you seek medical attention? Was an endoscopy performed (A tube with a camera put down your food pipe to view) 3. Were you ever diagnosed with a condt called: Barret’s Oesophagus? 4. Do you smoke?/Drink alcohol? 5. FH of Oesophageal Ca 6. H/o swallowing acid or alkalies 7. H/o Chest radiation 8. H/o Achalasia 9. Any skin tightness 10. Change in color of digits when exposed to hot or cold enviorenments 11. CNS: H/o stroke/weakness H/o DM H/o HIV

Back to Content

Please note that this is only a draft version based on several sources, including: Dr. Basel Mohasen’s lectures, Toronto Notes, Therapeutic Choices and others. Edited and organized for the sake of all attendances of the Canadian Osce Exams: NAC OSCE and MCCQE2. by: Dr. Merlyn D’Souza and Dr. Zeev Gross, Spring 2011. Page 90

Chest Pain 45 M x 6 wks

GERD ONSET: When did you first notice it? How did it start? What were you doing at that time? Course: 1. Is it the same intensity it was at the beginning? 2. Is it increasing in intensity? 3. Is it decreasing in intensity 4. Is the frequency increasing or decreasing or same? Duration: Foe how long now you’ve the pain? POSITION: Can you tell me exactly where it hurts you? Quality Can you describe the nature of your pain? Burning,Tightnes.... Radiation: Does it move anywhere else in your body? To the (L) arm,Jaw,Neck,Back? Severity: On a scale of 1 –10 ...... How has this pain affected your life? Time Does it occur at a particular time? Does it awaken you at night? Aggravating fcts: 1. Exercise/Stress 2. Food: (peppermint,fatty food,Citrus fruit) 3. Tobacco 4. Alcohol 5. Hot or cold food Relieving fcts: 1. Antacid 2. Elevation of head end of bed Assoc Fcts: Do you have 1. Heart burn 2. Acid reflux 3. Difficulty swallowing 4. Dark stools? 5. Cough Please note that this is only a draft version based on several sources, including: Dr. Basel Mohasen’s lectures, Toronto Notes, Therapeutic Choices and others. Edited and organized for the sake of all attendances of the Canadian Osce Exams: NAC OSCE and MCCQE2. by: Dr. Merlyn D’Souza and Dr. Zeev Gross, Spring 2011. Page 91

6. 7. 8. 9.

Hoarse voice Sore throath Wheezing Dental problems (dental erosions)

10. Palpitations 11. SOB/PND/Orthopnea 12. Constitutional Symptoms: 13. Loss of appetite & Wt loss 14. Fever & Chills 15. Lumps & bumps 16. Lupus To gain more insight into your condition, I need to ask you some details about your life: Do you smoke Drink Alcohol *** ? Recreational drugs? Do you eat a lot of fatty foods? How soon afer dinner do you go to bed? PAST Med H 1. Are you on any medication/ 2. Do you take Aspirin or any pain relievers or any OTC products? 3. Is there any herbal medication you’re on? 4. Are you allergic to anything? 5. Have you ever been diagnosed to have a High BP or high BSL? 6. Have you ever had a heart attack or stroke? 7. Were you ever hospitalized? 8. Did you ever undergo any surgical procedure? FAMILY History Social History MGment:

Nonpharmacologic Choices        

Dietary modifications (avoid chocolate, caffeine, acidic citrus juices, large fatty meals) Weight loss if obese (BMI > 25 to 30 kg/m2) No snacks within 3 hours before bedtime No lying down after meals Reduce alcohol intake Elevate legs under the head of the bed on 10 to 15 cm blocks Stop smoking Avoid tight clothing

Please note that this is only a draft version based on several sources, including: Dr. Basel Mohasen’s lectures, Toronto Notes, Therapeutic Choices and others. Edited and organized for the sake of all attendances of the Canadian Osce Exams: NAC OSCE and MCCQE2. by: Dr. Merlyn D’Souza and Dr. Zeev Gross, Spring 2011. Page 92

Pharmacologic Choices When possible, eliminate drugs that impair esophageal motility and lower esophageal sphincter tone (e.g., calcium channel blockers, theophylline, tricyclic antidepressants, beta-blockers, anticholinergic agents).

Back to Content

Please note that this is only a draft version based on several sources, including: Dr. Basel Mohasen’s lectures, Toronto Notes, Therapeutic Choices and others. Edited and organized for the sake of all attendances of the Canadian Osce Exams: NAC OSCE and MCCQE2. by: Dr. Merlyn D’Souza and Dr. Zeev Gross, Spring 2011. Page 93

6th Feb

GU If CC Urinary Sx: I. II. III.

Obstruction Irritation Urinary changes

OSTRUCTION 4 qns: 1. Difficulty initiating urine ----->Do U need to strain? 2. Did you notice change in stream? 3. Dribbling? 4. After passing urine, do you still need to pass more?

IRRITATION: 1. 2. 3. 4. 5. 6. 7. 8.

How many time do you need to go to Wash room: Now,Before At Night? Does it affect your sleep? Do you feel you need to rush to WC Are you able to make it in time? Have you ever lost control? Burning sensation Flank pain Fever

URINE CHANGES: COCA + Bld Consistency,remember: 1. Frothy urine 2. Cloudy urine 3. Not clear urine

Back to Content

Please note that this is only a draft version based on several sources, including: Dr. Basel Mohasen’s lectures, Toronto Notes, Therapeutic Choices and others. Edited and organized for the sake of all attendances of the Canadian Osce Exams: NAC OSCE and MCCQE2. by: Dr. Merlyn D’Souza and Dr. Zeev Gross, Spring 2011. Page 94

67 M ER reten48 hrs/Colleague passed F Catheter & got 1.2 L urine As I understand, you’re here today because you’d difficulty in passing urine since 24 hrs.& one of my colleagues has passed a Foley catheter & drained 1.2L urine. HOW DO YOU FEEL RIGHT NOW? I’m glad to know you’re better

If Pt c/o pain: Bear with me few minutes as soon as I finish with asking you a few Qns I will deal with it. Can you tell me about it since it all began? OCD -------- U V How many times did you try to void? Were you able to pass any amt? Is it the first time/Can be first time to this extent Recently have you noticed any changes in your urine?you.g: Do U need to rush? When did it first start? From that time till now, is it Increasing/Decreasing?  Sx of Obstruction: 1. Difficulty initiating urine ----->Do U need to strain? 2. Did U notice change in stream? 3. Dribbling? 4. After passing urine,do U still need to pass more? EMPATHY  Sx of irritation: 1. How many times do U need to go to Wash room: Now, Before At Night? Does it affect your sleep? 2. Do U feel U need to rush to WC Are you able to make it in time? Have you ever lost control? 3. Burning sensation 4. Flank pain 5. Fever  Urine changes: COCA + Bld Ask if H/o passing stones in urine C Sx: ASx: Trauma to back:Back pain Sx of GU: asked in earlier Qns Mets to Liver: Sx of Liver Disease Mets to lung: Cough/Haemoptysis Mets to CNS: RISK FCTS: 1. Have you ever been screened or diagnosed as prostrate disease? Please note that this is only a draft version based on several sources, including: Dr. Basel Mohasen’s lectures, Toronto Notes, Therapeutic Choices and others. Edited and organized for the sake of all attendances of the Canadian Osce Exams: NAC OSCE and MCCQE2. by: Dr. Merlyn D’Souza and Dr. Zeev Gross, Spring 2011. Page 95

2. 3. 4. 5. 6. D/D: 1. 2. 3. 4. 5. 6.

Have you ever done the blood test for PSA FH of Ca Prostrate: Who & at what age? Do you smoke Take alcohol How is your diet?

Are you on any medications? Do you take psychiatric medications Meds for Glaucoma? H/O Stroke H/o Urethritis Sx of renal failure: Puffy face, swollen ankles PMH: Since this is the first time I’m seeing you, I need to ask you some qns regarding your Past Medical History: Do you have DM/HTN Hospitalized or had any surgeries? SOCIAL H:

Back to Content

Please note that this is only a draft version based on several sources, including: Dr. Basel Mohasen’s lectures, Toronto Notes, Therapeutic Choices and others. Edited and organized for the sake of all attendances of the Canadian Osce Exams: NAC OSCE and MCCQE2. by: Dr. Merlyn D’Souza and Dr. Zeev Gross, Spring 2011. Page 96

67 M brought in by daughter,as she is concerened that he is not himself Greeting: As I understand .......... Whenever a pt is brought in by someone always ask:DO U AGREE? If Yes:...I’m glad you’re here as we can find a working solution If NO: I would appreciate that you’re here just to make you daughter happy, I promise you I will be as fast as I can. Tell me more about the concern: She is worried that I’m not going out as I used to before. Do you stay at home? I’m not going away to far places or out with my friends When did this happen? Do you share her concern? My daughter overreacts I’m Glad you’re here What prevents you from going out? GIVE CONFIDENTIALITY I wet myself When How many times? How did you react at that time? The go to Urine History: Obstructn/Iritation/Urine If Pt does not divulge, Go to Geriatric History: 1. What Medications?do you take & ask to see list or bag of meds Do you take sleeping pills (specifically ask for that as it affects memory & gait)OTC/Herbal products 2. Screen for Mood & Memory 3. Vision & Hearing/Balance & Falls 4. Urine: Retention in males & Incontinence in females 5. Sleep: How many hours 5.1 When do you go to bed? 5.2 Do you get up frequently? Follow event.......... If still refuses to talk,do review of Sx

Back to Content

Please note that this is only a draft version based on several sources, including: Dr. Basel Mohasen’s lectures, Toronto Notes, Therapeutic Choices and others. Edited and organized for the sake of all attendances of the Canadian Osce Exams: NAC OSCE and MCCQE2. by: Dr. Merlyn D’Souza and Dr. Zeev Gross, Spring 2011. Page 97

67 M Dark urine x 1 week (H x 5 min) D/D: 1. 2. 3. 4. 5. 6. 7. 8.

Bleeding/SE of warfarin PSGN Stone Nephrotic syndrome Renal or bladder Ca Trauma Berger’s disease Infection

If CC is Dark Urine, make sure it is haematuria & not Jaundice What do you actually mean? Dark like Tea/Cola or Red? OCD O:Sudden/gradual C: from that time till now,is it: Off & on or continous/Same D: Has it ever happened before? P Does it occur at the: 1. Beginning of stream?(Urethra) 2. End of the stream? (bladder) 3. Whole of the stream? (Kidney) V: Urine changes:COCA + Bld Consistency,remember: 1. Frothy urine 2. Cloudy urine 3. Not clear urine Obstrn 1. Difficulty initiating urine ----->Do you need to strain? 2. Did you notice change in stream? 3. Dribbling? 4. After passing urine, do you still need to pass more? Irritation: 1. How many time do you need to go to Wash room: Now,Before At Night? 2. Does it affect your sleep? 3. Do you feel you need to rush to WC 4. Are you able to make it in time? 5. Have you ever lost control? 6. Burning sensation 7. Flank pain 8. Fever Please note that this is only a draft version based on several sources, including: Dr. Basel Mohasen’s lectures, Toronto Notes, Therapeutic Choices and others. Edited and organized for the sake of all attendances of the Canadian Osce Exams: NAC OSCE and MCCQE2. by: Dr. Merlyn D’Souza and Dr. Zeev Gross, Spring 2011. Page 98

C Sx: ASx: 1. Back trauma 2. H/O recent Sore throat or skin infection If yes to sore throat: When was that/Was there swelling of feet/Puffy face? 3. H/o bleeding tendencies or blood thinners If Yes to blood thinners: Which one/Why/How long/How much do you take/When was last F/U/what was your last INR/What is the target/Any new medications/Any antibiotics? 4. Did you notice bleeding from any other sites? Gums/Nose/Malena/CNS:Numbness,weakness,difficulty finding words ...... 5. H/o stones

Back to Content

Please note that this is only a draft version based on several sources, including: Dr. Basel Mohasen’s lectures, Toronto Notes, Therapeutic Choices and others. Edited and organized for the sake of all attendances of the Canadian Osce Exams: NAC OSCE and MCCQE2. by: Dr. Merlyn D’Souza and Dr. Zeev Gross, Spring 2011. Page 99

Neuropathic Pain Neurologic Disorders: Neuropathic Pain Table 1: Types of Neuropathic Pain Peripheral Neuropathic Pain

Central Neuropathic Pain



Nerve root pain



Central post-stroke pain



Carpal tunnel syndrome



Spinal cord injury pain



Trigeminal neuralgia



Brain injury



Postherpetic neuralgia



Multiple sclerosis



Incisional neuralgia



Syringomyelia



Nerve trauma (causalgia)



Phantom limb pain

Investigations History with attention to: o temporal profile and characteristics of the pain o functional status, mood, quality of life, insomnia, sexual function, previous and current treatments, especially concurrent medications o present or past chemical dependency, especially if opioids are considered  Physical examination: o determine areas of sensory loss (hypoesthesia) and skin sensitivity characteristic of neuropathic pain determine other neurologic findings that might indicate a progressive lesion requiring imaging and surgery o determine concurrent conditions that contribute to the pain problem, e.g., concomitant muscular pain and psychological factors  Other investigations: o imaging with CT or MR scanning if a space-occupying lesion is suspected o electromyography o diagnostic sympathetic blockade if complex regional pain syndrome is suspected o although there is no established therapeutic range, monitoring serum levels of tricyclic antidepressants (TCAs) and antiepileptic drugs may help to assess adherence and guide dosage

Guideline for Use of Opioids in Chronic Nonmalignant Pain 

Consider after other reasonable therapies have failed.

Please note that this is only a draft version based on several sources, including: Dr. Basel Mohasen’s lectures, Toronto Notes, Therapeutic Choices and others. Edited and organized for the sake of all attendances of the Canadian Osce Exams: NAC OSCE and MCCQE2. by: Dr. Merlyn D’Souza and Dr. Zeev Gross, Spring 2011. Page 100











Perform a complete pain and psychosocial history, physical examination and appropriate diagnostic tests. A history of substance abuse, tension-type headaches, frequent migraine headache, muscular pain (myofascial pain, fibromyalgia) or pain that appears to be largely determined by psychologic factors is a relative contraindication to the use of opioid therapy. A single physician/prescriber/pharmacy is optimal. The prescriber may choose to set up a contract with the patient. The agreement should specify the drug regimen, possible side effects, the functional restoration program and that violations may result in termination of opioid therapy. The opioid analgesic of choice should be administered around the clock and may include a provision of “rescue doses” for breakthrough pain. Controlled-release preparations include morphine, oxycodone, hydromorphone, tramadol and transdermal fentanyl. Avoid meperidine primarily because of accumulation of its excitotoxic metabolite normeperidine. Codeine is a poor analgesic for moderate to severe pain because it has to be metabolized to morphine. Drug administration should include a titration phase to minimize side effects. If a graded analgesic response to incremental doses is not observed, the patient may not be opioidresponsive, and opioid treatment should probably be terminated. The patient should be seen monthly or more often for the first few months and every 2–3 months thereafter. At each visit 1. assess pain relief (0–10 scale), 2. mood, 3. side effects, 4. quality of life, 5. adherence to functional goals and 6. presence of drug-related behaviour. Optimally, affix a copy of the prescription and drug therapy flow sheet to the medical record. The goal of opioid therapy is to make the pain tolerable. For some patients with chronic noncancer pain (e.g., postherpetic neuralgia), the administration of an opioid analgesic can mean the difference between bearable and unbearable pain.

Therapeutic Tips  

   

Two to three months constitutes a reasonable trial of medication for neuropathic pain. While patients frequently say they have used amitriptyline or carbamazepine or other agents, these drugs have often been used in too high or too low a dose and for too short a period of time. It is useful to re-institute these drugs to evaluate their effectiveness when used appropriately: start low, go slow, increase dose until relief of symptoms or side effects occur and treat side effects when possible. Be sure the patient understands the goals of therapy: reduction in pain from moderate or severe to mild, at the price of some side effects that may be tolerable or treatable. Use a pain assessment tool, such as a scale of 0–10 where 0 is no pain and 10 the worst pain imaginable, to evaluate pain with and without activity, and before and after medication. As a matter of course, prescribe an artificial saliva mouth spray with TCAs and a stool softener with TCAs or opioids. Use controlled-release formulations of carbamazepine and opioids

Please note that this is only a draft version based on several sources, including: Dr. Basel Mohasen’s lectures, Toronto Notes, Therapeutic Choices and others. Edited and organized for the sake of all attendances of the Canadian Osce Exams: NAC OSCE and MCCQE2. by: Dr. Merlyn D’Souza and Dr. Zeev Gross, Spring 2011. Page 101



 

   

It may be possible to reduce or gradually withdraw medication after initial control of pain and a period of relief of 1–3 months (pain such as postherpetic neuralgia may resolve spontaneously and trigeminal neuralgia may go into remission). Gradual reduction is important to avoid withdrawal symptoms. Always consider combining pharmacotherapy with appropriate psychological and physical measures. Try different drugs within a class (e.g., a TCA or a gabapentinoid such as gabapentin or pregabalin), drugs of different classes and combination therapy (polypharmacy) for a possible additive or synergistic effect; do not combine TCAs with SNRIs. If opioids are used, guidelines are important and should be worked through with the patient. A trial and error approach of scientifically unproven treatments is reasonable if standard therapy fails. Repeated visits can provide important psychological support and hope for desperate patients as trial and error approaches are utilized. If chronic neuropathic pain is being managed in general practice, semi-annual or annual visits to a pain specialist (where available) help provide support to the family practitioner for contentious approaches such as opioids, and offer the chance of a novel therapy for the patient

Back to Content

Please note that this is only a draft version based on several sources, including: Dr. Basel Mohasen’s lectures, Toronto Notes, Therapeutic Choices and others. Edited and organized for the sake of all attendances of the Canadian Osce Exams: NAC OSCE and MCCQE2. by: Dr. Merlyn D’Souza and Dr. Zeev Gross, Spring 2011. Page 102

HEADACHE: Dd: 1. Tension H 2. Cluster H 3. Migraine 4. Temporal A 5. Cervical Spondylitis 6. Meningitis 7. SOL 8. SAH 9. Depression 10. Spousal abuse

Red Flags for Serious Headache 1. Age of onset 2. Type of onset 3. Temporal sequence

Middle-aged to elderly patient Severe and abrupt Progressive severity or increased frequency 4. Pattern Significant change in headache pattern 5. Neurologic signs Stiff neck, focal signs, reduced consciousness 6. Systemic signs Fever, appears sick, abnormal examination Caution: If headache does not fit typical pattern, a serious diagnosis can be missed.

***** Chronic Daily Headache & Medication overuse Headache Chronic headache occurs daily or almost daily for 15 days per month, for 6 months or longer. The most common causes of these headaches are transformed migraine and chronic tension-type headache. In the former there is history of migraine attacks and over several years the migraine attacks become more frequent. Soon the migraine characteristics give way to chronic daily headache with a daily or near-daily background headache that often resembles a typical “tension-type headache.” People with chronic tension-type headache may have no history of distinct migraine.

Patients with these disorders frequently use excessive amounts of abortive agents, including ergots, acetaminophen, ASA and opioid analgesics. They can have rebound headaches as a result of medication-overuse, while some may have symptoms of depression or other psychological comorbidities. Rebound headaches can also occur with the overuse of triptans. Most will improve in days or a few weeks with the discontinuation of these medications, especially mixed analgesics. Please note that this is only a draft version based on several sources, including: Dr. Basel Mohasen’s lectures, Toronto Notes, Therapeutic Choices and others. Edited and organized for the sake of all attendances of the Canadian Osce Exams: NAC OSCE and MCCQE2. by: Dr. Merlyn D’Souza and Dr. Zeev Gross, Spring 2011. Page 103

Generally, simple analgesics should be used less than 15 days per month in primary headache disorders such as migraine or tension-type headache or they will lead to the development of medication-overuse headache and chronic daily headache. Further, if chronic daily headache develops, other useful abortive and prophylactic medications usually have less efficacy.

Management includes recognition of these disorders, tapering and stopping the offending agent(s), and starting a prophylactic medication such as amitriptyline or another agent listed in. During withdrawal, particularly in patients with transformed migraine, use abortive agents such as DHE or a triptan for treatment of the migraine headaches that emerge. Short-term admission to hospital may be required to use the Raskin protocol (using DHE) and give support. If psychological comorbidities such as depression are present, they must be managed and treated. Consider referral to a multidisciplinary pain management clinic for cases failing to respond to therapy.

Therapeutic Tips     

   

Give abortive treatment, without exceeding recommended dosages, as soon as possible. Use simple analgesics less than 15 days per month, and ergots, triptans, opioids or analgesic combinations less than 10 days per month. A calendar or diary of headaches is useful in follow-up assessment. Keep a record of medications (usefulness, dosage and side effects). If migraine that does not respond adequately to symptomatic therapy occurs more than 3–4 times per month, try prophylactic medications for several months and then discontinue if possible, to assess ongoing need.2 Different medications may need to be tried, including different members of the same class, such as triptans. Follow-up is most important in managing chronic headache. Reassurance and explanation are most important to the patient in the long term. Always offer hope to patients with chronic headache even if no cure is available; most primary headaches can be controlled.

Back to Content

Please note that this is only a draft version based on several sources, including: Dr. Basel Mohasen’s lectures, Toronto Notes, Therapeutic Choices and others. Edited and organized for the sake of all attendances of the Canadian Osce Exams: NAC OSCE and MCCQE2. by: Dr. Merlyn D’Souza and Dr. Zeev Gross, Spring 2011. Page 104

ACUTE HA x 10 days: 67M in ER HISTORY: OCD PQRSTUV Onset: Sudden Vs Gradual Course:  All the time  Is it increasing or decreasing or is it the same?  VARIATION: Did you notice any variation?  Is it the same throughout the day  Does it awaken you at night? (EMPATHIZE++++) Duration:  How long the whole disorder  How long each attack  How frequent: off & on Posn:  Unilareral/Bilateral  Where is it exactly?  Does the part where it hurts is tender (Temporal Arteritis) & do you feel like a cord-like structure there? Quality:  Throbbing  Burning  Tightness/Pressure  Ice pick like Radiation? front,side,back of head,or in the eyes,ears or throats? Severity: On a scale of 1 – 10 Can you say It is the worst HA of your life? Timing Triggers:(not when single episode) U: Qns for empathy**  How has it affected U in your daily life?  How r U coping with it?  How do U feel abt it?  What r your expectations from today’s visit? V= deja Vu Has it happened before? Aggravating factors:  Eating (Jaw claudication) Please note that this is only a draft version based on several sources, including: Dr. Basel Mohasen’s lectures, Toronto Notes, Therapeutic Choices and others. Edited and organized for the sake of all attendances of the Canadian Osce Exams: NAC OSCE and MCCQE2. by: Dr. Merlyn D’Souza and Dr. Zeev Gross, Spring 2011. Page 105

 Bending forwards/Coughing/Lifting/Lying down (Inc ICP)  Lights/Certain foods etc (Migraine)  Eyestrain (vision correction)  Alcohol (cluster H) Alleviating fcts: Did you try any meds & were they helpful? Assoc.Symptoms: In addition to your headache did you notice any other symptoms: (Try & do constitutional sx first as you may forget them) Fever/Neck pain/Photophobia/Skin rash/Ear infection NEURO Screening: 1. Vision changes: What type of problem? 2. Hearing abnormalities 3. Difficulty swallowing 4. Weakness/Numbness 5. Difficulty finding words 6. Difficulty in balance or repeated falls 7. Changes n bowel/Urine Loss of bladder control 8. LOC 9. H/o Seizure MOOD Changes MEMORY problem Changes in CONCENTRATION Has anyone told you that you’ve ben acting strangely? MSK Screening Is there pain in your joints For how long? Can you raise your arms above your head? H/O INJURY: To head Did you have a fall & hurt your head? EXTRACRANIAL: EYE: Did you notice any redness or need eyeglasses? Sinusitis: Facial pain/flu Throath pain Dental pain Please note that this is only a draft version based on several sources, including: Dr. Basel Mohasen’s lectures, Toronto Notes, Therapeutic Choices and others. Edited and organized for the sake of all attendances of the Canadian Osce Exams: NAC OSCE and MCCQE2. by: Dr. Merlyn D’Souza and Dr. Zeev Gross, Spring 2011. Page 106

RISK FCTS: Do you smoke Take EtOh Take recreational drugs?

PMH: Have you taken pain killers,if +ve: How much & for how long (rebound HA) Did you take any OTC or herbal meds? Are you allergic to anything? Have you ever been diagnosed with HTN/DM/Stroke/MI/Ca? Were you ever hospitalized or underwent Surgery? FAMILY H: HTN/DM/Stroke/MI SOCIAL H: Who lives with you? How do you support yourself financially? DIAGNOSIS: Temporal Arteritis

Back to Content

Please note that this is only a draft version based on several sources, including: Dr. Basel Mohasen’s lectures, Toronto Notes, Therapeutic Choices and others. Edited and organized for the sake of all attendances of the Canadian Osce Exams: NAC OSCE and MCCQE2. by: Dr. Merlyn D’Souza and Dr. Zeev Gross, Spring 2011. Page 107

24 M/HA 6 wks Office 10 mins H & C Primary Headache

Secondary Headache

Type

%

Type

%

Migraine

16

Systemic infection

63

Tension-type

69

Head injury

4

Cluster

0.1

Vascular disorders

1

Idiopathic stabbing

2

Subarachnoid hemorrhage

1 h and ≤ 4.5 h before alteplase administration 3. Stroke deficit that is disabling or measurable on the NIH Stroke Scale 4. No intracranial hemorrhage on CT or MRI scan Exclusion criteria 1. Time of stroke onset unknown or > 4.5 h 2. Any hemorrhage on brain CT or MRI scan 3. Symptoms suggestive of subarachnoid hemorrhage 4. CT or MRI signs of acute hemispheric infarction involving more than 1/3 of the MCA 5. History of intracranial hemorrhage 6. Stroke or serious head or spinal trauma within the preceding 3 mo 7. Seizure at stroke onset 8. Systolic blood pressure ≥ 185 mm Hg or diastolic blood pressure ≥ 110 mm Hg or aggressive treatment (intravenous medication) necessary to reduce blood pressure to these limits 9. Recent major surgery Please note that this is only a draft version based on several sources, including: Dr. Basel Mohasen’s lectures, Toronto Notes, Therapeutic Choices and others. Edited and organized for the sake of all attendances of the Canadian Osce Exams: NAC OSCE and MCCQE2. by: Dr. Merlyn D’Souza and Dr. Zeev Gross, Spring 2011. Page 115

10. 11. 12. 13. 14. 15.

Arterial puncture at a noncompressible site within the previous 7 days Elevated activated partial thromboplastin time International normalized ratio > 1.7 Platelet count < 100 × 109/L Blood glucose concentration < 2.7 or > 22 mmol/L Any other condition that could increase the risk of hemorrhage after alteplase administration

Alteplase in Acute Ischemic Stroke: Monitoring5 Blood Pressure and Neurological Signs • Baseline, then Q15min × 2 h after starting alteplase • Then Q30min × 6 h • Then Q1H until 24 h after starting alteplase • Call MD if the systolic BP is > 180 mm Hg or if the diastolic BP is > 110 mm Hg on 2 or more occasions taken 5–10 min apart • Stop the infusion, obtain emergency CT scan and notify MD if there is neurologic deterioration, severe headache, or new onset of nausea or vomiting Blood Glucose • Call MD if glucose > 12 mmol/L Lines and Tubes • Delay placement of nasogastric tubes, indwelling catheters or intra-arterial pressure catheters Medications • No ASA, ticlopidine, clopidogrel, heparin or warfarin for 24 h • Acetaminophen 650 mg po or pr Q4H if body temperature is ≥ 38°C or for analgesia • O2 via nasal prongs or face mask to keep O2 saturation > 90% • After the alteplase infusion is completed, continue iv normal saline (with or without KCl) Investigations • CT brain scan after 24 h

Carotid endarterectomy (CEA)2 Patients with carotid territory transient ischemic attack or nondisabling stroke and ipsilateral 70– 95% internal carotid artery stenosis should be offered carotid endarterectomy within 2 weeks of the incident transient ischemic attack or stroke unless contraindicated. CEA is also appropriate for selected patients with moderate (50–69%) symptomatic stenosis. These patients should be evaluated by a physician with expertise in stroke management. Carotid stenting may be considered for patients who are not CEA candidates for technical, anatomical or medical reasons.

Antiplatelet therapy  

If intracranial hemorrhage is excluded by CT scan, but alteplase is not indicated, give ASA 160 mg immediately. This is followed by ASA 80–325 mg daily. When alteplase is used, wait until intracranial hemorrhage is excluded by CT scan 24 hours later and give ASA 160 mg once. This is followed by ASA 80–325 mg daily.

Please note that this is only a draft version based on several sources, including: Dr. Basel Mohasen’s lectures, Toronto Notes, Therapeutic Choices and others. Edited and organized for the sake of all attendances of the Canadian Osce Exams: NAC OSCE and MCCQE2. by: Dr. Merlyn D’Souza and Dr. Zeev Gross, Spring 2011. Page 116







Administer ASA as a suppository or via nasogastric tube to dysphagic patients. Use enteric-coated formulation for patients who can swallow. No evidence supports the use of ASA doses greater than 325 mg/day for secondary stroke prevention. The GI side effects of ASA are dose related. For patients who were taking ASA prior to their stroke, consider other antiplatelet agents, such as clopidogrel 75 mg daily or a combination of ASA and sustained-release dipyridamole 25/200 mg twice daily, although these regimens have not been tested in acute stroke. The combination of ASA and clopidogrel is not recommended for long-term secondary stroke prevention.

Anticoagulant therapy 

 



Immediate systemic anticoagulation with unfractionated heparin, low molecular weight heparin, heparinoids or specific thrombin inhibitors is not recommended in the setting of acute ischemic stroke, not even for patients in atrial fibrillation (AF), because there is no evidence of short- or long-term benefit. Specifically, reduction in early recurrent ischemic stroke is completely offset by an increase in major intracranial and extracranial bleeding.18 ASA is as effective as warfarin for secondary stroke prevention in patients in normal sinus rhythm, and does not require laboratory monitoring. For patients in AF, use warfarin at a dose to maintain the INR in the range 2.0 to 3.0, provided there are no contraindications to anticoagulation. For patients who cannot take warfarin, use enteric-coated ASA 80–325 mg daily. The best time to initiate anticoagulant therapy is unclear. For patients with minor strokes, start warfarin as soon as intracranial hemorrhage has been excluded by CT scan. For patients with major strokes, delay warfarin until a CT scan done about a week or two after the stroke has excluded hemorrhagic transformation of the infarct.

Blood pressure lowering treatment Randomized controlled trials have not defined the optimal time to initiate blood pressure lowering therapy after stroke.19 Oral blood pressure lowering treatment should be initiated (or modified) prior to discharge from hospital in patients whose blood pressure is ≥ 140/90.

IV. Restore Function of the Individual  

Outcomes are optimized by care on a stroke unit provided by a coordinated interdisciplinary team (Start rehabilitation as soon as the patient is medically stable. Family and community supports are important for social reintegration.

Therapeutic Tips 

The effectiveness of thrombolytic therapy with alteplase is exquisitely time dependent; delays of any sort should not be tolerated. A minority of patients present to hospital within the first 90 minutes of stroke onset, leaving limited time to act. Immediate contact with the patient, rapid triage, and (most importantly) staying with the patient continuously during the clinical assessment, CT scan, blood tests and consent procedures are vital in ensuring that the appropriate steps are being taken as rapidly as possible prior to alteplase administration. For example, it is not necessary to wait for hospital porters to take the patient to the CT scanner.

Please note that this is only a draft version based on several sources, including: Dr. Basel Mohasen’s lectures, Toronto Notes, Therapeutic Choices and others. Edited and organized for the sake of all attendances of the Canadian Osce Exams: NAC OSCE and MCCQE2. by: Dr. Merlyn D’Souza and Dr. Zeev Gross, Spring 2011. Page 117





  

Determining the time of stroke onset is critical in deciding to use alteplase, but checking the clock is not a natural reaction in the setting of an acute stroke. Encourage patients and families to think of “time anchors” (e.g., what was on the radio or TV at the time, or at what point in the patient's daily routine did the symptoms first occur). Patients with acute stroke are often unable to communicate. When possible, the next-of-kin should travel with the patient to hospital (or between hospitals if the patient is transferred) to provide collateral history and consent for treatment before the time window for intervention closes. If the patient is referred to a tertiary care hospital, have the stat blood work (CBC, INR) drawn at the community hospital and the results faxed to the referral centre as soon as possible. Point-of-care INR testing , if available, can provide results quickly. Signs of infarction on a CT scan done within 4.5 hours of stroke onset are usually subtle. If the CT scan of a patient being considered for treatment with alteplase shows a very definite infarct in a location that explains the presenting clinical symptoms and signs, recheck the time of onset.

Back to Content

Please note that this is only a draft version based on several sources, including: Dr. Basel Mohasen’s lectures, Toronto Notes, Therapeutic Choices and others. Edited and organized for the sake of all attendances of the Canadian Osce Exams: NAC OSCE and MCCQE2. by: Dr. Merlyn D’Souza and Dr. Zeev Gross, Spring 2011. Page 118

A 30 YOF with right arm weakness for 10 hours, Hx for 5m Intro Where is your weakness? Can you still work with your hand or no? Do you have burning or tingling sensation on your hand or shoulder? How about numbness? Any problem on your right foot? Lt. arm or leg? Is it the first time? (If the patient says that she had it before than: “When was it? How long did it last? Which medication did she take?) Did you fall or lost your consciousness? Any change in your vision? Loss of vision? Double vision? Blurry vision? Any change in your hearing? Buzzing sound? Diffucult in finding words? Any change in balance? Any change in urination and bowel movement? When you bent your neck do you fill electrical shock along your spine? Do you difficult to swallow? Have you ever had dizziness, headlightedness, loss of consciousness, jerky movement, seizure? How is your mood / concentration / memory? Any change in your personality? When you touch your face do you feel any electrical shock? Uhthoff’s sign: when they get hot water or hot weather – trigger for their symptoms (especially optic neuritis). Review systems from head to toe: chest pain, heart racing, sob, cough and phlegm, abdominal pain, nausea and vomiting, joint pain, skin rash, diabetes, thyroid disease, anemia CSx RF for MS, PMHx, FMHx

Back to Content Please note that this is only a draft version based on several sources, including: Dr. Basel Mohasen’s lectures, Toronto Notes, Therapeutic Choices and others. Edited and organized for the sake of all attendances of the Canadian Osce Exams: NAC OSCE and MCCQE2. by: Dr. Merlyn D’Souza and Dr. Zeev Gross, Spring 2011. Page 119

Please note that this is only a draft version based on several sources, including: Dr. Basel Mohasen’s lectures, Toronto Notes, Therapeutic Choices and others. Edited and organized for the sake of all attendances of the Canadian Osce Exams: NAC OSCE and MCCQE2. by: Dr. Merlyn D’Souza and Dr. Zeev Gross, Spring 2011. Page 120

PAEDS 1. 2. 3. 4. 5. 6. 7. 8.

Pediatrics (30) Psychiatry (30) Physical exam (30) Management (12) OBGYN ( Communication Skills (10) Counseling (10) Medicine (CVS 15, Neu 15, Med 20)

Pediatrics Consider abuse There are no children in the room, only parents. Maternity leave – either husband or wifes.

Please note that this is only a draft version based on several sources, including: Dr. Basel Mohasen’s lectures, Toronto Notes, Therapeutic Choices and others. Edited and organized for the sake of all attendances of the Canadian Osce Exams: NAC OSCE and MCCQE2. by: Dr. Merlyn D’Souza and Dr. Zeev Gross, Spring 2011. Page 121

A child 9m – chronic diarrhea (CF, Celiac, HIV; Lactose deficien cy)

Back to Content 5y.o fever – take history. Skin rash – ask questions about it (distribution, relation to vfever) HSP

Back to Content Son, 3y.o is coughing for 4wk and they want to renew his antibiotics This shows there was a condition Ask What Ab,for which condt,When?. Don’t waste your time –R/O: Hyperactive airways or is it infection that has not cleared,or could be asthma.

Back to Content A mother just delivers a baby who is IUGR all questions should about pregnancy and delivery. 1. Reassure her, 2. Note appearance of the child 3. Note Paediatrician’s visit, 4. History of pregnancy/Obstetric History 5. Family history. In case mum was smoking,taking alcohol & drugs & asks if her fault if child has IUGR Don’t reproach her – it is NOT her mistake. It is a multi-factorial condition.Can be due to various causes,some genetic,pregnancy,related to baby Because safe levels of smoking,drugs & alcohol not known, We always recommend not to smoke or drink for futurepregnancies. Parents are concerned that their child is not growing enough: What his weight in birth.

[AGEx2+8]

14m 8Kg (birth weight 3.5Kg). He is underweight. CC

OCD COCA-B AA ASx PMHx Please note that this is only a draft version based on several sources, including: Dr. Basel Mohasen’s lectures, Toronto Notes, Therapeutic Choices and others. Edited and organized for the sake of all attendances of the Canadian Osce Exams: NAC OSCE and MCCQE2. by: Dr. Merlyn D’Souza and Dr. Zeev Gross, Spring 2011. Page 122

SHx 0-6m:

BINDE

Birth – Pregnancy: Was it a planned pregnancy? 1. Did you have any regular follow-up? 2. Did you have any US? Was it normal or not? 3. During your pregnancy did you have any fever or skin rash? 4. Any contact with sick person or cats? 5. Any medication/smoking/drugs/alcohol? 6. Screened for HIV/Syphilis/GBS/Hepatitis B? Blood group?

Birth – Delivery: 1. 2. 3. 4. 5. 6. 7. 8. 9. 10. 11. 12.

Was it in term or not? What is the route? (Cs/NVD) How long it took? (18hr is normal for primi, 12hr for multi), Early gush of water? Any need for augmentation? What was the APGAR score? Did the baby cry immediately? Did your baby need any special attention? Any bulging or bruising in his body? When were you sent home?(C/S 3d, V/D – 1d). After delivery did you have any fever/vaginal discharge/any medication? Were you told that your baby had any congenital deformity?

Immunization – if he says that the child is not immunized you have to inquire for the reason. If he is not vaccinated because the parent is busy – look for child abuse RED FLAG.Ask wt & milestones If it is due to religion believe – you don’t have to ask more. Otherwise – ask about nutrition.

Nutrition – WEIGHT: 1. What his weight today, 2. Weight at birth, 3. Highest weight, Growth chart. X (birth), 5m-2x, 1y-3x, 2y-4x. Weight: Agex2+8 H (birth, about 50cm), 1y-1y, 2y-1.75H (half of his adult height), 4y-3.5H HC (at birth): 35cm What do you FEED your baby If formula: – When did you start the formula If B Fed at all Did you consider B feeding? what type of formula do you use? How do you prepare it? Please note that this is only a draft version based on several sources, including: Dr. Basel Mohasen’s lectures, Toronto Notes, Therapeutic Choices and others. Edited and organized for the sake of all attendances of the Canadian Osce Exams: NAC OSCE and MCCQE2. by: Dr. Merlyn D’Souza and Dr. Zeev Gross, Spring 2011. Page 123

Was there any changes in the feeding? Did you add any solid food or supplements (any fortified serials or iron) do you feed him with any bread, solid food – when started the diarrhea (before the solid food or after?) Development – At the end 1y they use words, 2y – two words at one sentence, 3y – 3words in one sentence; 4y – speak normally. Gross motor: role – 4m, seat- 6m, crowling – 9m, standing – 1y, climbing upstairs – 18y, riding bicycle – 3y Environment – with whom do you live at home? Any other children? Relation between your child and other households? Who spends most of the time with the child? Financially how do you support yourself? Do you live in your own house? Do you have basement in your house? Anybody drinks or uses drugs? Building – basement (mold) and Old houses (lead poisoning). 6-12m: School Performance: comparing the grades between now and previous. >12-14 yrs: HEAADDSSS Home: with whom do you live? Education: Which grade? How are your marks? What do you want to be? Recent drop in grades? Activities: Any hobbies? (in case of epilepsy – ask for the risky activities) Alcohol: do you smoke, drink, (a lot of people of your age might experiment with drugs? How about you?), Smoking Diet: do you have any special diet? Drugs: have you ever tried recreational drugs? Smoke Sexual: are you in relationship? Suicide: how is your mood? Questions for dehydration: Does baby have tears when crying? How many times you pee? How many times you change his diapers?

Back to Content

Please note that this is only a draft version based on several sources, including: Dr. Basel Mohasen’s lectures, Toronto Notes, Therapeutic Choices and others. Edited and organized for the sake of all attendances of the Canadian Osce Exams: NAC OSCE and MCCQE2. by: Dr. Merlyn D’Souza and Dr. Zeev Gross, Spring 2011. Page 124

5 Day old infant with yellow discoloration since he was 2 days old A mother who is after 5days from deliver (if she uses “jaundice” – what do you mean be that?). Is it early in the second day is it pathological. Late in the second day – it has no value. Make sure that the baby is stable – Red flags: 1. High pitched crying, 2. Poor feeding & Poor sucking 3. Floppy baby, 4. If above three are present, it is a problem, decide to reassure her or tell her you’ve to do a physical exam& admit If you have to take history and counsel ------>Reassuarence Only history-------->Pathological m/p it is not physiology What is the name of your child? (He is yellow) Good you are here; I hope you can reassure you at the end. OCD Tell me more about it since the moment it started O: Is it early in the second day is it pathological. Late in the second day – it has no value C: D: Where did you notice it? Is it spreading? Did it reaches the legs? Is it getting darker? In addition to it did you notice any dark urine? Pale stool (in bliary atresia – pale stool from the beginning?) ASx: INFECTION: In addition to that did you notice any fever, cough, discharge from ear,discharge diarrhea, vomiting, foul smell urine, is he crying, is he floppy, is he sucking well, (RED FLAGS) Rash, Dehydration: how many diapers did he change, any tears Transition: I am going to ask you some questions to see if any conditions cause this issue back to your pregnancy FHx: of liver disease and blood disease. If she are concerned – why you are concerned (will he be mental retarded).

Back to Content Please note that this is only a draft version based on several sources, including: Dr. Basel Mohasen’s lectures, Toronto Notes, Therapeutic Choices and others. Edited and organized for the sake of all attendances of the Canadian Osce Exams: NAC OSCE and MCCQE2. by: Dr. Merlyn D’Souza and Dr. Zeev Gross, Spring 2011. Page 125

Mike Tyson, Child crying for the last 10d, 6w years old. A child who’s with infantile colic. How do you feel when your child is crying. Crying child could be colic,screen for abuse,see how parents handle it The crying might cause abuse Weight, dehydration, Start to observe the body language of the father. OCD O; At that time was there any illness,like fever,runny nose C; Off & On/all the time Every single day,every day,how many days/week also “is he crying during the night?” – how does it affect you and your wife?) Aggravating FCts: Any chance he is hungry? Any chance he is wet & neds a diaper change? Diaper rash? Any chance he is too hot or cold? Alleviating Sx: Do you soothe him/hug him/carry him & walk/take hime for a ride/listen to music? Do you burp him/rock him/Skhe him? If Yes: How many times? When was the last time? What happens to him when you shake him? Does he stop crying? Does he pass out? (Children at this age cannot express their discomfort& only means of communication is by crying“I am going to ask some questions to see if there is any reason for this crying?” ASx; INFECTION: Fever, sweating, tender points in his body Running nose, coughing, vomiting, discharge from his ear, yellow discoloration, fowel smell urine, GI does he have distension of abdomen Gases Does he draw up his legs & cry Any relation to feeds BINDE (Partial) N 1st P Planned pregnancy, Reg F/u Was it term P Please note that this is only a draft version based on several sources, including: Dr. Basel Mohasen’s lectures, Toronto Notes, Therapeutic Choices and others. Edited and organized for the sake of all attendances of the Canadian Osce Exams: NAC OSCE and MCCQE2. by: Dr. Merlyn D’Souza and Dr. Zeev Gross, Spring 2011. Page 126

Any illness Smoke/Drugs, Term, Complicated, Needed special attention, separation, any congenital abnormality Environment: financial how do you support yourself, any financial stress, with whom do you live, repeated visits to ER, anyone in home have psychiatric problems/drugs/alcohol, relationship with your partner PMHx – diseases, hospitalizations

Back to Content

Please note that this is only a draft version based on several sources, including: Dr. Basel Mohasen’s lectures, Toronto Notes, Therapeutic Choices and others. Edited and organized for the sake of all attendances of the Canadian Osce Exams: NAC OSCE and MCCQE2. by: Dr. Merlyn D’Souza and Dr. Zeev Gross, Spring 2011. Page 127

Rita Gordon, mother of 5w who vomit for the last 10d Pyeloric stenosis GERD Infection Alleric to milk overfeeding

Not Pyloric stenosis If the colour of vomit is yellow or greenish discolouration Not projectile + Wt loss GERD No wt loss at 6 weeks Wt loss at 18 mo due to anaemia,due to bleeding due to oesophageal bleeding

Confidential – give it early according to the cues. Depressed: after my son become vomit or preceding the vomit. Not reliable history. CC OC fD O: C; Off & on/All the time How many/day Increasing/Decreasing or same? COCA±B Forceful Feed: Formula/Breast? COCA How much F do you give? & How much does he vomi t out? AA: Any particular posn improves it? ( GERd upright better) IMPACT: WT & Dehydration Do you feel he is still hungry after you feed him? How many diapers do you change,Now & at the beginning? ASx: wt & s/o dehydration Gerd – no weight loss Pyeloric stenosis – yellowish colour (ask specifically about the colour, relation to feeding – up to half hour can be related to PS, what about position, do you burp him?) ASx: Infection – any signs of infection BINDE: N 1st If formula fed? Did you change the Formula? Have you considered breastfeeding? Is there any reason not to breast feeding? Please note that this is only a draft version based on several sources, including: Dr. Basel Mohasen’s lectures, Toronto Notes, Therapeutic Choices and others. Edited and organized for the sake of all attendances of the Canadian Osce Exams: NAC OSCE and MCCQE2. by: Dr. Merlyn D’Souza and Dr. Zeev Gross, Spring 2011. Page 128

Overfeeding – overweight Allergy – less likely if she uses it from birth Abdominal distended B PWas it planned pregnancy Were there regular F/u? How do yu feel about being a mum? MOOD & INTERSET Any chance of being depressed? Any chance you feel like harming the baby or yourself? Do you have any support at home? “I see you are preoccupied / overwhelmed” Child abuse/neglect If there is a growth chart – it it is from the beginning. 4 min on the child, last 1 min to concentrate to the mother.

Back to Content

Please note that this is only a draft version based on several sources, including: Dr. Basel Mohasen’s lectures, Toronto Notes, Therapeutic Choices and others. Edited and organized for the sake of all attendances of the Canadian Osce Exams: NAC OSCE and MCCQE2. by: Dr. Merlyn D’Souza and Dr. Zeev Gross, Spring 2011. Page 129

Sandra Bullock, 19 y.o, 8m child, pale. Intro Name of the child CC: Anemia ? OCD IMPACT Causes: Red Flags (bleeding & BINDE) Past MH FH ENv: Old house Pale – what do you mean? Who told you that OCD If told by another person/ If you think about it,any chance he was pale before that or just now?,& you were unaware I like to see how it has affected your son: IMPACT: Is he as active as before? Crawl? Playful as before? LOC? Heavy breathing? Stop to breath when you feed him? T: I’m going to ask you some questions that could be the cause of this? Asx: Infection: Sweat, Fever, Loose of weight, Painful points? Does your child have bleeding? Bruises? Coughing blood? Tarry stool? BINDE: N 1st What do you feed him? (B/F) Any solids /supplements P: Was it a term Preganacy? IMMUNIZATION: ENV: With whom do you live? Any financial concerns? Old/new home Do you’ve a supportive family? PMH: FH: Any bleeding disorder? Repeated lver disease Any gall bladder disease or splenectomy Certain blood disease are more common in particular parts of the world & for that reason I need to know your & partner’s ethnic background.

Back to Content Please note that this is only a draft version based on several sources, including: Dr. Basel Mohasen’s lectures, Toronto Notes, Therapeutic Choices and others. Edited and organized for the sake of all attendances of the Canadian Osce Exams: NAC OSCE and MCCQE2. by: Dr. Merlyn D’Souza and Dr. Zeev Gross, Spring 2011. Page 130

ANEMIA: 29/F MCV Inc (Counsel) D/d:[TN10/H21] A. MEGALABLOBALSTIC: a) B12 defeciency: I. Diet (vegan) II. Gastric: a) Mucosal atrophy b) Pernicious An c) Post G-ectomy III. Intestinal Absorption a) Malabsorption (Crohn’s,celiac sprue,pancreatic disease) b) Stagnant bowel (blind loop,stricture) c) Fish tapeworm d) Resection of ileum b) Folate deficiency I. Diet II. Intestinal malabsorption III. Drugs/Chemicals: a) Alcohol b) Anticonvulsants c) Methorexate d) Birth control pills IV. Inc demands: V. Pregnancy/Hemolysis/Hemodialysis/Psoriasis c) Drugs (Methroxate,azathioprine) B.Non Megalobalstic: I. Liver disease II. Alcohol III. Hypothyroid IV. Myelodysplastic syndromes Start By saying: I’ve the results of your test with me & before I proceed I need to get some information abt you that will help me understand: If Pt asks if Serious: STOP & ask WHAT is her concern. There can be many reasons for this result,though most are simple,however some can be serious, 2QNS: 1. What is the reason for doing the test 2. Is it the first time? Then explain the results

Please note that this is only a draft version based on several sources, including: Dr. Basel Mohasen’s lectures, Toronto Notes, Therapeutic Choices and others. Edited and organized for the sake of all attendances of the Canadian Osce Exams: NAC OSCE and MCCQE2. by: Dr. Merlyn D’Souza and Dr. Zeev Gross, Spring 2011. Page 131

   





Search for the cause of iron deficiency, including very careful consideration of occult gastrointestinal bleeding Menorrhagia must be convincing before it is accepted as the sole cause of iron deficiency. This may be an opportunity for early recognition of a gastrointestinal malignancy—don’t miss it! A reticulocyte response should be evident within one week of beginning iron therapy, with subsequent improvement in the Hgb of about 10 g/L every 7–10 days. If the Hgb fails to respond as anticipated, consider that there may be: o ongoing blood loss o use of other medications that impair iron absorption o a different or concurrent cause of anemia and/or an impaired erythropoietic response o compliance issues Gastrointestinal side effects are the most common reasons for non-compliance: o use a graduated approach to dosing. Begin with a single tablet taken after a meal. On a weekly basis, as tolerance permits, add another tablet until the patient is taking one dose with each meal. Thereafter, gradually shift the timing of the doses to the beginning of meals o small oral doses may be adequate in patients that are susceptible to gastrointestinal upset. In the elderly, daily doses of elemental iron as low as 15 to 50 mg are effective in the treatment of iron deficiency anemia9 In pregnant women, 20 mg/day of elemental iron, started at 20 weeks' gestation, is sufficient to prevent iron deficiency11 o iron contained in enteric-coated tablets is poorly absorbed. These products should be avoided Some physicians replenish iron stores while others prefer to stop therapy when the Hgb normalizes, so that further blood loss will not be masked by robust iron stores. As a compromise: o completely replenish iron stores when the cause of iron deficiency has been identified and corrected o do not replenish iron stores when investigation has failed to

Please note that this is only a draft version based on several sources, including: Dr. Basel Mohasen’s lectures, Toronto Notes, Therapeutic Choices and others. Edited and organized for the sake of all attendances of the Canadian Osce Exams: NAC OSCE and MCCQE2. by: Dr. Merlyn D’Souza and Dr. Zeev Gross, Spring 2011. Page 132

Therapeutic Tips    



Search for the cause of iron deficiency, including very careful consideration of occult gastrointestinal bleeding Menorrhagia must be convincing before it is accepted as the sole cause of iron deficiency. This may be an opportunity for early recognition of a gastrointestinal malignancy—don’t miss it! A reticulocyte response should be evident within one week of beginning iron therapy, with subsequent improvement in the Hgb of about 10 g/L every 7–10 days. If the Hgb fails to respond as anticipated, consider that there may be: o ongoing blood loss o use of other medications that impair iron absorption o a different or concurrent cause of anemia and/or an impaired erythropoietic response o compliance issues Gastrointestinal side effects are the most common reasons for non-compliance: o use a graduated approach to dosing. Begin with a single tablet taken after a meal. On a weekly basis, as tolerance permits, add another tablet until the patient is taking one

Please note that this is only a draft version based on several sources, including: Dr. Basel Mohasen’s lectures, Toronto Notes, Therapeutic Choices and others. Edited and organized for the sake of all attendances of the Canadian Osce Exams: NAC OSCE and MCCQE2. by: Dr. Merlyn D’Souza and Dr. Zeev Gross, Spring 2011. Page 133



dose with each meal. Thereafter, gradually shift the timing of the doses to the beginning of meals o small oral doses may be adequate in patients that are susceptible to gastrointestinal upset. In the elderly, daily doses of elemental iron as low as 15 to 50 mg are effective in the treatment of iron deficiency anemia9 In pregnant women, 20 mg/day of elemental iron, started at 20 weeks' gestation, is sufficient to prevent iron deficiency11 o iron contained in enteric-coated tablets is poorly absorbed. These products should be avoided Some physicians replenish iron stores while others prefer to stop therapy when the Hgb normalizes, so that further blood loss will not be masked by robust iron stores. As a compromise: o completely replenish iron stores when the cause of iron deficiency has been identified and corrected o do not replenish iron stores when investigation has failed to

Please note that this is only a draft version based on several sources, including: Dr. Basel Mohasen’s lectures, Toronto Notes, Therapeutic Choices and others. Edited and organized for the sake of all attendances of the Canadian Osce Exams: NAC OSCE and MCCQE2. by: Dr. Merlyn D’Souza and Dr. Zeev Gross, Spring 2011. Page 134

32 M Fever & Tiredness x 6 wks ---- 10 mins focused History 3 Scenarios: H/O Splenectomy IV drug user Unprotected Intercourse After introduction,Analyse Fever Constitunat Sx Then go to causes from Head to toe End with Liver Risk Fcts Travel Drugs PAST MH FH Social H (Which is linked to Risk Fcts) Can you tell me more about your fever from the moment it started? O; Sudden/Gradual When it first started did you have any other illness? Did you seek medical attention then? What made you come in today? C; 1. Is it on & Off/All the time/everyday 2. Does it inc/Dec or is it the same? 3. Any variation during the day, like more in morning? Any particular patern? 3 rd or 4th day or alt days 4. Did you measure it? 5. How often do you measure it? 6. Which was the highest temp? 7. Does it increase at night? 8. Did you take any meds?/were they helpful? 9. Is it the first time or have you ever had it before? 10. Anything increases or decreases it?

Please note that this is only a draft version based on several sources, including: Dr. Basel Mohasen’s lectures, Toronto Notes, Therapeutic Choices and others. Edited and organized for the sake of all attendances of the Canadian Osce Exams: NAC OSCE and MCCQE2. by: Dr. Merlyn D’Souza and Dr. Zeev Gross, Spring 2011. Page 135

CONSTITIONAL Sx: Fever/chills/N Sweats/wt loss/Lumps/bumps TRANSITION: I’ve to ask a couple of more qns to help me come to a diagnosis. If you’ve concerns at any time please tell me & I will answer them CNS: HA/N/Vx/Photophobia/neck pain/Neck stiffness Ear pain/Discharge from ear/runny nose/Facial pain/Sinusitis/Sore th/Difficulty swallowing Dental pain/Tooth ache CVS: H racing/Chest pain/SOB RS: Cough/Phglem/wheezing/H-maemesis Contact with TB/Have you ben screened for TB? GI Abd Pain/Diarrhoea/Malena GU Flank pain/burning urine/bld in urine/Inc freq in passing urine MSK Jt pain/Swelling/Skin rash/Ulcers in mouth/red eyes Have you ever been Dsed as a condt called Autoimmune Disease? Or has anyone else in your family been diagnosed? LIVER DISEASE:  Have you ben screened for liver disease?  Have you been vaccinated against Hepatitis A & B?  Sx of Ac Liver Disease: Yellow discoloration of skin & nails/Pale stools/Dark urine/Itchy skin  Sx of Ch L Disease: Inc abd girth/bruises /leg swelling/vomiting bld/memory changes TRANSITION: I’ve to ask you some questions to see if you were exposed to liver disease without being aware of ,some of these qns may be personal, but it is imp that I ask them.All that you tell me is confidential & the information will not be released without your permission, unless I’m requested by law TRAVEL & CAMPING H Travel outside Canada H/o eating raw fish,raw shell fish.Have you visited a new restrauant? Please note that this is only a draft version based on several sources, including: Dr. Basel Mohasen’s lectures, Toronto Notes, Therapeutic Choices and others. Edited and organized for the sake of all attendances of the Canadian Osce Exams: NAC OSCE and MCCQE2. by: Dr. Merlyn D’Souza and Dr. Zeev Gross, Spring 2011. Page 136

H/O Surgery/Hospitalizations Donated/recvd bld Tattooing/Piercings Smoke/Drink Alcohol/Recreational drugs? Any injectable drugs? SOCIAL H: Whom do you live with? How long have you been with your partner? If for a specified time with a partner,ask if had any other sexual partners,though this qn is personal,I’ve to ask it as it is imp: When was the last time you’d sex with another partner/ Did you use a condom then? If YES: Ask Discharge/Lumps in groin/Genital ulcers How is wife:Does she have: Fever/Sx/Discharge? RISK FCTS: How do you support your self financially? Have you ben exposed to body fluids/TB H/Ca Any contacts with fever? HIV SEXUAL HISTORY: Before marriage or before current relationship; 1. Did you have sexual partners? 2. At what age were you sexually active? 3. From that time till now, how many partners did you have? 4. Did you practise Safe Sex (Use of condoms?) 5. What is your sexual preference? M? F? Or Both? 6. What type of sexual activities do you prefer? Anal/vaginal/oral 7. Were you ever screened or diagnosed for STIs? 8. Did you have any sexual relationship besides your regular partner

Laboratory investigations: o

HIV antibody test (repeat to rule out lab error)

Please note that this is only a draft version based on several sources, including: Dr. Basel Mohasen’s lectures, Toronto Notes, Therapeutic Choices and others. Edited and organized for the sake of all attendances of the Canadian Osce Exams: NAC OSCE and MCCQE2. by: Dr. Merlyn D’Souza and Dr. Zeev Gross, Spring 2011. Page 137

o

plasma HIV RNA level (viral load) with the CD4 lymphocyte count is the best prognostic marker for progression to AIDS and survival.

o

viral drug resistance mutations become harder to detect over time. Therefore conduct a resistance test at entry into treatment program even if use of antiretroviral treatment is not currently contemplated2 CD4 lymphocyte count and percentage is useful in determining where a patient lies in the continuum of HIV disease and the need for specific intervention (Table 1). Knowledge of the CD4 count can also help to narrow the differential diagnosis in a symptomatic HIV-infected patient. In adults, a CD4 count of 430 to 1360 cells/μL (0.43 to 1.36 Giga/Litre or G/L) is considered normal in most laboratories screen all patients for the presence of the HLA-B*5701 allele before starting or restarting abacavir.2 , 3 A positive result indicates a very high risk for severe allergy to abacavir and should be filed in the patient's chart perform a tropism assay to determine the chemokine receptor status (CCR5, CXCR4 or dual-mixed tropic) if considering use of the CCR5 inhibitor maraviroc. A plasma viral load of at least 1000 copies/mL is required to perform this test CBC, differential and platelet count liver (AST, ALT, GGT, LDH, CPK, alkaline phosphatase, bilirubin, INR, albumin) and renal (BUN, creatinine, electrolytes, urinalysis) profiles metabolic profiles (fasting glucose and lipids—total cholesterol, LDL, HDL, triglycerides) hepatitis B, hepatitis C, syphilis, cytomegalovirus (CMV) and toxoplasmosis serologies cultures and smears for sexually transmitted diseases as indicated tuberculosis skin tests, sputum cultures and smears for mycobacteria as indicated chest x-ray

o

o

o

o o o o o o o

Management of Patients with HIV Infection CD4 Count (cells/μL) At all levels

Action      



General counselling (safer sex, nutrition, need for follow-up, importance of adherence, etc.) History and physical examination every 3– 6 mo Plasma viral load and CD4 count at least every 3– 4 mo Herpes suppression if frequent recurrences (more than 4–6 episodes per year) Syphilis serology Pneumococcal vaccine; hepatitis A and B vaccines if appropriate; update diphtheria, tetanus and inactivated polio vaccines as needed; consider annual influenza vaccinations TB skin test and isoniazid prophylaxis if indicated (consider

Please note that this is only a draft version based on several sources, including: Dr. Basel Mohasen’s lectures, Toronto Notes, Therapeutic Choices and others. Edited and organized for the sake of all attendances of the Canadian Osce Exams: NAC OSCE and MCCQE2. by: Dr. Merlyn D’Souza and Dr. Zeev Gross, Spring 2011. Page 138

CD4 Count (cells/μL)

Action repeating skin test yearly)

< 500

 

Plasma viral load and CD4 count every 3– 4 mo Clinical evaluations and laboratory investigations at least bimonthly if symptomatic, diagnosed with AIDS, or on antiretroviral therapy

< 200



Start prophylaxis for Pneumocystis jirovecii pneumonia (PCP)

< 100



Start toxoplasmosis prophylaxis if seropositive and not on trimethoprim/sulfamethoxazole for PCP prophylaxis

< 75



Consider MAC prophylaxis

< 50



Screen by an ophthalmologist for early CMV retinitis (repeat at 3– 6 mo intervals) or consider CMV prophylaxis

Advise patients with HIV infection and immunosuppression that their risk of infections can be reduced by following good hygienic practices. 1. Ensure thorough hand washing after contact with potentially contaminated substances (diapers, soil, uncooked meat and produce) or handling pets 2. Avoid raw or uncooked meat and eggs, e.g., Caesar salad 3. Drink from treated water sources only 4. Avoid handling sick animals or pet (especially cat) litter 5. Avoid cat scratches and do not allow cats to lick wounds 6. Avoid contact with reptiles

Back to Content

Please note that this is only a draft version based on several sources, including: Dr. Basel Mohasen’s lectures, Toronto Notes, Therapeutic Choices and others. Edited and organized for the sake of all attendances of the Canadian Osce Exams: NAC OSCE and MCCQE2. by: Dr. Merlyn D’Souza and Dr. Zeev Gross, Spring 2011. Page 139

45M with tiredness x 6 weeks As I understand you’re having Tiredness since 6 weeks, can you tell me more about it since it all started? Pt says he is concerned. STOP & ask about his concern. Pt says, he is also never been so tired before. Clarify: What do you mean about Tiredness?  Sometimes I do not feel refreshed after sleep.  Do you feel lack of energy? Like you cannot move your arm above your head. The Statement: NOT REFRESHED ANYMORE points to an organic cause

OCD + Relation to sleep +/- Mood If Mood Sx + ------ MOAPS If Organic cause ---- Red Flags OCD At onset you can ask if there were any flu-like Sx initially (Thyroiditis)  Do you sleep more  When do you go to bed?  Do you wake up in the middle of the night  Which time of day/Night do you feel most?  Ask Nature of work; If shift  With whom do you sleep?  Does your partner C/o you snoring or jerky movts of limbs? (Restless Leg)  Do you feel better in morning or evening(if tired in morning-->Depression If evening-------->Organic cause)  Depression 1st Low mood then tired  In organic 1st Tired the Low mood 11. Ask Constitutional Sx first: Fever/Wt loss/Night sweats/Chills/Lumps & Bumps Then quick review of Sx: 12. Cardiac: Chest Pain/SOB/Palpitations 13. Pulmonary: Cough/Wheezing/Phglem 14. GI: N/V Abnormal bowel movts/Diarrhoea/Malena/ 15. Liver: Dark urine/Yellow sclera/Abd pain/Loss of aooetite/Pale stool/Itchy skin 16. GU: Change in color of urine/Amt of urine/Cloudy/Frothy urine/Dysuria/Facial swelling 17. Anemia: Bleeding gums/Easy bruising/Malena/Haematuria/Female: Meorrhagia & LMP Please note that this is only a draft version based on several sources, including: Dr. Basel Mohasen’s lectures, Toronto Notes, Therapeutic Choices and others. Edited and organized for the sake of all attendances of the Canadian Osce Exams: NAC OSCE and MCCQE2. by: Dr. Merlyn D’Souza and Dr. Zeev Gross, Spring 2011. Page 140

18. Autoimmune Disease: Joint Pains/Skin rashes/Oral Ulcers 19. Endocrine: Thyroid: Feel hot/Cold Skin Moist/Dry/Tremors/Wt loss If Pt has thyroid Sx,ak if on Thyroxine When Dsed If thyroxine levels are monitored? 20. DM: Risk fcts: FH & Lifestyle Once Pt has DM in history, GO over ALL Sx & Sy: RISK FCTS: 7. Diet 8. Exercise 9. FH 10. Smoking 11. Alcohol 12. Recreational drugs PAST MEDICAL HISTORY: Any medications/OTC/Herbal/LMP Surgery/Hospitalization FAMILY HISTORY: DM/HTN/Stroke/MI SOCIAL H: Habits With whom do you live? How do you support yourself financially?

Hypothyroidism (underactive thyroid) is a condition in which your thyroid gland doesn't produce enough of certain important hormones. Hypothyroidism upsets the normal balance of chemical reactions in your body. It seldom causes symptoms in the early stages, but, over time, untreated hypothyroidism can cause a number of health problems, such as obesity, joint pain, infertility and heart disease. The good news is that accurate thyroid function tests are available to diagnose hypothyroidism, and treatment of hypothyroidism with synthetic thyroid hormone is usually simple, safe and effective once the proper dosage is established.

Back to Content Please note that this is only a draft version based on several sources, including: Dr. Basel Mohasen’s lectures, Toronto Notes, Therapeutic Choices and others. Edited and organized for the sake of all attendances of the Canadian Osce Exams: NAC OSCE and MCCQE2. by: Dr. Merlyn D’Souza and Dr. Zeev Gross, Spring 2011. Page 141

NEEDLE STICK INJURY Michael Jackson, Nurse in hospital, Needle stick 20min ago; History and counsel, 10min Variations: Janitor who was pricked in junk yard (here touch TT prophylaxis) 0.3% - HIV; 3% HCV; 30% HBV “The treatment will be the same no matter what is the situation of the other patient...” Did anybody talk to him? Did he accept to get his HIV status? “By law we are not allowed to take his blood without his consent” QUESTIONS RELATED TO EVENT: 1. Size of needle 2. Blunt/hollow 3. Any blood on it 4. How deep was the injury? 5. What was gauge of needle? 6. Where was the location of the prick? 7. Any bleeding after that? 8. Whether he was wearing gloves? 9. What measures did he take? (Wash hands?) 10. Is it the first time? If Pt insists on doing HIV testing of the contaminated pt: I know it is of great concern about the pt’s HIV status, however from the ethical point of view we cannot do the HIV test without the pt’s consent. I can go after our interview & personally request him If Still he insists or ask for CD4 count: How do you think this will help us? It is a reasonable way of thinking. There are different conditions reflecting CD4 count & ethically not the right step We do it to obtain Pt’s information & not for the best interest of the pt However whatever the CD4 count it makes no sense in our management. We’ve to follow protocol: RISKS Give him the risks of being infected with N Stick injury: HIV------->0.3% Hepa C----->3% Hepa B-------> 30% .” In order to know what is the best line for you I need to ask you more questions. Do you know what the chances for getting infected are? (Out of 1000 people – only 3 will be affected). ASSESMENT: Being a health care provider – 1. Have you been vaccinated before for Hepa A & B 1.1 How many doses? 1.2 When was the last dose? 2. Liver Disease: Have you ever been yellowish? Itchiness? Dark urine? Pale stool? Btuises in body? C Sx: Repeated infections? Chronic diarrhea? Have you been screen for HIV or HCV? Please note that this is only a draft version based on several sources, including: Dr. Basel Mohasen’s lectures, Toronto Notes, Therapeutic Choices and others. Edited and organized for the sake of all attendances of the Canadian Osce Exams: NAC OSCE and MCCQE2. by: Dr. Merlyn D’Souza and Dr. Zeev Gross, Spring 2011. Page 142

I am going to ask you some questions if you were exposed before for any of the viruses mentioned above 1. Any travel outside Canada? 2. Any surgery 3. Any blood transfusions/ 4. Tattoos/Piercings SH With whom do you do live. For how long have you been together COUNSELLING: Whenever we face such a situation,we’re faced with three possible infections that could be transmitted: HIV------->0.3% Hepa C----->3% Hepa B-------> 30% Good news – HBV high risk but good plan; we are going to measure the titer of antibody in your blood. If Okay,you need not worry,if low you may need an Immunoglobulin or revaccination What do you know about HIV. If infected,some bcome carriers,not all develoe into AIDS,we will screen you today: For screening you we need to sample today to have base line. Most of the patients don’t react until 6wk, few until 6m – we have to take it in these times. If at the end of 6 mo tests come back negative, you’re cleared If not you’re infected Other options: we will also refer you to occupation clinic – Who will start you on prophylaxis treatment. It consists of three medications usually. They will explain to you which medication and describe the SE. It will decrease the chances by ...% HCV – This is of concern as We don’t have prophylaxis yet there is more than 50% to be carrier, more than 50% of them become chronic, 50% of them will get cancer. However certain medications might help like Interferon & Anti retrivirals “How do you feel about it?” Sometimes patients are overwhelmed and might harm yourself or others” From now till the results of your blood tests: Practise Safe sex Do not donate blood Joint a support group.” I wii file an incident report.”

Back to Content

Please note that this is only a draft version based on several sources, including: Dr. Basel Mohasen’s lectures, Toronto Notes, Therapeutic Choices and others. Edited and organized for the sake of all attendances of the Canadian Osce Exams: NAC OSCE and MCCQE2. by: Dr. Merlyn D’Souza and Dr. Zeev Gross, Spring 2011. Page 143

35y.o, male, counselling about HIV test Wants to do HIV test as his partner has tested =ve As I understand you’e here as you want a blood test.Can you tell me which blood test you specifically need? Can you tell me what made you come in today? **PT: I feel I’m at risk screen me for disease We cannot order all bl works, we’ve to look for a specific disease e.g: for TB we do a CXR/DM BSL/HIV Bl tests ** My Partner has tested for HIV + EMPATHY: I’m sorry, when was that? How is she doing now? How long have you two been together? How has it affected you? How do you feel? This can be a difficult for you, & you’ve done the right thing,& definitely we can arrange for a blood test “20 years ago we had no options, now even if you are positive we will have treatments and prophylaxis.” In order to get the diagnosis we need to do more questions.” 1. “Have you ever been screened for HIV”or HCV If Yes: When & where? 2. Any Sx OF HIV CSx, Mouth, Ulcers,difficulty swallowing RS;Cough ,Diarrhea, Discharge,Ulcers,Skin rash/yellowish, Dark urine/Pale stool I am going to ask you some questions if you were EXPOSED BEFORE for any of the viruses mentioned above 1. Any travel outside Canada? 2. Any surgery 3. Any blood transfusions/ Tattoos/Piercings? SEXUAL HISTORY Relationships now and before / Sexual predilection / Sexual practice When were you sexually active? How many partners have you had? Did you practise safe sex? PMH Any long term disease/hospitalization/allergies/medications COUNSELLING “What do you know about HIV?” “Nowadays we have better control over the disease. Once they start get the infection they called AIDS patients.” Please note that this is only a draft version based on several sources, including: Dr. Basel Mohasen’s lectures, Toronto Notes, Therapeutic Choices and others. Edited and organized for the sake of all attendances of the Canadian Osce Exams: NAC OSCE and MCCQE2. by: Dr. Merlyn D’Souza and Dr. Zeev Gross, Spring 2011. Page 144

“HIV is a virus which affects our immunologic system. It is different if you are the carrier as oppose to have symptoms when you are an AIDS patient. HIV is a virus. HIV attacks the immune system itself - the very thing that would normally get rid of a virus It takes around ten years on average for someone with HIV to develop AIDS ” Is that reasonable? Am I clear? Do you have any questions? In order to know whether you are infected or no we need to do a blood work. We need your consent for that. They will give you the results within two weeks. If the results are positive – they will call you back. If it negative – they will not call you. Options to send the sample: Nominal – with your name Non-nominal – put a bar-code (the public health and the doctor will know the identity) Anonymous – put a barcode on the sample and only you know the results (needs a lot of counselling. “How do you feel about it?” Sometimes patients are overwhelmed and might harm yourself or others” From now till the results of your blood tests: Practise Safe sex Do not donate blood Joint a support group.” I like you to know that in 2011 there are a lot of options open, with Rx it is controllable & people can live with it for a long time. If you test positive you’ve a have legal obligation to inform your partner.

Back to Content

Please note that this is only a draft version based on several sources, including: Dr. Basel Mohasen’s lectures, Toronto Notes, Therapeutic Choices and others. Edited and organized for the sake of all attendances of the Canadian Osce Exams: NAC OSCE and MCCQE2. by: Dr. Merlyn D’Souza and Dr. Zeev Gross, Spring 2011. Page 145

Freddy Mercury, 37y.o, Male, HIV results came back and are positive Divide time: 2min telling the results, 3min assessing symptoms, 2min explain about the virus, 3min the plan HIV treatment in Ontario is covered. “Nice to meet you. Or Hello” Because this is the first time I see you I am going to ask you some questions, to get a better understanding of your results: “Why/Who/Is the first time/When you did it?” ** Somebody I knew died from it last week “Who is the person that you got it from him?” (nature of the relationship) “People don’t get it from normal daily contact. Was there any direct contact?” SPIKE Setting Perception – what do you know about HIV? ―What did you think was going on with you when you felt the lump?‖ ―What have you been told about all this so far?‖ “Are you worried that this might be something serious?”

Invitation – how much details you want me to discuss? DO you want someone else to be present? ―Are you the kind of person who prefers to know all the details about what is going on?‖ ―How much information would you like me to give you about your diagnosis and treatment?‖ “Would you like me to give you details of what is going on or would you prefer that I just tell you about treatments I am proposing?”

Knowledge ―Unfortunately, I’ve got some bad news to tell you, Mr. Andrews.‖ “Mrs. Smith, I’m so sorry to have to tell you….”

Empathy – “What are your expectations from this visit?” If he is not very anxious you might take some time until giving him the results. Otherwise you give them immediately. “I wish I’d better news for you. Unfortunately the results came back and I am very sorry to tell you that the result is positive.” Silence. Wait x 10 sec if he cries “How do you feel right now?” Do you need more time/ Do you need water? Do you want me to proceed? **If mistake? “Whenever we do a screening test we confirm it if it positive. So the result is very accurate.” The initial test is ELISA & then we do a confirmatory test called Western Blot Part of F/u atre other tests like the CD4 count & Viral load SOCIAL Hx: We will ask you several questions concerning your sexual partners. Drug use Asx: Please note that this is only a draft version based on several sources, including: Dr. Basel Mohasen’s lectures, Toronto Notes, Therapeutic Choices and others. Edited and organized for the sake of all attendances of the Canadian Osce Exams: NAC OSCE and MCCQE2. by: Dr. Merlyn D’Souza and Dr. Zeev Gross, Spring 2011. Page 146

CSx: AIDS Sx: PMH: any long term disease? HTN/DM? Any hospitalizations/Surgeries? If does not want to inform his wife From experience it is not necessarily that your partner will leave you. From that reason we need to inform your wife. Part of the public health job is to tell her. The same measures we are taking for you we should do for her. It is better that she will know it from you rather than from the Public Health – otherwise she’ll loose the trust in you. Do you have any symptoms relating to HIV? PMHx and drugs. COUNSELLING If asked what he knows about HIV – don’g repeat. Otherwise you explain here. From HIV+ to AIDS. Significance of CD4 and Viral load: Viral load,amt of HIV virus existing in your body, lower the viral load,& higher the CD4 count, better condition We should think about HIV these days like a chronic disease as DM or HTn,it can be controlled but not cured. Part of your treatment is to refer you to HIV clinic – they will treat you based on these parameters. The newer medications are effective and control your disease – however they have side effects. How’s your mood, how you feel about that, there are a lot of support groups. I’ll give you “hot lines” number. From now on you have to practice safe sex & do not donate blood

In case of the resident who was asked to backup his supervisor orthopaed 1. I am competent – to emphasize 2. Short term – we don’t have time so we need to see her urgently 3. Long term – solve the situations that it wouldn’t occur again

Dr. Smith, Chief of staff of the hospital, ask another doctor to talk with the doctors me because they smell alcohol from the doctor. You smelled like alcohol. People have different ways to relieve their stress – how do you relieve your stress? Team worker – interpersonal relation It is better to provide prescription than getting pregnancy “I may share your point of view – that doesn’t say it gives me the right to impose my beliefs.” My concern is if we face the same situation in the future...what will we do? Please note that this is only a draft version based on several sources, including: Dr. Basel Mohasen’s lectures, Toronto Notes, Therapeutic Choices and others. Edited and organized for the sake of all attendances of the Canadian Osce Exams: NAC OSCE and MCCQE2. by: Dr. Merlyn D’Souza and Dr. Zeev Gross, Spring 2011. Page 147

Why won’t we contact the College? Don’t give any names. In case of report – it should be reported to the college. “We are here to help you. Moving to new place can be stressful. The reason of this meeting we have received two complaints – they claimed they have smelled alcohol from you. Is that happened? If you don’t mind me asking few more questions: do you drink more, or you did it on lunch time? Before working here – where else did you work? Did you ever have a complain about drinking? I would recommend that you will contact the program for doctors who drink. They will suspend your license. After stop drinking you will resume your work. At the end you will have your career back.

Back to Content

Please note that this is only a draft version based on several sources, including: Dr. Basel Mohasen’s lectures, Toronto Notes, Therapeutic Choices and others. Edited and organized for the sake of all attendances of the Canadian Osce Exams: NAC OSCE and MCCQE2. by: Dr. Merlyn D’Souza and Dr. Zeev Gross, Spring 2011. Page 148

24M: HA 6 wks ER 10 mins H & Counsel OCD PQRSTUV INC ICP:  SOL: Brain Tumors  Mets  Infection  Toxoplasmosis HA + Inc ICP Constitutional Symptoms: fever: Always ask what came first: fever or headache * If primary tumor; FH of Malignancy H/o Cancer,Leukemia,Melanoma H/o HIV:  Ask if HIV status known,  Have you ever been checked?  I’m concerned because of the risk factors involved

IF HIV status known ask:      

When was the last time you saw your Dr What was the last CD4 count Are U on any Anti AIDS meds? TB Skin test results Syphillis tst results Date & results of PAP’s smear * * * * * *

Ocurence f opportunistic infections,malignancies, STIs: Hepatitis B & C,Syphillis,Gonorrhoea,Chlamydia,Molluscum contagious Other bacterial infections,fungal infections,Malignancies Travel History,illness while away & use of preventative vaccines Medication History: Antiretroviral History (including response,CD4,Viral load) adherence,toxicity,any resistance testing & results

If HIV or AIDS & not on meds: Asses condt by asking: Constitutional symptoms:  Fever  Repeated chest infections  Cough thrush Please note that this is only a draft version based on several sources, including: Dr. Basel Mohasen’s lectures, Toronto Notes, Therapeutic Choices and others. Edited and organized for the sake of all attendances of the Canadian Osce Exams: NAC OSCE and MCCQE2. by: Dr. Merlyn D’Souza and Dr. Zeev Gross, Spring 2011. Page 149

  

Odonophagia TB In Female: Cx al Ca

OCD ONSET: COURSE: When Pt says HA now Ct ous Ask:  When did it become constant?  In beginning how often did you have it?  What time of the day?  Is it more in the morning? Or is it worse in the evening?  Does it wake you up? DURATION: PQRST UV S: How was it in the beginning as compared to ‘Now’ U Qns for empathy**    

How has it affected U in your daily life? How r U coping with it? How do U feel abt it? What r your expectations from today’s visit?

Aggravating fcts: Coughing/leaning forwards/lying down Alleviating fcts: ASOC Sx: CONstitonal Sx: Fever/nightsweats/chills (if before headache indicates patho) Ask when Wt loss started LOCal Sx: NEURO Screening: 1. Vision changes: What type of problem? 2. Hearing abnormalities 3. Difficulty swallowing 4. Weakness/Numbness 5. Difficulty finding words 6. Difficulty in balance or repeated falls Please note that this is only a draft version based on several sources, including: Dr. Basel Mohasen’s lectures, Toronto Notes, Therapeutic Choices and others. Edited and organized for the sake of all attendances of the Canadian Osce Exams: NAC OSCE and MCCQE2. by: Dr. Merlyn D’Souza and Dr. Zeev Gross, Spring 2011. Page 150

7. Changes n bowel/Urine Loss of bladder control 8. LOC 9. H/o Seizure MOOD Changes MEMORY problem Changes in CONCENTRATION Has anyone told you that you’ve been acting strangely? H/O INJURY: To head Did you have a fall & hurt your head? EXTRACRANIAL: EYE: Did you notice any redness or need eyeglasses? Sinusitis: Facial pain/flu Throath pain Dental pain

PMH: RISK FCTS: Do you smoke Take EtOh Take recreational drugs? Route Tattoo COUNSELLING: Do you have any qns for me? Based on what you’ve told me,the symptoms are concerning & I need to admit you today, as you’d stopped your meds, you may be exposed to an infection Your HA may be caused by this infection. I will refer you to an Infectious Disease Specialist Also do some blood investigations & Imaging of your head. DIAGNOSIS: CNS Toxoplasmosis in HIV +ve Male

Back to Content

Please note that this is only a draft version based on several sources, including: Dr. Basel Mohasen’s lectures, Toronto Notes, Therapeutic Choices and others. Edited and organized for the sake of all attendances of the Canadian Osce Exams: NAC OSCE and MCCQE2. by: Dr. Merlyn D’Souza and Dr. Zeev Gross, Spring 2011. Page 151

22F sudden loss of vision x 2 wks seen by 2 drs one opthal (Somatization GAO -207) As I understand you’re here because you’d loss of vision in (R) eye x 10 days, I understand you’ve been seen by 2 Drs    

What did the drs tell you? What diagnosis did they give you? Any investigations were done? How’re you doing today/Tell me how it all began...... OCD

ONSET: Sudden/gradual What were you doing at that time? Anything particular happened at that time? Is the loss all the time or off & On? Ask a little about local Sx: Pain Photophobia/Injury U V SOCIAL H:  Speak about confidentially here  Who lives with you?  What is your relationship?  Any difficulties in your relationship? Emphasize confidentiality  Was there a stressful situation before you lost your vision?

 Let us go back to that day........  Make sure no suicidal/Homicidal ideation COUNSELLING: This seems like a stressful situation for you & sometimes when we face such situations our brain finds it difficult to deal with it & this stress can be manifested by loss of function. In this case you saw something that made you lose your sight.It is not uncommon & called CONVERSION DISORDER I will refer you to a therapist to help you understand the stressor in your life & learn how to handle it.

Please note that this is only a draft version based on several sources, including: Dr. Basel Mohasen’s lectures, Toronto Notes, Therapeutic Choices and others. Edited and organized for the sake of all attendances of the Canadian Osce Exams: NAC OSCE and MCCQE2. by: Dr. Merlyn D’Souza and Dr. Zeev Gross, Spring 2011. Page 152

LOSS OF VISION: 32 M/F Pt presented with vision loss x 2 weeks OCD Onset after car accident Here enquire about the nature of accident Ask whether she or other driver was hurt & how is her driving now, does she still drive? Gradual C: Is it increasing now?/Same/Varies at certain times of day U &V CONSTITUNAT SX: Assoc Sx:

*HA: Here +ve,OCD: PQRST: When does it occur? Morning or evening Vomiting +/Quality---- projectile Weakness/Numbness/Difficulty finding words ENDOCRINE: Thyroid Disease: feel hot/cold/Warm & moist skin/tremors Pituitary:Sometimes Pts in similar situations can notice breast engorgement & secretions from breast (For Males) & changes in sexual life ,desire & habits In Females ask directly about amenorrhoea galactorrhea SX of Acromegaly: Inc size of shoes/Tight ring FH of Kidney stone/Pancreatic Ca/Diarrhea/Foul smelly stools Past H Fh Social H

Please note that this is only a draft version based on several sources, including: Dr. Basel Mohasen’s lectures, Toronto Notes, Therapeutic Choices and others. Edited and organized for the sake of all attendances of the Canadian Osce Exams: NAC OSCE and MCCQE2. by: Dr. Merlyn D’Souza and Dr. Zeev Gross, Spring 2011. Page 153

VISION LOSS 1. 40 M difficulty in vision 2 wks 10 mins H & C 2. 22F sudden loss of vision x 2 wks seen by 2 drs one opthal (Somatization GAO -207) 3. 40M diificulty in vision x 4 wks seen by optometrist counsel All D/vision: Screening Qn: What do you mean? Pt will answer: I’m not seeing well Now ask close ended qns: 1. One/(B) eyes 2. Blurry V 3. Double vision: a. Relieved by covering one eye? b. Horizontal/Vertical/Oblique c. Worse in one direction of gaze? d. Fluctating or constant? (Gets worse at end of day) 4. Loss of vision 5. Curtain falling 6. Dark spots/flashes 7. Difficulty seeing on sides/when you drive do you have difficulty changing lanes? 8. Do you bump into objects when walking? 9. Do you see halo around objects? OCD: O;Suden/gradual Painless/Painful PAIN: Assoc with:  Blinking  Eye movts  HA/N/V  Brow/Temporal pain  Photophobia

 Gritty sensation How has it affected your life

Has it happened before? Asso Sx: Fatigue wt loss,joint SxNight sweats,ever Polyuria/poly dipsia Please note that this is only a draft version based on several sources, including: Dr. Basel Mohasen’s lectures, Toronto Notes, Therapeutic Choices and others. Edited and organized for the sake of all attendances of the Canadian Osce Exams: NAC OSCE and MCCQE2. by: Dr. Merlyn D’Souza and Dr. Zeev Gross, Spring 2011. Page 154

Tingling /Numbness Past Occular H: Use of eyeglasses/Contacts: Duration H/O Occular surgery,Laser Rx,Infection,trauma,FB Presence of Ch eye disease: Glaucoma/DM PMH: Htn/DMMS/HIV Asthma Allergies Meds:Occular meds Current+Past

Back to Content

Please note that this is only a draft version based on several sources, including: Dr. Basel Mohasen’s lectures, Toronto Notes, Therapeutic Choices and others. Edited and organized for the sake of all attendances of the Canadian Osce Exams: NAC OSCE and MCCQE2. by: Dr. Merlyn D’Souza and Dr. Zeev Gross, Spring 2011. Page 155

Somatization Disorder 22 YOF Somatization disorder 4-2-1-1 If seen by a surgeon – suspect somatisation. What did the surgeon tell you. OCD PQRST During the day or night. CSx Jaundice, white stool and dark urine Foul smell, bulking, droplets Change in bowel movement First time to have this pain or had it before (V) MRI – why do you think it is important? Somatic pain disorder / Somatization Pains: headache, joints, back, pain with intercourse Sexual: You are here because you are concern. The pain you have, and multiple doctors – all these are consistent with somatisation. Once every 2-3weeks See Zu09: 207-8

Please note that this is only a draft version based on several sources, including: Dr. Basel Mohasen’s lectures, Toronto Notes, Therapeutic Choices and others. Edited and organized for the sake of all attendances of the Canadian Osce Exams: NAC OSCE and MCCQE2. by: Dr. Merlyn D’Souza and Dr. Zeev Gross, Spring 2011. Page 156

Introduction to OBGYN OCD COCA +/- Blood ΑA ASx:– which organism MSGO PMHx OBGYN MAP CC Menses Gynecology history Obstetriscs h Sexual h VAGINAL DISCHARGE. OCD COCA ΑA AsSx – which organism MSGO PMHx AMENORRHEA OCD MAGOS VAGINAL BLEEDING OCD COCA +/- Dc ΑA M ASx G O SHx PHx FHx INFERTILITY I+O Tr Partner C M G S Intro How many months trying to conceive? Please note that this is only a draft version based on several sources, including: Dr. Basel Mohasen’s lectures, Toronto Notes, Therapeutic Choices and others. Edited and organized for the sake of all attendances of the Canadian Osce Exams: NAC OSCE and MCCQE2. by: Dr. Merlyn D’Souza and Dr. Zeev Gross, Spring 2011. Page 157

If less than 35 y – wait for 1yr If around 40 – wait for 6m If greater than 40 – immediately O – if children from previous relationship Transition: ―In order for a couple to achieve pregnancy both partner involves should be relatively healthy and capable of having children. For that reason I need to ask questions about your and your partner’s health. Some of these questions can be personal, but important to ask, but I can assure that everything king is strictly confidential. The male factor is responsible for 40%.‖ COITAL H. How often do you have intercourse? Do you monitor tmp? MGOS All causes of secondary amenorrhea: Endometriosis Past medical history Family History Social history MENSES Use the word ―period‖ 1. When was your last period? 2. Are your periods regular / not 3. How often? 4. How many days? 5. How many pads do you use/change? 6. Are the pads full? 7. How long does it last? 8. Are they heavy? 9. Do you see clots? 10. Between periods do you have spotting? 11. From your last menstrual period was your period different from the current one? 12. At what age did you start your periods? 13. Were they regular/irregular? 14. When did it become regular? 15. Are your periods painful / painless? 16. If irregular, from beginning? 17. Discharge – ask if pregnant and when LMP GYENECOLOGY 1. Any history of Gyn. Disease – polyps or cysts 2. History of pelvic surgery (if yes – when?) 3. Have you used any birth control? When/type/any complications? 4. If less than 50 – have you ever done PAP (if yes – when and what were the results?) 5. If 50 and older – in addition ask for mammogram 6. If more than 65 – ask for bone density OBSTETRICS Have you ever been pregnant How many times, how about abortion and termination/ Abortion – termination medically; Please note that this is only a draft version based on several sources, including: Dr. Basel Mohasen’s lectures, Toronto Notes, Therapeutic Choices and others. Edited and organized for the sake of all attendances of the Canadian Osce Exams: NAC OSCE and MCCQE2. by: Dr. Merlyn D’Souza and Dr. Zeev Gross, Spring 2011. Page 158

How many live children, what was the route, any complication? Were there any complications with the children? During pregnancy: any HTM/GDM/Vaginal bleeding How do you feel about (miscarriage?) If NULLIPAROUS:FH of HTN/DM?cong anomalies/repeatd C S/Twins SEXUAL Hx: 1. With whom do you live? 2. How long have you been together? (a relationship below 6 month is not stable) 3. If you live alone – are you in relationship? 4. Are you sexually active? 5. Do you practice safe sex – using condoms? 6. When did you start to be sexually active? 7. How many partners you had last years? 8. What is your sexual preference? 9. What type of sexual activity do you practice? 10. Have you ever been diagnosed with PID 11. Any Vaginal discharge? 12. How about your partner? Does he have any symptoms have you ever been screned for HIV? PREGNANT IN T3: Reg F/U: No-----> Social Hx Yes:------->When If recent ask 2-3 qns about PET: 1. What was your BP 2. Was there swelling? MUM’s STABILITY: 1. Abd pains 2. Abd cramps 3. Vaginal bleeding or discharge 4. Any gush of water BABY: 1. Is baby kicking like before? 2. 10 movts/12 hrs

Back to Content

Please note that this is only a draft version based on several sources, including: Dr. Basel Mohasen’s lectures, Toronto Notes, Therapeutic Choices and others. Edited and organized for the sake of all attendances of the Canadian Osce Exams: NAC OSCE and MCCQE2. by: Dr. Merlyn D’Souza and Dr. Zeev Gross, Spring 2011. Page 159

19 year old with Vaginal discharge for the last 10 days History 5 minutes VAGINAL DISCHARGE. OCD COCA ΑA AsSx – which organism MSGO

PMHx: Recent use of Ab + DM Intro: How can I help you today? O: Sudden(Allergey)/Gradual How did it start? C: 1. Is it all the time or on and off? 2. Is it increasing, decreasing or the same? COCA + BL 1. Can you estimate the amount for me?Do you use pads? How many? 2. How about the colour? Is it greenish, whitish or yellowish? 3. How about the consistency? Is it thick or watery? 4. Is the smell offensive? 5. Is this your first time? A & A:Does it increase after IC? I would like to ask you a few personal questions, hope you don’t mind? Sexual Hx: as part of A & A 1. Are you sexually active? 2. Any relation to your periods or with intercourse? 3. Do you have any pain with intercourse? 4. Any itching, redness, blisters or ulcers? U: V: Associated symptoms: LOCAL Sx: Pain with IC Itching/redness/blisters/warts/ulcers GU: Any burning in urination in urination? GI: Any change in bowel movements? PID : Any abdominal pain? If yes, then OCD, PQRST. :MSK: Any ulcers in mouth, difficulty swallowing, joint pain, skin rash or red eyes? Constitutional Sx: Menstrual Hx: 1. When was your last menstrual period? Please note that this is only a draft version based on several sources, including: Dr. Basel Mohasen’s lectures, Toronto Notes, Therapeutic Choices and others. Edited and organized for the sake of all attendances of the Canadian Osce Exams: NAC OSCE and MCCQE2. by: Dr. Merlyn D’Souza and Dr. Zeev Gross, Spring 2011. Page 160

2. Are your periods regular? 3. Was the last period the same as before? Gynecological Hx: 1. Do you use any form of contraception? 2. Have you had a pap smear? When and what was the result? Obsteterics Hx: 1. Have you ever been pregnant? 2. Have you ever had an abortion or miscarriage?

Sexual Hx: As I understand you’re in a relationship… How long? Do you practise safe sex? Does the partner have any urinary symptoms,discharge? Before this? What age were you sexually active? How many partners in last one year? st

PMH:Since it is 1 time I’m seeing you,I need to ask some qns about PMH: SHx:

Please note that this is only a draft version based on several sources, including: Dr. Basel Mohasen’s lectures, Toronto Notes, Therapeutic Choices and others. Edited and organized for the sake of all attendances of the Canadian Osce Exams: NAC OSCE and MCCQE2. by: Dr. Merlyn D’Souza and Dr. Zeev Gross, Spring 2011. Page 161

RX:

Infectious Diseases: Sexually Transmitted Infections

Please note that this is only a draft version based on several sources, including: Dr. Basel Mohasen’s lectures, Toronto Notes, Therapeutic Choices and others. Edited and organized for the sake of all attendances of the Canadian Osce Exams: NAC OSCE and MCCQE2. by: Dr. Merlyn D’Souza and Dr. Zeev Gross, Spring 2011. Page 162

Table 1: Differential Diagnosis of Vaginal Discharge1 , 2 Candidiasis

Trichomoniasis

Bacterial Vaginosis

Pruritus

+

+



Odour



+

+ (fishy)

white, clumpy & curdy

off-white or yellow, frothy

grey or milky, thin, copious

+

+



< 4.5

> 4.5

> 4.5



+/-

+

budding yeast, psuedohyphae

motile trichomonads

clue cells,b predominant Gramnegative curved bacilli and coccobacilli

PMNs

++

+++



Lactobacilli

+





Signs/symptoms:

Discharge Inflammation Simple tests: pH "Whiff" testa

Microscopic findings: Specific

a. b.

Malodour often intensified after addition of 10% potassium hydroxide (KOH). Clue cells are vaginal epithelial cells covered with numerous coccobacilli.

Back to Content

Please note that this is only a draft version based on several sources, including: Dr. Basel Mohasen’s lectures, Toronto Notes, Therapeutic Choices and others. Edited and organized for the sake of all attendances of the Canadian Osce Exams: NAC OSCE and MCCQE2. by: Dr. Merlyn D’Souza and Dr. Zeev Gross, Spring 2011. Page 163

36 weeks pregnant with vaginal bleeding for 2 hrs History 5 min

Differentials: 1. 2. 3. 4. 5. 6.

When did the bleeding start? How long has it been? Were the pads fully soaked? Any clots? What were you doing at the time? Any H/o trauma? Did you have any abdominal pain? If yes:  Did the pain start first or the bleeding?  Are you having any pain now?  OCD, PQRST 7. Any gush of water? 8. Are you under regular F/U? 9. When was the last F/U ? If missed, why? 10. Symptoms of pre-eclampsia: a. Weight gain? b. Headache? c. High blood pressure? d. Flashing lights or disturbance in vision? e. Swelling? 11. Stability of Mother: .........................? Have you had an U/S? When was the last one? # of babies? Position of placenta? Amount of fluid? 12. Stability of fetus: Is your baby kicking like before? Obsteterics Hx: 1. Have you ever been pregnant? 2. Have you ever had an abortion or miscarriage? 3. Any complications during previous pregnancy?

Please note that this is only a draft version based on several sources, including: Dr. Basel Mohasen’s lectures, Toronto Notes, Therapeutic Choices and others. Edited and organized for the sake of all attendances of the Canadian Osce Exams: NAC OSCE and MCCQE2. by: Dr. Merlyn D’Souza and Dr. Zeev Gross, Spring 2011. Page 164

39 year old with vaginal bleeding for 50 days History 5 min OCD + COCA +/- Discharge AA ASx: M G O S Risk Fcts: GPOS D/d: PMH Fh SH INTRO: 1. What made you come here today? 2. Did you seek medical attention before? OCD: O: How did it start? What were you doing at the time? Did it start gradually or suddenly? C: Is it on and off or all the time? Is it increasing, decreasing or the same? COCA Can you estimate the amount for me? Foul smell? A&A: Is there any relation to periods or with intercourse? Menstrual Hx: 1. 2. 3. 4.

When was your last menstrual period? Are your periods regular? Was the last period the same as before? Can you differentiate this bleeding from periods?

Please note that this is only a draft version based on several sources, including: Dr. Basel Mohasen’s lectures, Toronto Notes, Therapeutic Choices and others. Edited and organized for the sake of all attendances of the Canadian Osce Exams: NAC OSCE and MCCQE2. by: Dr. Merlyn D’Souza and Dr. Zeev Gross, Spring 2011. Page 165

I’m going to ask a few qns to see how it has affected your life: IMPACT: Are you having any dizziness? Hrt racing? LOC?

Associated symptoms: CSx: Local & Mets 1. Local symptoms: Any itchiness, redness, discharge, pain during intercourse? Itching/rednes/blisters/warts 1. Any abdominal pain? When was your last Pap’s smear? What was result? O Sexual Hx: 1. With whom do you live? 2. Are you sexually active? D/D; Hypothyroidism Bl thinners Bleding disorders PMH FH of Ca

Back to Content

Please note that this is only a draft version based on several sources, including: Dr. Basel Mohasen’s lectures, Toronto Notes, Therapeutic Choices and others. Edited and organized for the sake of all attendances of the Canadian Osce Exams: NAC OSCE and MCCQE2. by: Dr. Merlyn D’Souza and Dr. Zeev Gross, Spring 2011. Page 166

AMENORRHOEA 22 yr old Female: PEP: 1. what is your Ds? 2. What is your inv? 3. What is your Rx? Always R/o pregnancy Intro: As I understand you’re here today because you did not have your periods since last 6 mo….. Can you tell me more about it? CC-------> When was your LMP? Any spotting in between? Let us talk about your periods from the beginning…….. st 1. When did you have your 1 period? 2. Was it regular from the start? Or was it irregular? 3. When did it become irregular? 4. When your periods were regular,How often did they come? 5. How long did each cycle last? 6. When irregular,How often did they come? How many days did they last? 7. When periods were regular, were you using any contraception? If Yes: How long? When did you stop?(if Inj Depo provera I yr post injection amenorrhoea OC can be upto 6 mo Amenorrhoea) With whom do you live? Are you sexually active? Any chance that you may be pregnant? Ask Sx of pregnancy:  Breast tenderness  N/Vx  Increased visits to washroom OB Hx: Any time you were pregnant? Any abortions or miscarriages?

RISK FCTS: I’m going to ask you questions to help reach what could be the cause HYPOTHALAMIC: 1. Are you under stress? 2. Are you losing wt? 3. How do you perceive yourself when you look into the mirror? 4. Do you exercise excessively? PITUITARY: 1. Any change in vision/Any difficulty in seeing on sides or changing lanes when driving? 2. Discharge from nipples & breast engorgement? THYROID: 1. Do you feel hot when others around feel cold or do you feel cold when others around feel cold? Please note that this is only a draft version based on several sources, including: Dr. Basel Mohasen’s lectures, Toronto Notes, Therapeutic Choices and others. Edited and organized for the sake of all attendances of the Canadian Osce Exams: NAC OSCE and MCCQE2. by: Dr. Merlyn D’Souza and Dr. Zeev Gross, Spring 2011. Page 167

2. Do you have constipation/Diarrhoea? OVARIAN: PCO: 1. Any acne 2. Increased facial hair? 3. Are you concerned about your weight? 4. Are you trying to lose weight? 5. Is there h/o DM? (ask for Sx of DM) 6. Any FH of PCOs or infertility? Premature Ov Failure: 1. Hx of Chemotherapy/Radiation to pelvis 2. Hot flushes 3. Night sweats Ovarian tumors: 1. Increased muscle bulk 2. Change in voice C Sx: Gyn Hx: Sexual Hx; Any H/o STis PMH: st Since it is 1 time I see you,do you have any H/o HTN,DM Have you ever seen a psychiatrist before or used antipsychotic medications/

Back to Content

Please note that this is only a draft version based on several sources, including: Dr. Basel Mohasen’s lectures, Toronto Notes, Therapeutic Choices and others. Edited and organized for the sake of all attendances of the Canadian Osce Exams: NAC OSCE and MCCQE2. by: Dr. Merlyn D’Souza and Dr. Zeev Gross, Spring 2011. Page 168

31 year old woman with 36 weeks pregnancy:BP155/110 Urine Protein +++ Intro: As I understand…..,you’re here today for a F/u visit,& nurse measured your BP & did a urine test.I’ve your results here & will discuss them with you, But I need to ask you some qns to gain a better insight into your condition 1. Were you ever diagnosed with increased blood pressure prior to this pregnancy? 2. When was your last F/U visit? 3. What was your Bp the last time? 4. What about your blood tests? 5. Were you anemic? Based on your BP & urine, these results are consistent with pregnancy induced HTN,& I need to ask you qns,to see if you’ve Sx pertaining to that. It could be a serious condition 1. Do you’ve H/O: HA ---------> OCD 2. How is your Vision ------------->Do you see flashes of light/Blurring 3. CNS --------> Weakness/numbness 4. Nx/V/Chest pain/SOB 5. ABD PAIN? 6. Bruises on body? 7. Yellow discoloration OF SKIN/ITCHINESS/Pale stool/Dark Urine 8. Swelling feet/Tight shoes/Rings tight 9. Difficulty opening eyes in morning/Inc wt gain 10. Vaginal bleeding/Discharge?  When was the last US: 1. How many babies 2. Is the baby kicking When is the due date? Have you been pregnant before? PMH FH of PET COUNSELLING: Do you like me to explain it to you? It is a very serious condition What do you know about it?

Preeclampsia is a condition of pregnancy marked by high blood pressure and excess protein in your urine after 20 weeks of pregnancy. Preeclampsia often causes only modest increases in blood pressure. Left untreated, however, preeclampsia can lead to serious — even fatal — complications for both you and your baby. Preeclampsia develops only during pregnancy. Risk factors include:

1. History of preeclampsia. A personal or family history of preeclampsia increases your risk of developing the condition.

2. First pregnancy. The risk of developing preeclampsia is highest during your first pregnancy or your first pregnancy with a new partner.

Please note that this is only a draft version based on several sources, including: Dr. Basel Mohasen’s lectures, Toronto Notes, Therapeutic Choices and others. Edited and organized for the sake of all attendances of the Canadian Osce Exams: NAC OSCE and MCCQE2. by: Dr. Merlyn D’Souza and Dr. Zeev Gross, Spring 2011. Page 169

3. Age. The risk of preeclampsia is higher for pregnant women younger than 20 and older than 40.

4. Obesity. The risk of preeclampsia is higher if you're obese. 5. Multiple pregnancy. Preeclampsia is more common in women who are carrying twins, triplets or other multiples.

6. Prolonged interval between pregnancies. This seems to increase the risk of preeclampsia. 7. Gestational diabetes. Women who develop gestational diabetes have a higher risk of developing preeclampsia as the pregnancy progresses.

8. History of certain conditions. Having certain conditions before you become pregnant —

such as chronic high blood pressure, migraine headaches, diabetes, kidney disease, rheumatoid arthritis or lupus — increases the risk of preeclampsia. Most women with preeclampsia deliver healthy babies. The more severe your preeclampsia and the earlier it occurs in your pregnancy, however, the greater the risks for you and your baby. Preeclampsia may require induced labor and delivery by Caesarian section. Complications of preeclampsia may include: Lack of blood flow to the placenta. Preeclampsia affects the arteries carrying blood to the placenta. If the placenta doesn't get enough blood, your baby may receive less oxygen and fewer nutrients. This can lead to slow growth, low birth weight, preterm birth and breathing difficulties for your baby. 1. Placental abruption. Preeclampsia increases your risk of placental abruption, in which the placenta separates from the inner wall of your uterus before delivery. Severe abruption can cause heavy bleeding, which can be life-threatening for both you and your baby. 2. HELLP syndrome. HELLP — which stands for hemolysis (the destruction of red blood cells), elevated liver enzymes and low platelet count — syndrome can rapidly become life-threatening for both you and your baby. Symptoms of HELLP syndrome include nausea and vomiting, headache, and upper right abdominal pain. HELLP syndrome is particularly dangerous because it can occur before signs or symptoms of preeclampsia appear. 3. Eclampsia. When preeclampsia isn't controlled, eclampsia — which is essentially preeclampsia plus seizures — can develop. Symptoms of eclampsia include upper right abdominal pain, severe headache, vision problems and change in mental status, such as decreased alertness. Eclampsia can permanently damage your vital organs, including your brain, liver and kidneys. Left untreated, eclampsia can cause coma, brain damage and death for both you and your baby. 4. Cardiovascular disease. Having preeclampsia may increase your risk of future cardiovascular disease. Admit you …….. Stabilize you MgSo4 IV Labetolol May consider Steroids for babe

Back to Content

Please note that this is only a draft version based on several sources, including: Dr. Basel Mohasen’s lectures, Toronto Notes, Therapeutic Choices and others. Edited and organized for the sake of all attendances of the Canadian Osce Exams: NAC OSCE and MCCQE2. by: Dr. Merlyn D’Souza and Dr. Zeev Gross, Spring 2011. Page 170

REQ FOR CS 34 wks pregnant primi requesting for CS Intro: As I understand you’re 34 weeks pregnant & arte requesting a Cs. Before I proceed any further,I need to ask qns to reach the best plan. WHY? Pt: I believe it is painfull  What makes you believe it is painful?  Have you’d a prior experience? Pt: My sister had a NVD Never imply that sister did not receive best care There are 2 options: There are a lot of options to control pain nowadays & different people have a different pain threshold   

Are you on regular F/u? Is it a planned pregnancy? When was the last time you saw a doctor?

if not on reg F/u as she is alone BF left etc always Empathize,Ask how she is coping with him leaving & how she is handling the stress How do you support yourself financially? I can see that this is a very difficult period & I want you to know that there are a lot of help & resources available in the community. I will make sure you’re connected to a social worker who will help you support you & your child & will help you to start your life If on reg F/U  When was the last time you’d your BP measured?  Sx of PE: Any HA/Nx/V Blurry vision/Abd pain...... MUM’S STABILITY: Abd pain Contractions Vaginal bleeding Vaginal discharge BABY: Is baby kicking like before DUE DATE OBG Hx: Have you been pregnant before? How many times How about abortions/Mc If +  At how many weeks?  What reason  When  Any complications  How did you feel about that? PMH: Risk fcts NEXT come to PAIN CONTROL: As I understand you want a CS,however if pain is a major concern, there are several options: You can attend antenatal classes that will teach you to breathe, meditate During your delivery a person will accompany you to give you support & emotionally support you If that does not work, there is another very popular method of delivery: EPIDURAL Anaesthesia ... explain....

Please note that this is only a draft version based on several sources, including: Dr. Basel Mohasen’s lectures, Toronto Notes, Therapeutic Choices and others. Edited and organized for the sake of all attendances of the Canadian Osce Exams: NAC OSCE and MCCQE2. by: Dr. Merlyn D’Souza and Dr. Zeev Gross, Spring 2011. Page 171

An epidural block is a common type of anesthesia for labor and delivery. During labor, a needle is placed in the epidural space, which is just outside the spinal canal. A small, hollow tube called a catheter is inserted through this needle. Once the catheter is in place, the needle is removed and medication is injected through the catheter to numb your lower abdomen and birth canal. It may take 10 to 20 minutes to feel pain relief from an epidural block. As labor continues, the medication can be adjusted to help keep you comfortable. Painless & effective SE: rarely it may causeHa/Dizziness/infection & may prolong labour If still does not accept:

What is your understanding about CS? It is a major surgery which is effective & lifesaving. However if there is no real indication,NVD is preferred as it is natural With CS there is a scar Longer recovery More bleeding Higher risk of infection Why don’t you think about it Give brochures Refer to Obstetrician for 2nd opinion F/U in 2 weeks

Back to Content

Please note that this is only a draft version based on several sources, including: Dr. Basel Mohasen’s lectures, Toronto Notes, Therapeutic Choices and others. Edited and organized for the sake of all attendances of the Canadian Osce Exams: NAC OSCE and MCCQE2. by: Dr. Merlyn D’Souza and Dr. Zeev Gross, Spring 2011. Page 172

32F with 34 weeks pregnancy in hosp clinic,3yrs ago had an urgent CS due to cord prolapsed,needs her file,counsel her See if request is logical Ask type of Cs Why not happy with last Cs May be bad experience pain/bleeding/Complications Maybe dead baby Was it 1st Cs or 2nd Was it CLASSICAL Cs? Then always Cs Risk of rupture of Classical Cs ------12% of which 10% will die LSCS risk of rupture is 1% INTRO: As I understand you’re here cuz you want your file & based on your report you’d a hospital delivery because of cord prolapsed & it was an urgent Cs. Why? PT: Delivery by midwife who wants to look at it We will give you the file, but until then I want to discuss Pt: in a hurry Because you’ve had a previous Cs & you want a midwife. In order to make a proper decision you’ve some imp info to know What happened the last time? When did you know? How many week s were you? How did you feel? What was done? Did they explain it to you? Was there any bleeding/Infection How was the recovery period/ How is the baby/ Is it a boy/Girl How old? Is the baby healthy? If baby was fine & no complications: Looks like it was a right decision & the outcome was good What is your understanding about cord prolapsed? Cord is squeezed between head of baby & pelvic bones.It is a life threatening condition & needs urgent intervention Have you ever been pregnant other times? How are you doing in this pregnancy? What was your last F/u Bp? US Baby kicking PMH Due Date Waht is your understanding of Cs? There are different methods: Classical Section Vertical incision Lowere Segment C Section ----->transverse incision Most common is the transverse sectionThe cut is parallel to the fibres & thus it is a strong scar If you go into labour there is a lot of pressure & tension on the scar & with continuous pressure there can be rupture of scar this is concerning

Please note that this is only a draft version based on several sources, including: Dr. Basel Mohasen’s lectures, Toronto Notes, Therapeutic Choices and others. Edited and organized for the sake of all attendances of the Canadian Osce Exams: NAC OSCE and MCCQE2. by: Dr. Merlyn D’Souza and Dr. Zeev Gross, Spring 2011. Page 173

There will be a lot of bleeding we might not be able to help you & the mechanism of delivery will stop Chances of rupture in cl S is 12% of which 105 will die However if you want to continue the decision is yours Your life & the baby’s are endangered With a transverse Cs We can give you chance of normal delivery in hospital,as in case we need to do an urgent Cs we can If Not ConVinced: Why don’t you go back to your midwife & talk to her & mention She is trained & qualified We share the same guidelines We can arrange 2nd opinion F/U

Back to Content

Please note that this is only a draft version based on several sources, including: Dr. Basel Mohasen’s lectures, Toronto Notes, Therapeutic Choices and others. Edited and organized for the sake of all attendances of the Canadian Osce Exams: NAC OSCE and MCCQE2. by: Dr. Merlyn D’Souza and Dr. Zeev Gross, Spring 2011. Page 174

54 year old female comes to clinic concerning about using HRT. . When a patient has concern about any subject, address it very soon. Don't wait to the end. Dr: As far as I understand you're here as you have concern about using HRT. Patient: yes Dr. I feel I am confused about using HRT. Always ask what do you mean by HRT. So the patient will tell you how much they known about HRT. Dr: I'm glad you're here so we can discuss about it and address your concerns and hopefully by the end of the session you can make a decision regarding using HRT. Or hopefully by doing this discussion you will have a better understanding of HRT. Or you can say: I agree with you as there are a lot of confusion about HRT and the reason for this confusion is that in the past because it was used to be given routinely to all women who reach a certain age, however 10 years ago there was study called " women health initiative" in which the authors found that the numbers of the patients with serious side effects are very high. However those ladies used HRT for a long time.

Serious side effects are Cancer, Heart attacks and Strokes. For that reason the routine use of HRT was stopped. Nowadays we have a better understanding and have better guidelines. Not only that we do it on the individualized basis. We use it only for short time, they don't exceed five years. So using HRT within five years is safe. So I would take some information from you and we will discuss about the risk factors and if you are a good candidates we can make a decision to prescribe it or not. Dr: What makes you interested in HRT? Patient: because of hot flushes.

At this stage if the patient give you the symptom, it is your chief complaint. But if patient doesn't give you any symptoms, you should start with her LMP If she starts with the symptom of hot flushes, ask the patient 1. When did hot flushes start, 2. Is it all the time, 3. On & off or continues, 4. How many attacks, 5. Day or night, 6. How do you feel that you have it. 7. Night episodes, you have any night sweating, does it wake you up. Asked patient if the hot flushes wake her up during the night, and if she needs to change her gown of nights sweats. 1. Affect your sleep and how does it affect your concentration. 2. Change in your mood, anybody has told you that your short tempered, and if you 3. feel tired. 4. Some women with the same symptoms may notice some change in their sexual life. d) So the doctor should ask with whom do you live? e) Are you sexually active? f) Any dryness or pain during the intercourse? 8. Any change in your urination? Have you ever lost control? Please note that this is only a draft version based on several sources, including: Dr. Basel Mohasen’s lectures, Toronto Notes, Therapeutic Choices and others. Edited and organized for the sake of all attendances of the Canadian Osce Exams: NAC OSCE and MCCQE2. by: Dr. Merlyn D’Souza and Dr. Zeev Gross, Spring 2011. Page 175

9. Last period?  Are you periods regular or not?  If it's irregular, when did it start to become irregular?  Are your periods heavy or not?  Any clots?  *Any bleeding or spotting between periods? This is a very important point. 10. Bone pain? Any fractures? Any family history of osteoporosis? If yes, tell the patient that you will discuss this in another meeting. Because that's another session to discuss about using steroids, smoking, alcohol, caffeine, warfarin and diet. If she takes calcium supplements. MGOS for GYN cases: Menstural, Gynecologic diseases, Obstetrics, Sextually transmitted disease Dr: any history of gynecological disease like polyps, cysts, any pelvic intervention/instrumentation, surgeries. Dr: did you use to take any oral contraception? If yes, which one and did you have any side effects? Also you should ask about her last smear. Because she is 50+ you should ask about her mammogram. At any age you ask about Pap smear, once you reach 50 to ask about mammogram and when the patient pass 65 you should add bone density. You can ask about her obstetrics history, like have you ever been pregnant if yes how many times you have been pregnant? Now use the transition... Because this is the first time I met you, I would like to ask you about your past medical and social history. Is there any long-term disease, hospitalization before, any surgery, diabetes, or hypertension. Any history of allergy, and the medication she takes. ABCD: Active liver disease, vaginal Bleeding, Cancer, DVT For A you ask about any history of Active liver disease. Have you ever been yellowish? Any dark urine or pay stool? For B you should ask about any vaginal Bleeding? ... You have already asked these question before For C you should check about Cancer. I would like to ask about constitutional symptoms here to see if there is any endometrial cancer. Fever, chills, weight loss, appetite, lumps & bumps. A history of cancer in yourself or family (breast cancer, endometrial cancer,and colon cancer). For D you should ask about any history of swelling in the legs (DVT), any history of heart attacks, pulmonary embolism or stroke. Social history: smoking, taking alcohol, recreational drugs, how does she support financially herself, how does this affect her life and ask about osteoporosis. Usually in this set of scenario, you tell her on the basis of the history you are good candidates for HRT. However as I told you it is an important information to tell you to make your decision. As we go through different stages of life usually for ladies, we go to the stage called menopause which is vary between person to person. At this stage there is hormonal changes and ovaries start to produce less hormones specialty estrogen and progesterone and that changes affect the whole body. It can explain about dryness, decreasing or absence of periods. And that's why we try to replace those decreased hormones by HRT. They are the same hormones but we give it through external sources either tablets or skin patches. As I told you before there is a balance it's your decision to make. And the balance is to use it up to five years. Using more than five years would increase the risk of stroke, heart attack or some Please note that this is only a draft version based on several sources, including: Dr. Basel Mohasen’s lectures, Toronto Notes, Therapeutic Choices and others. Edited and organized for the sake of all attendances of the Canadian Osce Exams: NAC OSCE and MCCQE2. by: Dr. Merlyn D’Souza and Dr. Zeev Gross, Spring 2011. Page 176

cancers depending on what we call it estrogen dependent that includes breast and endometrial cancer. And some studies showed that it might increase the risk of Alzheimer's disease. So the risk of use for less than five years is not significant and still acceptable. So if you want to use it the shorter the better. To get rid off the hot flushes there are other measures like exercise or herbal supplements that you can try to improve the symptoms. The HRTs are the same as OCP's but in smaller doses and you can take one tablet a day. They have a few side effects like weight gain, bloating, nausea, abdominal distention and pain but they improve by time. The serious side effects are headaches, swelling of the legs or chest tightness which whenever happen you should go to emergency room. By using these HRT's your periods may stop or you may see spottings. If the patient had hysterectomy before you only give estrogen without progesterone, otherwise you should give both. Because you take it regular shootout regular ultrasound scans to check the thickness of the endometrium and sometimes we should take a sample

Back to Content

Please note that this is only a draft version based on several sources, including: Dr. Basel Mohasen’s lectures, Toronto Notes, Therapeutic Choices and others. Edited and organized for the sake of all attendances of the Canadian Osce Exams: NAC OSCE and MCCQE2. by: Dr. Merlyn D’Souza and Dr. Zeev Gross, Spring 2011. Page 177

INFERTILITY (Sometimes it is not easy/Sometimes it takes time/I’m glad you’re here) Intro: As I understand you’re here because you’ve been to get pregnant for the last 14 mo, during the next few minutes tell me more about this difficulty. Did you seek medical attention before? (40 ASAP) PID & other med condts: ASAP How long have you been in this Relationship? How long have you tried? Have you ever been pregnant before? Have you ever had Mc or Abortion? Spouse: has he had children from a previous relationship? Let us talk about your Partner: (If less time Fast otherwise get details) Fast: Was he ever investigated? Did he have Semen analysis? What was his sperm count? Detailed: How is his health Does he have (Htn?DM/On meds) Any back trauma,back pain? Any Surgeries Any H/o mumps in childhood? H/O Ca, Rxt Cxt,STIs? Any Psy meds,Stress,travel a lot? Exposed to heat at work or recreational way? COITAL Hx Some qns about intimacy: How often do you have IC with husband? How do you monitor your temp? How do you measure your urine test? Is your husband capable of having an erection & ejaculation? Do you use any lubrication? MENSTURAL When was your LMP? GYN: SEXUAL: Any STIs

Back to Content

Please note that this is only a draft version based on several sources, including: Dr. Basel Mohasen’s lectures, Toronto Notes, Therapeutic Choices and others. Edited and organized for the sake of all attendances of the Canadian Osce Exams: NAC OSCE and MCCQE2. by: Dr. Merlyn D’Souza and Dr. Zeev Gross, Spring 2011. Page 178

16 YOF information about Pap Smear, counselling, health maintaining issues (comes with HEADDSSS). Whenever there is counselling – take history. . Pap smear – What do you like to know about Pap smear? Usually we offer it for people who are sexually active, for that reason I’d like to know if you are in a relationship? Are you sexually active? When did you start? Any other relationships or partners prior? Do you use protection? What oprotection do you use? Any STD (blisters, ulcers, warts) in the last 6 mo? MGOS: M:LMP How often do you get your menses? Are they regular? Are your periods painful? Are they heavy? G – any gynaecologic disease? Any pelvic exam? O: Any H/o pregnancies/Abortions? Past medical Hx? HEADDSS .... Counsel about Seat belts Mood & Risks of suicide? Counselling: Why? & How? I am glad you came here today to talk about Pap smear. As a matter of fact PS is one of the most successful screening tests to pick up one type of dangerous cancer called “Ca Cx” which is caused by a virus called Human Papilloma virus & a condom does not protect you from this. It is important to pick it up early, since by time it starts to give symptoms it is too late. Let me explain it to you. The area connecting the vagina to womb is called Cervix, & from the outside it looks like this:(Draw the circle with a dot) from this part we’ve to get a sample it gets infected with HPV virus which is similar to wart virus, but in the cervix, it leads to cancer. It should be done a week after your menses. It has to be done in a certain way, there will be a nurse with me,& you will be on your back, the exam bed has pedals to support your feet. We will use a speculum which come in different sizes & are plastic & disposable & we use a water based lubricant. If spatula rotate it to 360 & put on a slide, fix it & send to lab. If brush, rotate it 5 times put in fluid & send to lab. Results will be back in 2 weeks. If all is well, we will not contact you. PS has to be done every year .Once results are normal for 3 yrs & you’re with same partner you casn do it every 2 years till 69 years old, when you can stop ,If you change your partner, you’ve to do it yearly again. Other hazards – drinking and driving.

Back to Content YOUNG WOMAN: ANTENATAL COUNSELLINg Please note that this is only a draft version based on several sources, including: Dr. Basel Mohasen’s lectures, Toronto Notes, Therapeutic Choices and others. Edited and organized for the sake of all attendances of the Canadian Osce Exams: NAC OSCE and MCCQE2. by: Dr. Merlyn D’Souza and Dr. Zeev Gross, Spring 2011. Page 179

History Ask how Pt feels about being pregnant When did you do the test? How did you find out? Congratulate if she is happy. It is a very exciting time of your life. Ask questions about the pregnancy LMP: LLMP was it similar to prev menses or less bleeding? Calculate EDD; - 3mo + 7 days Sx:, N/V/Breast engorgement/Inc visits to washroom Rh status If nausea severe, ask about dizziness O: GTPAL Any complications in previous pregnancies: HTN/DM/Twins/Congenital anomalies G: (surgeries, infections, PAP’s) S: Any STIs PMH: Vaccinations, diabetes, hypertension, heartdisease, genetic diseases, kidney diseases, immunological diseases Past history of surgery-especially childhood Family Hx: Genetic disease, prematurity, early onset deafness SHx:Medications, Smoking, Alcohol, Recreational drugs OTC if on Aspirin ask to stop & change to Tylenol Social Hx: partner, support, provisions for child With whom do you live? How does your partner feel about this pregnancy? Do you feel safe in this relationship? Do you have pets? If has cat,not to change litter. COUNSELLING: I will confirm pregnancy by blood work

Physical Vitals, weight, full exam including PAP smear (if not done in last 6 mo)and cultures

Investigations CBC, Lytes, INR/PTT, Urea, Creatinine, Blood Type, VDRL, Rubella antibody, Serum folate, Hepatitis, +/- HIV, Urine dip and microscopy, ECG if indicated, +/- sickle cell and thalessemia screens. Nuchal Translucency at 12 weeks Maternal serum screen at 16 weeks Anatomy ultrasound at 18-20 weeks Glucose challenge test at 24 weeks +/- Rhogam at 28 weeks Diet, smoking, alcohol, exercise, medications, morning sickness Average weight gain is 25-35 lbs with 5-10ibs up to 20 weeks and then 1lb/week thereafter Risks of Down’s 1/200 at 35 Consult MD prior to meds For morning sickness eat bland foods, small portions, Diclectin is an option Hemorrhoids, back pain, heartburn and increased vaginal discharge are common Visits are every 4 weeks until 28 weeks then every 2 weeks

Back to Content 19/2/2011 Please note that this is only a draft version based on several sources, including: Dr. Basel Mohasen’s lectures, Toronto Notes, Therapeutic Choices and others. Edited and organized for the sake of all attendances of the Canadian Osce Exams: NAC OSCE and MCCQE2. by: Dr. Merlyn D’Souza and Dr. Zeev Gross, Spring 2011. Page 180

Introduction What to write on the note before entering the room:  Name  Age  CC  What required  DDx First buzz – turn and read the stem (2m) Second buzz – knock the door and enter. The examiner might tell you that at the end he will ask you 1 or 2 questions. In the next buzz – it will be the time for the questions. If he doesn’t ask – complete the task. After another minute there will be a longer bip sound. Short station (history, physical exam, s/e of psychiatric medications): 1.5 m buzz – enter Short buzz after 4.5 m (do “Thank you for the information, I’ll do the physical exam and I’ll take it from there.” CC “As I understand you have ...for ... can you tell me more from the moment you started to notice it.” “I am glad you took the time to come here In case of physical exam: Intro: name, position, why you are here, time that I am going to spend with you and for which purpose. “Good morning, my name is...I am the attending physician here. I understand you are here because....In the following 5 minutes I will perform physical exam, hopefully towards the end we will reach a workable plan. If you feel any discomfort please inform me. Do you have any questions?” Imaging, blood work “Hello, good afternoon Mr. ... as I understand you are here to get your blood work results, since this is the first time I see you before I am discussing the results with you I have some questions to discuss with you.” Than: Why, 1st time, who and when. In case of breaking bad news: SPIKE Setting Perception (What is your understanding about the test, and why you are doing that” Invitation (How much details do you like to discuss with you? DO you like anyone to be with you?) Knowledge (What do you know about the condition?) Expectations (What are your expectations from today’s visit? What is your expectation from the result?) Gives the result. Questions for telephone session: What’s your number? Where do you live? “...did I say it right?” Please note that this is only a draft version based on several sources, including: Dr. Basel Mohasen’s lectures, Toronto Notes, Therapeutic Choices and others. Edited and organized for the sake of all attendances of the Canadian Osce Exams: NAC OSCE and MCCQE2. by: Dr. Merlyn D’Souza and Dr. Zeev Gross, Spring 2011. Page 181

Connecting with a colleague:

Psychiatry Psychosis 1. 55 yo, believe that have strange feeling in hands. Do mental exam. Either organic, late onset of schizophrenia, not complying with medication. 2. 35 yo, believes that the RCMP chasing him. Persecutory delusions. Reassurance about his safety. DDx substance abuse. 3. 24 yo, brought by his roommate because haven’t been himself in the last 10days. Can be acute psychosis, substance abuse, HIV, mania 4. 30 yo, wants to arrange DNA test for his children. 5. 17 yom, worried about contamination – wants to be admitted to get rid from it. 10min – councsel. 6. 22yo, diagnosed with schizophrenia 6wk ago, concerned about his condition. Think about suicide! 7. 17 yo male, pain in his neck. s/e of drugs. 8. 35 yo, brought by the police because he wanted to slaughter his children (thinks he his Abraham). Ask him “Who is Abraham?” Ask early about: “How is your mood today?” – To differentiate from mania. Mood (Presenting symtoms) MI PASS ECG 1. Low mood for the last 6w. 2. Patient with difficulty to sleep: 2.1. 22 yof 2.2. 35 yof 2.3. 75 yof asking for sleeping pills 3. Suicide case. 4. Presentation with tiredness 34 yo. 5. 40 yom hasn’t been himself for the last 3w – his wife concerned. 6. 70 yo has back pain for 3w (x2 cases). 7. Dysthemia case. A young lady with low mood for years DIG FAST 1. Impulsive behaviour – might be presented with intoxication to the ER. Sexual activity with no protection. Issues with the law (fighting in the bar, waking up the neighbours). 2. Grandiosity – some delusional ideas. 3. Patient who wants to discontinue the medication.

Anxiety 1. Panic attack – heart racing, sob, dizziness, tingling, numbness (hyperventilation – hypocapnea) STUDENTS FEAR 3C’s Please note that this is only a draft version based on several sources, including: Dr. Basel Mohasen’s lectures, Toronto Notes, Therapeutic Choices and others. Edited and organized for the sake of all attendances of the Canadian Osce Exams: NAC OSCE and MCCQE2. by: Dr. Merlyn D’Souza and Dr. Zeev Gross, Spring 2011. Page 182

2. Patient already diagnosed recently with PA or Panic disorder or generalized anxiety – discuss the treatment. Delirium and Dementia – Cognition disorders 1. 57 yom difficulty with her memory. History and mental status exam (mini mental). 5min. 2. 67 yof difficulty with her memory. Score for mini-mental 20. 3. 67 yom came with his wife, concerned about his memory for the last 3m. Next 15min talk with him. 4. 70 yom, s/p hip replacement 3d ago. Didn’t sleep last night (reversed sleep cycle) – delirium. Fragmented sleep cycle – dementia. 5. His dad is not being himself. You talk with the son. You cannot do mini-mental to the son. 6. Talk to the son about his mom that is in senior home. He is concerned – she was given 15u instead of 5u of Insulin. “It looks like there is some kind of medical error.” Eating disorder, borderline, schizotypal, conversion 1. 16yof, the parents concern that she loses weight. Part of the DDx is figure out that she has amenorrhea. 2. 22 yof wants to be admitted. She wants to kill herself. If is the first time – you need to admit her. If it is several times – it’s not necessary to admit her. You have to finish the assessment. If she lives the room before finish the interview – you will write form #1. Usually people with psychiatry problem have: social worker and case manager. Have you ever seen by psychiatry. 3. Schizotypal disorder 4. Sudden loss of function. Seen by two doctors, one of them specialist in that field – it means it is conversion. 4.1. Loss of vision in her rt eye. Seen by ophthalm. 4.2. 22 abdominal pain for 3w, seen by a surgeon a week ago. Counsel. 4.3. Headache for the last 6 mo, she wants to renew her thylanol 3 (x2) 5. Alcholism 5.1. AST>ALT, GGT elevated 5.2. His wife concerned he is not himself for the last 3m 6. Suicide (SAD PERSONS – score more than 4 you have to admit). 6.1. Overdose of aspirin. Medically clear.

Back to Content

Please note that this is only a draft version based on several sources, including: Dr. Basel Mohasen’s lectures, Toronto Notes, Therapeutic Choices and others. Edited and organized for the sake of all attendances of the Canadian Osce Exams: NAC OSCE and MCCQE2. by: Dr. Merlyn D’Souza and Dr. Zeev Gross, Spring 2011. Page 183

Psychiatry Assesment In PSY Ds look for: I. II.

TIME CRITERIA

If CC is psychiatric, make an early decision in MOAPS format, where:

M=Mood Depression --MI PASS ECG 





 

  

Mood How is your mood? Do you feel down? Do you cry a lot? Have you felt that before? “You look down for me – is there any chance you are depressed?” Is your mood always down or does it alternate? Have you been very happy at times? if YES: enquire about Mania INTEREST: Have you lost interest in activities in doing activities that were enjoyable to you? “Anything makes you happy?” If he doesn’t it any more – “Why?” (Doesn’t have time, no energy, or doesn’t enjoy it) PSYCHOMOTOR RETARDATION/AGITATION: “DO you feel things are getting slower? Do you need more time to do things you did before?” APPETITE “Did you lose weight deliberately?” SUICIDAL Ideation “Any plan?” “Did you live a note?” “Did you start to give your belongings to others?” SLEEP “When you go to sleep? When wake up? Do you feel fresh?” ENERGY “Do you feel tired?” CONCENTRATION “When you read an article can you finish it to the end?” “Do you find to focus to concentrate in one subject?”

Please note that this is only a draft version based on several sources, including: Dr. Basel Mohasen’s lectures, Toronto Notes, Therapeutic Choices and others. Edited and organized for the sake of all attendances of the Canadian Osce Exams: NAC OSCE and MCCQE2. by: Dr. Merlyn D’Souza and Dr. Zeev Gross, Spring 2011. Page 184



GUILTY “Do you feel guilty?” “Do you feel there is no hope in life?” After getting two depression episodes. If they are at least two month apart – Major depressive For teen age istead of mood and interest is replaced by irritability and droped in school performance. In elder person you might have need somatic disorders.

Bipolar I (Mania) - DIG FAST (elevated mood + at least three out of the seven for a week) sometimes it is irritated mood – than you need 4 out of seven for a week. Usually they don’t last a week – so if they end up in hospital look for the criteria even for less than a week.  DISTRACTATIBILITY: “DO you find difficult to focus on one subject?” “Are you working on more than one project at the same time?” “How many projects do you work in?” – “Are you able to finish it or not?”  IMPULSIVITY “Are you spending more time than before?” “Are you borrowing money from other people?” “For what reason?” “Are you drinking more than before? Do you use cocaine? Which happen first? – elevation of mood or using cocaine?” “With whom do you live? Are you sexually active? How many partners do you have? Do you practice safe sex?” “Do you have any problems with the law? Speeding tickets? Any fights?  GRANDIOSITY: “Do you believe you’re a special person?” “Do you believe you deserve to be treated in a special manner?” “Do you feel you’ve a special power?” “Do you feel you’ve a special mission?--- if Yes Always ask what is the mission? & probe deeper & inquire about Delusions*  FLIGHT OF IDEAS: Do you feel thoughts racing in your head? Do people say you’re jumping from topic to topic  GOAL DIRECTED ACTIVITY: “How much time you spend in your activity?”  SLEEP  TALKATIVE “Anybody mentioned that you are talking faster or more than others?” Ask:If first episode or has it occurred before? Please note that this is only a draft version based on several sources, including: Dr. Basel Mohasen’s lectures, Toronto Notes, Therapeutic Choices and others. Edited and organized for the sake of all attendances of the Canadian Osce Exams: NAC OSCE and MCCQE2. by: Dr. Merlyn D’Souza and Dr. Zeev Gross, Spring 2011. Page 185

Also look for OPPOSITE mood

Relapse rate for the first time: 60% next time it is 80% third time 95%. Intro Why? Concern Assess mood today How you were diagnose with bipolar I? When? Why? Were there any serious consequences? Regular follow up? When you saw last your doctor? What was the level of Lithium that time? How do you feel about Lithium? Did you notice any s/e? Have you ever forget to take the drug? (It will be easier in the counselling). “I know that you have been this question before but I am going to ask you again – do you hear any voices. Do you worry a lot...” Counselling Compare mania to depression. What is your understanding of mania. It is a condition...

O=ORGANIC (I MAD): 

ENDOGENOUS (ILLNESS); Depression: Hypothyroid/Lupus/Ca Pancreas/Post MI/CVA  EXOGENOUS: (Substances: MAD) M: Medications: Dosages/duration/SE/Toxicity A: Alcohol: How much/day? D: Drugs: 1. What drugs have you tried? 2. When 3. How much 4. Any Hx of O/D,W/d,SE,hospitalizations? 5. Which drugs NOW?

A=ANXIETY SCREEN: Do you worry a lot?

Interview Questions to Establish Specific Anxiety Diagnosis Please note that this is only a draft version based on several sources, including: Dr. Basel Mohasen’s lectures, Toronto Notes, Therapeutic Choices and others. Edited and organized for the sake of all attendances of the Canadian Osce Exams: NAC OSCE and MCCQE2. by: Dr. Merlyn D’Souza and Dr. Zeev Gross, Spring 2011. Page 186

Questions Further Inquiry 1. Do you have sudden episodes of intense anxiety? Establish nature of attack

*

2. Do you have difficulty going to places to Inquire about crowded places, line-ups, 3. 4. 5. 6.

7. 8.

movies, highways, distance from home. which you used to be able to go? Do you have difficulty talking to people Establish situations (one-on-one or groups). in authority or speaking in public? Are you afraid of blood, small animals or Establish precise feared situation. heights? Do you repeat actions that you feel are Ask about washing, counting, checking and hoarding. excessive? Do you have thoughts that keep going in Ask nature of thoughts (illness, harm, sex) Relieved by washing hands/praying. your mind that you can't stop? Do these thought cause stress for you? How do you relieve this stress? Have you experienced any emotionally Establish the nature (accident, sexual, torture) and timing of the trauma. stressful events? When & What happened? Ask about worries related to health, family, Do you worry a lot of the time? job and finances.

P=PSYCHOSIS 1. 2. 3. 4. 5. 6. 7. 8.

HALLUCINATIONS: VISUAL HALLUCINATIONS: Do you sense things that are not actually there? Do you see things that others do not see? What do you see? Can you describe what you see? Does it have a message for you? Does the message ask you to harm yourself? How do you feel about it? Is this the first time? AUDITORY HALLUCINATIONS: 1. Do you hear voices other people cannot hear? OR : a) If you’re alone & nobody with you, do you hear voices? b) Do you hear voices inside your head? 2. How many voices? 3. Are the voices familiar? 4. Do you recognize the voices? 5. Do they talk to you?

Please note that this is only a draft version based on several sources, including: Dr. Basel Mohasen’s lectures, Toronto Notes, Therapeutic Choices and others. Edited and organized for the sake of all attendances of the Canadian Osce Exams: NAC OSCE and MCCQE2. by: Dr. Merlyn D’Souza and Dr. Zeev Gross, Spring 2011. Page 187

6. Do they talk about you? 7. What are they asking you to do? 8. Do they ask you to harm yourself? 9. Do they ask you to harm anybody else? If YES: 10. What is preventing you from doing this?........Screens for INSIGHT 11. How do you feel about these voices? (“Some people feel comforted when they hear these voices, others feel threatened”). DELUSIONS: 1. Do you feel anyone wants to hurt you or harm you? If YES: WHO & WHY? 2. Anybody tries to control you? 3. Anybody wants to put thoughts into your head? (Thought Insertion)  Anybody wants to steal thoughts from your head? (Thought withdrawl) 4. Others can read your thoughts? (Thought broadcasting) 5. When you’re watching TV or reading the News, do you feel they’re talking about you? (Ideas of reference) 6. Do you feel any part of your body is rotting? 7. Do you feel everybody is falling in love with you?

S=SELF CARE   

HOMICIDE: SUICIDE SOCIAL HISTORY: o With whom do you live? o How do you care for yourself?

PAST PSY HISTORY: 1. Any similar Sx/Ds in past? 2. Any Other psy Sx/ Ds in past? If YES: 3. Analyse Sx/extent of incapacity/Rx recvd/names of hosp/Compliance PAST MEDICAL HISTORY: R/OAny medical illness:DM/HTN/Thyoriod/Surgery/Head trauma/HIV/AIDS/Syphillis SAD – Smoking, Alcohol. Drugs (especially long use of cocaine) Screen for anxiety – are you fear a lot? any fears, especially from open places? Screen for psychosis – “anybody wants to harm you? Sometimes people having similar experience – they might hear voices or see things other people don’t see. How about you?” FHx: Please note that this is only a draft version based on several sources, including: Dr. Basel Mohasen’s lectures, Toronto Notes, Therapeutic Choices and others. Edited and organized for the sake of all attendances of the Canadian Osce Exams: NAC OSCE and MCCQE2. by: Dr. Merlyn D’Souza and Dr. Zeev Gross, Spring 2011. Page 188

 Anyone in family with similar Sx/Ds  Anyone in family with other pSy Sx/Ds  Drinking / hospitalized from psychiatry reason?  Relationships SADD FHx:  Suicide  Alcohol  Depression/Divorce/Drug PERSONAL Hx: 1. Prea-dulthood 2. Adulthood: Social activity: Support system, Friendships (depth/duration/Quality) isolated, asocial With whom do you live? If he lives alone – do you have any friends you talk with? Current Living Situation: Where/with whom/Relationships at home/financial support/Assistance OCCUPATIONAL HX: How do you support yourself? What are your ambitions/goals/relationships/Conflicts at work?/STRESSES/ Job changes MARITAL & RELATIONSHIP Hx; Age/Duration/areas of (dis) agreements,outcomes MILITARY Hx Gen Adjustement,combat,Injury Educational Hx: Highest grade/Area of interest Religion:Strict/Permissive attitude towards suicide PSYSOCIAL Hx: Sx/attitudes/orientation/practises/STDs (HIV) 1. Are you currently in a relationship? 2. Are you sexually active? 3. Are you active with males, females or both? 4. How long have you been in the current relationship? 5. Are you practising safe sex? 6. Are you using condoms all the times or just sometimes? 7. Is there a risk for you to be at a risk for STDs like HIV/HBV/Syphilis? 8. How about your partner? 9. How about your previous partners? Or the previous partners of your partner? 10. Have you or your partner tested for HIV,HBV or Syphilis?/When/Outcome 11. Are you currently seeing anyone else? 12. What other relationship have you had in the past?/ Anytime with more than one person at a time 13. Have you ever paid/received money for Sex? Please note that this is only a draft version based on several sources, including: Dr. Basel Mohasen’s lectures, Toronto Notes, Therapeutic Choices and others. Edited and organized for the sake of all attendances of the Canadian Osce Exams: NAC OSCE and MCCQE2. by: Dr. Merlyn D’Souza and Dr. Zeev Gross, Spring 2011. Page 189

Allergies

Back to Content

MSE/MMSE APPEARANCE: 1. Well dressed 2. Well groomed 3. Dress matches weather 4. Given age matches chronological age BEHAVIOUR: 1. Agitated 2. Psychomotor retardation 3. Eye Contact 4. Co operative 5. Non hostile 6. No abnormal movts/Jerking/lip smacking C/SPEECH: 1. Volume 2. Tone 3. Fluency 4. Articulate MOOD& AFFECT: Mood;Subjective Sx in pts own words Affect (qarms) 1. Quality: Euthymic/depressed/elevated/Anxious 2. Appropiateness to thought content 3. Range:Full/Restricted/Flat/Blunted 4. Mood Congruence 5. Stability: Fixedt/Labile PERCEPTION: Hallucination Illusion THOUGHT PROCESS: Coherence/Incoherent Logical/Illogical Please note that this is only a draft version based on several sources, including: Dr. Basel Mohasen’s lectures, Toronto Notes, Therapeutic Choices and others. Edited and organized for the sake of all attendances of the Canadian Osce Exams: NAC OSCE and MCCQE2. by: Dr. Merlyn D’Souza and Dr. Zeev Gross, Spring 2011. Page 190

Circumstantiality/Tangentiality THOUGHT CONTENT:  Suicidal/Homicidal Ideation 1. Low-- fleeting thoughts,no formulated plan,no Intent 2. Intermediate--More frequent ideation,well formulated plan,No active intent 3. High --Persistent ideation & profound hopelessness/Anger,well formulated plan,active intent,believes suicide,homicide is only helpful option available  Obsession: 1. Recurrent or persistent thoughts,impulses or images that cannot be stopped which is intrusive or inappropriate 2. Cannot be stopped by reason & Causes marked anxiety & distress  Preoccuption:  Overvalued Ideas:  Ideas of reference:  Delusions:  Magical thinking:  First Rank Sx of Shz:Thought insertion/T withdrawal/T broadcasting COGNITION: MMSE Level of consciousness Orientation in time/place/person Memory: immediate,remote,recent Attention & Conc Global evaluation of intellect: Intellectual Fns: INSIGHT: JUDGEMENT:

Back to Content

Please note that this is only a draft version based on several sources, including: Dr. Basel Mohasen’s lectures, Toronto Notes, Therapeutic Choices and others. Edited and organized for the sake of all attendances of the Canadian Osce Exams: NAC OSCE and MCCQE2. by: Dr. Merlyn D’Souza and Dr. Zeev Gross, Spring 2011. Page 191

DELERIUM MMSE 1st reassuare the pt,calm him down talk & do MMSE

O-O-O-O-O = 5 = Time: Year/Season/Month/Day/Date O-O-O-O-O = 5 = Place:Country/Province/City/Street/No O-O-O = Immediate recall:Black/Honesty/Tulip (if he makes mistakes,correct him but give _ve

O-O-O-O-O = Concentration: Can you spell WORLD backwords? O-O-O = Delayed recall O-O-O = Comprehension: 3 step command O-O = Naming 2 objects (pencil & paper) O = Reading ; write a sentence: Close your eyes & ask him to follow the command O= Writing O = Repeating; No ifs ands or buts O= Copying Why Delerium: Fever Ha/photophobia? Did you eat last night? Abdominal pain /Flank pain? Calf pain? Medications Alcohol (Last time & now) CSx: Ask examiner for I/O chart & medication chart

Back to Content Please note that this is only a draft version based on several sources, including: Dr. Basel Mohasen’s lectures, Toronto Notes, Therapeutic Choices and others. Edited and organized for the sake of all attendances of the Canadian Osce Exams: NAC OSCE and MCCQE2. by: Dr. Merlyn D’Souza and Dr. Zeev Gross, Spring 2011. Page 192

FORMS to Be filled: [TN10-PS52] If during an interview a pt decides to leave & not finished...... If pt wants to kill someone or himself....ADMIT If Pt refuses to be admitted & insists on leaving: INVOLUNTARY ADMISSION----------FORM 1 And another doctor must come & asses him. Cannot hold in hospital for > 72 hrs If a wife /partner brings & dr assesses & there may be a chance that the Pt may commit suicide/homicide, pt can be sent home, on condition that if Pt detoriates she should call back & immediately & bring Can file FORM1 If pt refuses voluntary admission with first dr,but second D. Can assess & can discharge if he feels fit for discharge,or admit on VOLUNTARY basis Admission always better on voluntary basis If second dr admits on involuntary basis it is FORM 3 & valid for 2 wks During these 2 wks,pt improves,& so can be discharged, or gets voluntary admission When admission voluntary --- FORM 4 Form 4 is renewed Released on --FORM 5 Thus FORMS 3 & $ are for Rx

Back to Content

Please note that this is only a draft version based on several sources, including: Dr. Basel Mohasen’s lectures, Toronto Notes, Therapeutic Choices and others. Edited and organized for the sake of all attendances of the Canadian Osce Exams: NAC OSCE and MCCQE2. by: Dr. Merlyn D’Souza and Dr. Zeev Gross, Spring 2011. Page 193

PANIC Attack ONSET: 1. When did it start 2. How did it start? 3. What were you doing at that time? COURSE: 1. Is the Intensity same now as it was when it all started? 2. How about the frequency? 3. What made you come in today? DURATION: How long does each attack last? PQRSTUV 1. Where exactly does it hurt you? 2. Can you describe the pain? 3. Does it move to anywhere else n your body? 4. On a scale of 1 to 10,wher 1 is mild & 10 is max,where would you rate this pain? 5. Has this affected your life in any way? 6. Is there a particular time it comes on? 7. Did you ever have this before? AAA Alleviating Fcts: What makes it better? What Aggravates it? 1. Exercise 2. Stress 3. Certain situations or places? 4. Coffee? 5. Medications? 6. When passing urine or having a bowel movt or cough? ARE YOU AFRAID THAT AN ATTACK IS COMING? Assoc Sx: first R/o cardiac then GI & then shift to Psy  N/V/Diarrhoea  Heart racing/ Sweating/ Dizzy  Decreased wt & Increased appetite/ Tremors  Headache  Tingling & Numbness  Nervous & Out of control  Do you feel you’re going to die?  During these attacks do you feel things are unreal?  During these attacks do you feel you can see yourself?  Do you feel you’re going crazy?

Please note that this is only a draft version based on several sources, including: Dr. Basel Mohasen’s lectures, Toronto Notes, Therapeutic Choices and others. Edited and organized for the sake of all attendances of the Canadian Osce Exams: NAC OSCE and MCCQE2. by: Dr. Merlyn D’Souza and Dr. Zeev Gross, Spring 2011. Page 194



Do you feel worried about being in places or situations where escape might not be possible  e.g: Crowded places  Closed spaces

If YES: * What place or situation? * What happens in such a situation? * How has it impacted your life? * How have you dealt with this issue? Here ask for anxiety disorder: Are you a person who worries a lot?

Interview Questions to Establish Specific Anxiety Diagnosis 1. 2. 3. 4. 5. 6. 7. 8. a.

Questions Do you have sudden episodes of intense anxiety? Do you have difficulty going to places to which you used to be able to go? Do you have difficulty talking to people in authority or speaking in public? Are you afraid of blood, small animals or heights? Do you repeat actions that you feel are excessive? Do you have thoughts that keep going in your mind that you can't stop? Have you experienced any emotionally stressful events? Do you worry a lot of the time?

Further Inquiry Establish nature of attack Inquire about crowded places, line-ups, movies, highways, distance from home. Establish situations (one-on-one or groups). Establish precise feared situation. Ask about washing, counting, checking and hoarding. Ask nature of thoughts (illness, harm, sex). Establish the nature (accident, sexual, torture) and timing of the trauma. Ask about worries related to health, family, job and finances.

MOAPPS

MOOD RISK FCTS: To gain more insight into your condition, I need to ask some questions about your personal life : Do you:  Smoke  Take alcohol  Recreational drugs (in case of cocaine – ask if sniffs or injects it. If injects – continue by r/o HIV symptoms)  Are you on any medications?  Did you take anything for a cold or flu  Are you taking OTC products/herbal remedies?  Are you allergic to anything?  When was your last period? Are you going through a stressful situation in your life? Please note that this is only a draft version based on several sources, including: Dr. Basel Mohasen’s lectures, Toronto Notes, Therapeutic Choices and others. Edited and organized for the sake of all attendances of the Canadian Osce Exams: NAC OSCE and MCCQE2. by: Dr. Merlyn D’Souza and Dr. Zeev Gross, Spring 2011. Page 195

How are you coping with it? FAMILY H: Does anyone in your family have a similar condt? SOCIAL H: COUNSELLING: From what you’ve told me, your chest pain seems related to a condition called “Panic Attack” It is a fairly common condt   

It's not known what causes panic attacks or panic disorder. Things that may play a role include: Genetics Stress Certain changes in the way parts of your brain function Some research suggests that your body's natural fight-or-flight response to danger is involved in panic attacks. For example, if a grizzly bear came after you, your body would react instinctively. Your heart rate and breathing would speed up as your body prepared itself for a life-threatening situation. Many of the same reactions occur in a panic attack. But it's not known why a panic attack occurs when there's no obvious danger present

Nonpharmacologic Choices     

Caffeine or other stimulant use should be reduced and controlled. Alcohol use should be minimal; it should not be used to control anxiety. Reduce the “as-needed” use of short-acting benzodiazepines as much as possible; ideally, such use should not be continued for longer than 4 days. Stress reduction, including relaxation training and time management, is often helpful initially. Specific cognitive behavioural therapy (CBT) may be required;

he selective serotonin reuptake inhibitors (SSRIs) citalopram, escitalopram, fluoxetine, fluvoxamine, paroxetine and sertraline are all effective in reducing panic SSRIs and SNRIs have become first-choice agents in treating panic disorder with or without agoraphobia.2There is usually a delay in response to these agents that may be accompanied by initial agitation. Combining the SSRI or SNRI with a brief course of low-dose benzodiazepine augmentation therapy (i.e., no longer than 8 weeks) can increase adherence to medication and produce a more rapid response than with antidepressants alo

Panic Disorder with Agoraphobia The pharmacologic treatment of panic disorder with agoraphobia is the same as for panic disorder. However, much of the disability in panic disorder with agoraphobia arises from the avoidance behaviour rather than the panic attacks. This can be addressed with cognitive behavioural therapy (CBT), even if medication reduces or eliminates panic attacks. CBT can be more effective alone than when it is combined with medication.8 However, access to specialized CBT is often limited.

Back to Content

Please note that this is only a draft version based on several sources, including: Dr. Basel Mohasen’s lectures, Toronto Notes, Therapeutic Choices and others. Edited and organized for the sake of all attendances of the Canadian Osce Exams: NAC OSCE and MCCQE2. by: Dr. Merlyn D’Souza and Dr. Zeev Gross, Spring 2011. Page 196

Personality Disorders

Please note that this is only a draft version based on several sources, including: Dr. Basel Mohasen’s lectures, Toronto Notes, Therapeutic Choices and others. Edited and organized for the sake of all attendances of the Canadian Osce Exams: NAC OSCE and MCCQE2. by: Dr. Merlyn D’Souza and Dr. Zeev Gross, Spring 2011. Page 197

“Am I crazy?” “There is no medical condition called like that, however sometimes patient have some difficulties with their thoughts and reality, it is called schizophrenia.” Mental Status Exam: Appearance wise...dressed, gromed Behavioral wise: Speach wise: Mood wise: Perception: Thought processing: Thought content: Judgement: Mini-Mental: delirium, dementia, post-concussion Writing a chart (SOAP): Subjective Objective Assessment Plan

Borderline Personality Work on this event and previous attempts. If she was diagnosed – “have you ever seen by psychiatrist? What was the diagnosis? What you didn’t contacted your case manager/psychiatrist? ER or ICU or Weapons? In Toronto – contact with the case manager/psychiatrist. What is the trigger that makes her come today? In case of crisis – do you have anybody to contact? Refer to crisis team/Social worker/ “In order to determine if I can admit you or not I need more information... Always the same pressure like today? If the patient mentions work – “what do you do for work?” Anything happen recently? Have you had any other relations? Is it difficult for you to maintain relation? Mood, Anxiety, Drugs/Alcohol/

Back to Content Please note that this is only a draft version based on several sources, including: Dr. Basel Mohasen’s lectures, Toronto Notes, Therapeutic Choices and others. Edited and organized for the sake of all attendances of the Canadian Osce Exams: NAC OSCE and MCCQE2. by: Dr. Merlyn D’Souza and Dr. Zeev Gross, Spring 2011. Page 198

22 yof wants to be admitted. Borderline personality When Pt wants to be admitted she may say,if you do not admit her,something bad may happen like last time. Pick up early when she says this ...... Start with EVENT: Check previous attempts at suicide Pick up early when she says something bad happened Ask: WHAT happened? If Suicide attempt....... 1. When & How many times before 2. Was she seen by psychiatrist? 3. Has she been to ER before? 4. Was she diagnosed & Rxed 5. Why can;t she contact her psychiatrist? 6. Was she admitted in ICU? 7. Which Rx programme does she have? 8. Does she have a crisis team & case manager? 9. Why didn’t she contact them?

If repeated attempts at suicide: Which treatment programme does she have? Is admission one of it? CRITERIA FOR BPD:  Fluctuating mood either very happy or sad  Splitting  Feeling of emptiness  Failure in maintaining a relationship both on social & employment areas  Impulsivity Drugs & Sex  Was sexually abused as a child Let her go or admit 1st episode admit needs Psy assessment Look for TRIGGERS that made her come in today Do not let her manipulate you I really like to help you,I’m on Er duty My job is to asses you & admit you Once admitted another Dr will asses you   

Can you tell me why you want to be admitted? I really want to help you... What bad thing will happen?

Please note that this is only a draft version based on several sources, including: Dr. Basel Mohasen’s lectures, Toronto Notes, Therapeutic Choices and others. Edited and organized for the sake of all attendances of the Canadian Osce Exams: NAC OSCE and MCCQE2. by: Dr. Merlyn D’Souza and Dr. Zeev Gross, Spring 2011. Page 199

In order to admit you or not, I need more info & therefore I need to ask you, & admitting you is one option If pt says Something Bad will happen o What do you mean? o When did it start? o Did it happen before? o How many times before? NO EMPATHY When did it happen the 1st time? When was the last time? What was done? Were you admitted to ICU? In addition to slashing your wrists have you used any other methods?...Like weapons or medications? Pt says she feels some pressure Ask if the pressure felt today is the same as the pressure felt last time I want to help you looks like the last few days were stressful Have you been seen by Psy? What was the diagnosis? Do you still see the Psy? When was the last time you saw him/Her? Why did you stop? What medication were/are you taking? In addition to psy is there a case manager? In case of crisis do you have anybody to talk to/ or contact? What prevented you from talking to them today? Ask prior to Event? What happened 6 wks ago? How did you lose your job? What sort of job were you doing? What happened? Prior to that what sort of job were you doing? Is it difficult for you to stay in one job? Have you tried to find another job? Anything happened last night? If Boy friend left her.....How long have you been together? How does she feel about it? Was he supportive? At what age was she sexually active? Does she find it difficult to stay in a relationship? MOOD DEPRESSED: Ask for HOMICIDAL ideation Please note that this is only a draft version based on several sources, including: Dr. Basel Mohasen’s lectures, Toronto Notes, Therapeutic Choices and others. Edited and organized for the sake of all attendances of the Canadian Osce Exams: NAC OSCE and MCCQE2. by: Dr. Merlyn D’Souza and Dr. Zeev Gross, Spring 2011. Page 200

Manic Ask Drugs/Spending/Impulsivity ANXIETY After boyfriend ask about fly support......

Back to Content

Please note that this is only a draft version based on several sources, including: Dr. Basel Mohasen’s lectures, Toronto Notes, Therapeutic Choices and others. Edited and organized for the sake of all attendances of the Canadian Osce Exams: NAC OSCE and MCCQE2. by: Dr. Merlyn D’Souza and Dr. Zeev Gross, Spring 2011. Page 201

Depression Sleep If comes Tired  Sleep  Energy  Mood Sleep – how does it affect you? DO you feel tired?  

―During the past month have you often been bothered by feeling down, depressed or hopeless?‖ ―During the past month have you often been bothered by little interest or pleasure in doing things?‖

MI PASS ECG (TO diagnose depression – needs 5 out of the 9, in which one them should be either M or I. In teenagers irritability can replace either M or I, in elderly it can come with a somatic presentation) If it is one episode it is called: Major Depressive Episode (need 2w in which most of the days with depressed mood, and 4 more criteria). If there are 2 or more MDE within the same 2m – it is Major Depressive Disorder, if it is more than 2m – it is Recurrent Major Depressive Episodes. M How do you feel recently? How is your mood? Any chance you are depressed? I What do you enjoy doing? Are you still enjoy hobies? Anything brings happiness to you? Why don’t you enjoy any more? No time? No energy? P Do you think things are getting slower? Do you think you need more time to do things you used to do before? A Any change in appetite? Did you lose weight? How much weight did you lose? Was it intentional or not? S How about sleep? How many hours do you sleep? When do you go to bed? How long before you fall asleep? DO you wake up at night? Please note that this is only a draft version based on several sources, including: Dr. Basel Mohasen’s lectures, Toronto Notes, Therapeutic Choices and others. Edited and organized for the sake of all attendances of the Canadian Osce Exams: NAC OSCE and MCCQE2. by: Dr. Merlyn D’Souza and Dr. Zeev Gross, Spring 2011. Page 202

Why do you wake up at night? DO you feel refreshed at the morning? S Do you feel any chance that you might harm yourself, end your life, or any ones? If patient says I wish I am dead, consider either he has only a feeling or a plan (active) Do you have a plan? What is preventing you? Did you leave a note? Did you start giving your belongings to others? (These are definite questions for a plan) E DO you feel tired? C Do you find it difficult to focus on a specific task (for example if you are watching TV – you can stick to the same program all through? Can you finish an article?) G DO you feel there is no hope in your life? Do you feel guilty?

Regardless of any specific psychotherapy, measures to enhance treatment compliance are useful; e.g., providing psychoeducation with the following 5 simple messages is effective:19     

Take medication daily Call this number for questions about side effects or other issues Remember that it might take 2–4 weeks to see a noticeable effect from antidepressants Continue to take medication even if you are feeling better Do not stop taking the antidepressant without checking with the physician

Please note that this is only a draft version based on several sources, including: Dr. Basel Mohasen’s lectures, Toronto Notes, Therapeutic Choices and others. Edited and organized for the sake of all attendances of the Canadian Osce Exams: NAC OSCE and MCCQE2. by: Dr. Merlyn D’Souza and Dr. Zeev Gross, Spring 2011. Page 203

42 yom hasn’t been himself, his wife arranged for the meeting. INTRO: As I understand you’re here today,as your wife has some concerns about you. Can you tell me more about it? Give confidentiality. Counseling (last 2-3m): Based on what you told me your symptoms are consistent with a condition called “depression.” We believe it is caused because of imbalance in some of the chemicals in the brain. Sometime there is an event in life or cause that triggers that situation. It is common and treatable. We need to r/o other causes – and for that we need to do some blood work. What did I do wrong to feel so depressed? Depression does not occur because someone has done something "wrong". Like any other medical illness, depression is caused at least in part by biochemical changes in the brain, which lead to depressive symptoms. This is why medications which help correct chemical imbalances in the brain relieve depression. In fact, if a chemical imbalance is not present, antidepressant medications will not have any effect - they will not make a person "happy" when they are not clinically depressed. How long before I feel better? Generally speaking, people will start to notice improvement in symptoms such as sleep disturbances or crying spells and energy levels a few weeks after starting their treatment. Improvement in depressed mood is usually slower, and it may take six to eight weeks before people notice they are feeling much less depressed. If someone has not improved after three to four weeks of therapy, the dose of the initial medication may be optimized, a different drug may be added, or the initial drug may be substituted. Up to 80% of people with depression do get better with the right medication. Will my depression come back? The likelihood of depression recurring depends on how many previous episodes you have had. For people who are experiencing their first depression, the likelihood of having a second episode is around 50%. For people who've had two depressive episodes, chances of having a third are around 70% and for Those who've had three and more episodes, all but 10% will experience further illness. Having someone else in your family who has depression makes it more likely your own depression will recur. Other risk factors for recurrent depression are the presence of chronic medical problems, a history of early trauma or abuse, dysthymia, onset of depression younger than 25 years or older than 60 years, and a long pattern of negative thinking, low self-esteem and relationship difficulties. A depression which does not completely resolve with treatment, as well as severe depression, also increase the likelihood depression will recur. This is why most people with depression need to be treated for at least six to nine months to prevent relapse, and for greater than 12 months if someone is being treated for a recurrent episode. Depending on the likelihood of depression recurring, some people stay on the same dose of their medication for long-term maintenance therapy. The saying doctors have is, "The dose that gets you well is the dose that keeps you well" and people will do better over the long run if the same dose is used throughout.

Please note that this is only a draft version based on several sources, including: Dr. Basel Mohasen’s lectures, Toronto Notes, Therapeutic Choices and others. Edited and organized for the sake of all attendances of the Canadian Osce Exams: NAC OSCE and MCCQE2. by: Dr. Merlyn D’Souza and Dr. Zeev Gross, Spring 2011. Page 204

Can I pass depression on to my children? Certain types of depression, especially, bipolar affective disorder, would appear to run in families. However, even identical twins do not share an equal risk to develop depression, and depressive illness appears to be a combination of vulnerability to depression (part of which may be inherited but not necessarily), difficult life events and biochemical imbalances in the brain. I have trouble reaching orgasm now that I'm taking an SSRI. Can I stop my medication on weekends to improve my sexual function? Some doctors recommend drug holidays where people stop taking their medication on the weekend. The biggest concern about stopping and starting medication revolves around compliance issues, but there is some evidence that people may not respond as well to the medication if treatment is continuously interrupted. For these reasons, drug holidays are not recommended and an alternative antidepressant or an additional medication to offset unwanted sexual side effects are better solutions.

Regardless of any specific psychotherapy, measures to enhance treatment compliance are useful; e.g., providing psychoeducation with the following 5 simple messages is effective:19     

Take medication daily Call this number for questions about side effects or other issues Remember that it might take 2–4 weeks to see a noticeable effect from antidepressants Continue to take medication even if you are feeling better Do not stop taking the antidepressant without checking with the physician

There are good options to treat it. If you choose to go to talk therapy I can refer you to a psychology. On the other hand we can use medications which are generally safe. Called SSRI similar to Prozac, however like any other medical intervention have some side effects. Most of them are minor, usually improve with time – headache, sexual...however the improvement of your mood will lag behind your improvement in your energy, we call that the window gap, and this is of concern to us. All contracts are verbal, besides the drug contracts – “I promise I will not use ...again...”

Back to Content

Please note that this is only a draft version based on several sources, including: Dr. Basel Mohasen’s lectures, Toronto Notes, Therapeutic Choices and others. Edited and organized for the sake of all attendances of the Canadian Osce Exams: NAC OSCE and MCCQE2. by: Dr. Merlyn D’Souza and Dr. Zeev Gross, Spring 2011. Page 205

Mania DIG FAST For diagnosis we need elevated mood + 3 criteria of the above 7 for a whole week. Sometimes irritable – you need 4 criteria.

D DO you find it difficult to focus on one subject? Are you working on more than one project on the same time? How many projects are you working on? Can you finish it on time? I Are you spending more money than before? Are you borrowing money from others? Are you maxing out on your credit card? Do you drink alcohol? Are you drinking more than before? DO you smoke or take recreational drugs? (If taking recreational drugs – feeling high) If taking cocaine – what happened first: the episode or the taking the drug? With whom do you live? Are you sexually active? How many partners have you had recently? Have you used protection? DO you have any problems with law? Any speeding tickets? Any fights? G DO you feel you are special? Do you feel you deserve to be treated differently? DO you have special powers? Do you have special mission? F DO you have thoughts racing in your head? What kind of thoughts? A How much time do you spend in your activities? S Lack of sleep? T Did anyone tell you that you are talking faster than before? Please note that this is only a draft version based on several sources, including: Dr. Basel Mohasen’s lectures, Toronto Notes, Therapeutic Choices and others. Edited and organized for the sake of all attendances of the Canadian Osce Exams: NAC OSCE and MCCQE2. by: Dr. Merlyn D’Souza and Dr. Zeev Gross, Spring 2011. Page 206

Manic 1 Manic episode = Bipolar I Ask if it is the first time or has it happened before. How about the opposite. Have you ever felt high? Greater than 7 days in a row?

Insomnia in elder Difficulty in sleeping for 6 months Can you tell me more about it since it started? Did you seek medical attention? OCD Anything at that time? From that time till now – every night? When do you go to bed? Whe do you fall asleep? When do you wake up? Before you fall asleep what do you think? What comes to your mind? When you sleep – do you wake up? Any nightmares? If she says she has to wake up for breakfast ask why she has to wake up? How old is your son? Has he been always with you or is he left and come back? Can’t the prepare breakfast for himself? Anybody else at home? How about your husband? CSx PMHx SHx How does son support himself? Is he under stress? What is the nature of your relationship? Give confidentiality? How do you support yourself financially? Any financial concern? Ask if son contributes to finances? If son consumes Alcohol? (How much? Does he loose control/shouts?) Does he get angry to swear to get physical? Does he get accesses to your finances? Did you talk with anybody about it? Do you feel safe going back home? Does he have access to fire arms? DO you have suicidal or homicidal ideation? Based on what you told me – your sleeping troubles seems to be related to stresses in your life called ―Elder Abused‖ which is illegal and crime against law. It is nor your mistake and you should not accept that. You need to call the police. From studies it has shown that police interevention improves such situations. Son needs help – can you convince him? Please note that this is only a draft version based on several sources, including: Dr. Basel Mohasen’s lectures, Toronto Notes, Therapeutic Choices and others. Edited and organized for the sake of all attendances of the Canadian Osce Exams: NAC OSCE and MCCQE2. by: Dr. Merlyn D’Souza and Dr. Zeev Gross, Spring 2011. Page 207

Police will protect you and son will be sent to rehab and anger management. I’ll be giving you sleeping pills for three days and f/u within 3d.

Back to Content

Please note that this is only a draft version based on several sources, including: Dr. Basel Mohasen’s lectures, Toronto Notes, Therapeutic Choices and others. Edited and organized for the sake of all attendances of the Canadian Osce Exams: NAC OSCE and MCCQE2. by: Dr. Merlyn D’Souza and Dr. Zeev Gross, Spring 2011. Page 208

Marijuana Counselling (Mother comes in to see you as she has discovered Marijuana in her son’s belongings) INTRO: As I understand you’re here because you’re concerned about your son. What is his name? What is your concern? How much did you find? Did you ask him about it?  WHAT MAKES YOU BELEIVE IT IS MJ ? Is he using it? Or Is he carrying it? Is it the first time you’ve found it?  Did you notice any CHANGES in his behaviour? Is he excited? Laughing out of nowhere? Is he preoccupied? Does he stare at a wall? Does he talk to himself? Is he aggressive? Any problems with the law? Any fights? Any criminal records? Is he more isolated?  How is his MEMORY? Is he more forgetful/lose his stuff? Does he take more time to react? Does he spend more time in his room? How much time do you spend with him? How much time is he out of the home? How much time does he spend with his friends? Do you know any of his friends? What kind of activity are they involved in?  Does he have a lot of MONEY? Does he ask for money? Do you believe he steals money? Do you think he smokes/or drinks alcohol?  How would you describe his MOOD? Is he depressed? Is he still interested in his hobbies? Does he worry a lot? Does he have excessive fears & avoid situations? Do you have concerns that he may harm himself or anyone else? EDUCATION: How is he doing in school? Have his grades dropped? DIET: How is his general health? Please note that this is only a draft version based on several sources, including: Dr. Basel Mohasen’s lectures, Toronto Notes, Therapeutic Choices and others. Edited and organized for the sake of all attendances of the Canadian Osce Exams: NAC OSCE and MCCQE2. by: Dr. Merlyn D’Souza and Dr. Zeev Gross, Spring 2011. Page 209

Have you ever seen a psychiatrist? Fhx: SAD

COUNSELLING: Based on what you’ve told me.There are no changes in his health & behaviour (assumed that there were no changes in behaviour as per mum) When it comes to Marijuana it is a commonly used drug by teenagers, sometimes only once for experiment. When we talk about Substance Abuse & drugs we talk about different categories. Marijuana is a SOFT DRUG,others like: Coccaine,Heroin& Amphetmanies are HARD DRUGS Let us talk about Marijuana first. It is from the Cannabis family & affects the brain by feeling happy, excited & enhances experience.Sometimes with prolonged use or in high doses can cause side effects including apathy. It interferes with memory,& can interfere with his studies & function & fine motor skills & may not be able to operate machinery It impairs judgement & he might take risks. Can cause Lung cancer In some teens,in high doses unmasks schizophrenia & cause psychosis Interferes with sexual function & can cause infertility & weight gain By itself marijuana is not strongly addictive & hence he can stop it at any time with help.One of the concerns of Marijuana though is it acts as a bridge to Hard drugs which are addictive i.e you’ve to increase the dose to have the same effect,which is called “TOLERANCE”,& then one cannot stop the drug as it causes withdrawal . It is a crime to use,hold hard drugs.People can lose their jobs. If injected increases risk of HIV,Hepa B & C

PLAN If you like,bring your son here I can talk to him. It is better to be a confidante to him. Try to be close to him, someone he can trust & can talk to.Try to make sure who’re his friends,& make sure you know what he is doing.Keep him busy with activities. If there are any druh prevention programmes in your community or his school,get him to attend them & gets the knowledge.

In case of the resident who was asked to backup his supervisor orthopaed 4. I am competent – to emphasize 5. Short term – we don’t have time so we need to see her urgently 6. Long term – solve the situations that it wouldn’t occur again

Back to Content

Please note that this is only a draft version based on several sources, including: Dr. Basel Mohasen’s lectures, Toronto Notes, Therapeutic Choices and others. Edited and organized for the sake of all attendances of the Canadian Osce Exams: NAC OSCE and MCCQE2. by: Dr. Merlyn D’Souza and Dr. Zeev Gross, Spring 2011. Page 210

INSOMNIA: The Sleep History 1. Time data (can also be collected as part of a sleep diary – 1. Did you nap or lie down to rest today? If yes, when and for how long? 2. What time did you go to bed last night? 3. What time did you put out the lights? 4. How long did it take you to fall asleep? 5. How many times did you awaken last night? 6. How long was your longest awake period; when was it? What time did you finally awaken? 7. What time did you get out of bed? 8. How many hours sleep did you get last night? 2. Questions about the sleep period 1. Do physical symptoms, such as pain, prevent you from falling asleep? 2. Do mental or emotional symptoms (e.g., worry or anxiety) prevent you from falling asleep? 3. When you awaken during the night, what awakens you? (Snoring? Gasping for air? Dreams/nightmares? Noise?) 4. When you get up for the day, do you have any symptoms? (Headache? Confusion? Sleepiness?) 3. Questions for the patient's bed partner 1. Does your partner snore, gasp or make choking sounds during the night? 2. Does your partner stop breathing during the night? 3. Do your partner's legs twitch, jerk or kick during the night? 4. Has your partner's use of alcohol, nicotine, caffeine or other drugs changed recently? 5. Has your partner's mood or emotional state changed recently? 6. What do you think is the cause of your partner's sleep problem? Hygiene Guidelines 1. Keep a regular sleep–wake schedule, 7 days per week. 2. Restrict the sleep period to the average sleep time you have obtained each night over the preceding week. 3. Avoid sleeping in, extensive periods of horizontal rest or daytime napping; these activities usually affect the subsequent night's sleep. 4. Get regular exercise every day: about 40 minutes of an activity with sufficient intensity to cause sweating. If evening exercise prevents sleep, schedule the exercise earlier in the day. 5. Avoid caffeine, nicotine, alcohol and other recreational drugs, all of which disturb sleep. If you must smoke do not do so after 7:00 p.m. 6. Plan a quiet period before lights out; a warm bath may be helpful. 7. Avoid large meals late in the evening; a light carbohydrate snack (e.g., crackers and warm milk) before bedtime can be helpful. Please note that this is only a draft version based on several sources, including: Dr. Basel Mohasen’s lectures, Toronto Notes, Therapeutic Choices and others. Edited and organized for the sake of all attendances of the Canadian Osce Exams: NAC OSCE and MCCQE2. by: Dr. Merlyn D’Souza and Dr. Zeev Gross, Spring 2011. Page 211

8. Turn the clock face away and always use the alarm. Looking at the clock time on awakening can cause emotional arousal (performance anxiety or anger) that prevents return to sleep. 9. As much as possible, keep the bedroom dark and soundproofed. If you live in a noisy area, consider ear plugs. 10. Use the bedroom only for sleep and intimacy; using the bed as a reading place, office or media centre conditions you to be alert in a place that should be associated with quiet and sleep. If you awaken during the night and are wide awake, get up, leave the bedroom and do something quiet until you feel drowsy-tired, then return to bed. Note: Pharmacologic (or any) interventions will be less effective if these guidelines are not followed. In mild cases of insomnia, sleep hygiene guidelines, practised consistently and together, may be sufficient to reinstate a normal sleep pattern.

Difficult sleeping for the last 3m OCD How did it start? Suddenly / Gradually? From that time is it all the time or “on and off?” Shift to “whom do you live with?”

U: How did that lack of sleep affect your life?

Give confidentiality. R/O: Depression, Drinking (her or husband), “How much? How often? Does he drink more? What is the reason? Any change in your life? When your husband gets angry – does he start shout at you? (“Sometimes when people are drinking it can Did you go to the ER? How often you go to the ER? Did he ever shout at you? Does he swear?/Shout?/Call your names? How does it affect your self esteem? Did he ever become anger to the extent that he becomes physical? Pushing? Did he ever force you to have sex against your will? Did he ever hit the children? Did he ever abused you in front of the children? Who’s controlling spending? (If she says that the children are safe – you can say that children are smart and realize that).

Counselling Based on what you told me it is called “spouse abuse” it is illegal, it is a crime and against the law. You shouldn’t feel guilty about that. We know from studies that this situation will deteriorate, and without proper of help it might end badly. If you are concerned with the economic situation I’d like to know that there are a lot of resources. I’ll give the number of social support that We know from studies...he will have some restraining...usually situations might improve. Always give them follow up in three days.

Back to Content Please note that this is only a draft version based on several sources, including: Dr. Basel Mohasen’s lectures, Toronto Notes, Therapeutic Choices and others. Edited and organized for the sake of all attendances of the Canadian Osce Exams: NAC OSCE and MCCQE2. by: Dr. Merlyn D’Souza and Dr. Zeev Gross, Spring 2011. Page 212

55 yo, believe that have strange feeling in hands. Do mental exam. Either organic, late onset of schizophrenia, not complying with medication INTRO: 3 ways: 1.How did it start? OCD: What were you doing at that time? What happened at that time? Ms Franco 55/F strange feeling in (R) hand x 6 mo in ER talk to her for 10 mins

Can U tell me about it? Is it one or (B) hands? How did it start? Right now,how do you feel/ Right now u look concerened,anything bothering U? U’re in the right place What were U doing when it all started? Anything special happened at that time? Ask What events?....... Or Do you remember how it started 1st time? Is it all the time or off & on Any particular time of the day Any particular settings What is special in that setting? In your opinion what is responsible for it? May show a pic It may be like a radiation for you,but not for me Where exactly do you feel it? Anywhere else?Any weakness/numbess Ask for HOMICIDAL/SUICIDAL ideation Ask for Hallucinations:

P=PSYCHOSIS HALLUCINATIONS: VISUAL HALLUCINATIONS: 9. Do you sense things that are not actually there? 10. Do you see things that others do not see? 11. What do you see? You look preoccupied REASSUARE HER THAT SHE IS IN SAFE PLACE Please note that this is only a draft version based on several sources, including: Dr. Basel Mohasen’s lectures, Toronto Notes, Therapeutic Choices and others. Edited and organized for the sake of all attendances of the Canadian Osce Exams: NAC OSCE and MCCQE2. by: Dr. Merlyn D’Souza and Dr. Zeev Gross, Spring 2011. Page 213

DO NOT LOSE NERVE 12. Can you describe what you see? 13. Does it have a message for you? 14. Does the message ask you to harm yourself? 15. How do you feel about it? 16. Is this the first time? AUDITORY HALLUCINATIONS: 12. Do you hear voices other people cannot hear? OR : c) If you’re alone & nobody with you, do you hear voices? d) Do you hear voices inside your head? 13. How many voices? 14. Are the voices familiar? DO NOT LOSE NERVE 15. Do you recognize the voices? 16. Do they talk to you? 17. Do they talk about you? 18. What are they asking you to do? 19. Do they ask you to harm yourself? 20. Do they ask you to harm anybody else? If YES: 21. What is preventing you from doing this?........Screens for INSIGHT DELUSIONS: 8. Do you feel anyone wants to hurt you or harm you? If YES: WHO & WHY? 9. Anybody tries to control you? 10. Anybody wants to put thoughts into your head? (Thought Insertion)  Anybody wants to steal thoughts from your head? (Thought withdrawl) 11. Others can read your thoughts? (Thought broadcasting) 12. When you’re watching TV or reading the News, do you feel they’re talking about you? (Ideas of reference) 13. Do you think any part of your body is rotting? 14. Do you feel everybody is falling in love with you? MOOD ORGANIC: Since it is first time I’m seeing you.I’ve to ask you questions Please note that this is only a draft version based on several sources, including: Dr. Basel Mohasen’s lectures, Toronto Notes, Therapeutic Choices and others. Edited and organized for the sake of all attendances of the Canadian Osce Exams: NAC OSCE and MCCQE2. by: Dr. Merlyn D’Souza and Dr. Zeev Gross, Spring 2011. Page 214

Any long term diseases? Look for S/e of meds (Streoids,smoking,drugs) Head Injury Fever Csx: Look for Social Hx Past Psy Hx Self Care Admit If Pt asks: Am I crazy? Thre is no medical condt called crazy.Sometimes some pts have difficulty in handling their thoughts & this is called “Schizophrenia”

MSE APPEARANCE: 1. Well dressed 2. Well groomed 3. Dress matches weather 4. Given age matches chronological age BEHAVIOUR: 1. Agitated 2. Psychomotor retardation 3. Eye Contact 4. Co operative 5. Non hostile 6. No abnormal movts/Jerking/lip smacking C/SPEECH: 1. Volume 2. Tone 3. Fluency 4. Articulate MOOD& AFFECT: Mood;Subjective Sx in pts own words Affect (qarms) 1. Quality: Euthymic/depressed/elevated/Anxious 2. Appropiateness to thought content 3. Range:Full/Restricted/Flat/Blunted Please note that this is only a draft version based on several sources, including: Dr. Basel Mohasen’s lectures, Toronto Notes, Therapeutic Choices and others. Edited and organized for the sake of all attendances of the Canadian Osce Exams: NAC OSCE and MCCQE2. by: Dr. Merlyn D’Souza and Dr. Zeev Gross, Spring 2011. Page 215

4. Mood Congruence 5. Stability: Fixedt/Labile PERCEPTION: Hallucination Illusion THOUGHT PROCESS: 1. Coherence/Incoherent 2. Logical/Illogical 3. Circumstantiality/Tangentiality THOUGHT CONTENT:  Suicidal/Homicidal Ideation Low-- fleeting thoughts,no formulated plan,no Intent Intermediate--More frequent ideation,well formulated plan,No active intent High --Persistent ideation & profound hopelessness/Anger,well formulated plan,active intent,believes suicide,homicide is only helpful option available  Obsession: Recurrent or persistent thoughts,impulses or images that cannot be stopped which is intrusive or inappropriate Cannot be stopped by reason & Causes marked anxiety & distress  Preoccuption:  Overvalued Ideas:  Ideas of reference:  Delusions:  Magical thinking:  First Rank Sx of Shz:Thought insertion/T withdrawal/T broadcasting COGNITION: MMSE Level of consciousness Orientation in time/place/person Memory: immediate,remote,recent Attention & Conc Global evaluation of intellect: Intellectual Fns: INSIGHT: JUDGEMENT:

Back to Content

Please note that this is only a draft version based on several sources, including: Dr. Basel Mohasen’s lectures, Toronto Notes, Therapeutic Choices and others. Edited and organized for the sake of all attendances of the Canadian Osce Exams: NAC OSCE and MCCQE2. by: Dr. Merlyn D’Souza and Dr. Zeev Gross, Spring 2011. Page 216

24 yo, brought by his roommate because hasn’t been himself in the last 10days. D/d: 1. 2. 3. 4.

Ac. psychosis, Substance abuse, HIV, Mania

If started 10 days ago, why brought in today? (could’ ve been homicidal or suicidal) If carrying a book, ask Reason Ask Delusions for grandiosity :“Do you feel you’ve a special mission?--- if Yes Always ask what is

the mission? & probe deeper & enquire about Delusions

Mission imp May be Suicidal or Homicidal ideation

Back to Content

Please note that this is only a draft version based on several sources, including: Dr. Basel Mohasen’s lectures, Toronto Notes, Therapeutic Choices and others. Edited and organized for the sake of all attendances of the Canadian Osce Exams: NAC OSCE and MCCQE2. by: Dr. Merlyn D’Souza and Dr. Zeev Gross, Spring 2011. Page 217

17/M worried about contamination – wants to be admitted to get rid from it. 10min – counsel Delusions: Dd: 1. 2. 3. 4.

Schizophrenia Schiziod personality Disorder Schizotypal PD Isolated PD (older pt in 40s & usually delusions about fidelity)

Qns about Delusions

DELUSIONS: 15. Do you feel anyone wants to hurt you or harm you? If YES: WHO & WHY? 16. Anybody tries to control you? 17. Anybody wants to put thoughts into your head? (Thought Insertion)  Anybody wants to steal thoughts from your head? (Thought withdrawl) 18. Others can read your thoughts? (Thought broadcasting) 19. When you’re watching TV or reading the News, do you feel they’re talking about you? (Ideas of reference) MOOD: How is your mood,is it down,Up or N ? ORGANIC:R/O Head Injury CSx: HIV/Meningitis Medications: Steroids,smoking,alcohol,drugs S/e of meds,Pt may have stopped anti psychotics due to Se

Back to Content

Please note that this is only a draft version based on several sources, including: Dr. Basel Mohasen’s lectures, Toronto Notes, Therapeutic Choices and others. Edited and organized for the sake of all attendances of the Canadian Osce Exams: NAC OSCE and MCCQE2. by: Dr. Merlyn D’Souza and Dr. Zeev Gross, Spring 2011. Page 218

35 yo, believes that the RCMP chasing him. Persecutory delusions. Reassurance about his safety. DDx substance abuse. INTRO; Early on reassure pt that this is a safe place, & invite him to sit down As I understand you’re here because you have worries that the RCMP is chasing you. I want you to know that this is a safe place & please come & sit. I want to help you so please sit down Make sure he sits in front of you. Ask him:  Why chasing?  How long chasing?  How affecting him?  How does he handle it?  Does he talk to anyone about it? Here there is persecutory delusion Besides police does anyone else want to hurt him? Does he have special powers? FINISH the delusions Go to Hallucinations VISUAL HALLUCINATIONS:  Do you sense things that are not actually there?  Do you see things that others do not see?  What do you see?  Can you describe what you see?  Does it have a message for you?  Does the message ask you to harm yourself?  How do you feel about it?  Is this the first time? AUDITORY HALLUCINATIONS:  Do you hear voices other people cannot hear? OR :  If you’re alone & nobody with you, do you hear voices?  Do you hear voices inside your head?  How many voices?  Are the voices familiar?  Do you recognize the voices?  Do they talk to you?  Do they talk about you? Please note that this is only a draft version based on several sources, including: Dr. Basel Mohasen’s lectures, Toronto Notes, Therapeutic Choices and others. Edited and organized for the sake of all attendances of the Canadian Osce Exams: NAC OSCE and MCCQE2. by: Dr. Merlyn D’Souza and Dr. Zeev Gross, Spring 2011. Page 219

 

What are they asking you to do? Do they ask you to harm yourself?

Also ask for tactile hallucinations I see you’re scratching your hands Any other areas are scratching? When & How long?.....Pt will answer.... I do not know ....... Jump to cocaine Do you smoke/Take alcohol/Drugs I f Pt stands, you stand, reassure him & bring him back & ask again about drugs Did you take an increased amt recently? How do you take it? Snort/Smoke/IV? If IV ask about CSx;

MOOD R/o Mania & depression Suicide & Homicide If Pt leaves tell I want to file form 1 & call security

Back to Content

Please note that this is only a draft version based on several sources, including: Dr. Basel Mohasen’s lectures, Toronto Notes, Therapeutic Choices and others. Edited and organized for the sake of all attendances of the Canadian Osce Exams: NAC OSCE and MCCQE2. by: Dr. Merlyn D’Souza and Dr. Zeev Gross, Spring 2011. Page 220

17/M worried about contamination – wants to be admitted to get rid from it. 10min – counsel Delusions: Dd: Schizophrenia Schiziod personality Disorder Schizotypal PD Isolated PD (older pt in 40s & usually delusions about fidelity) Qns about Delusions MOOD: How is your mood,is it down,Up or N ? ORGANIC:R/O Head Injury CSx: HIV/Meningitis Medications: Steroids,smoking,alcohol,drugs S/e of meds,Pt may have stopped anti psychotics due to Se





    

A key feature of bipolar disorder is recurrent nonadherence to medication; including the patient in decision-making, together with psychoeducation, promotes a strong therapeutic alliance and enhances medication adherence. Patients taking lithium need to maintain their usual salt and caffeine intake and monitor fluid intake and output, making adjustments in the event of unexpected losses due to vomiting or diarrhea. During acute manic episodes, patients may exhibit increased tolerance to lithium. Advise patients taking antipsychotics about antipsychotic-associated body temperature dysregulation and strategies to prevent heat stroke (e.g., hydration, sun protection). For lithium-associated cognitive impairment, check lithium level and thyroid function. Lowering the dose or using a slow-release formulation may improve cognitive function. Patients who experience tremor while taking lithium may benefit from elimination of dietary caffeine, lithium dose reduction or addition of a beta-blocker such as propranolol or atenolol. Patients who experience diarrhea while taking slow-release lithium preparations may fare better with immediate-release formulations,17 particularly the oral liquid citrate salt.18 

Back to Content

Please note that this is only a draft version based on several sources, including: Dr. Basel Mohasen’s lectures, Toronto Notes, Therapeutic Choices and others. Edited and organized for the sake of all attendances of the Canadian Osce Exams: NAC OSCE and MCCQE2. by: Dr. Merlyn D’Souza and Dr. Zeev Gross, Spring 2011. Page 221

Pt wants to discontinue his Li st

If stop Li Relaspe 1 time: disct Rx:40 – 60 % 2nd time:80% 3rd time:>95% Can control BP1 but not cure 1. Want s to stop Li as handwriting not like before, Ask if any other concerns ..... Seems reasonable.... INTRO: As I understand, you’re her cuz you’ve been diagnosed with BP1 3 yrs ago & want to disct your Rx,during the next few minutes I will take Hx & towards the end hopefully we will reach a working plan Ask: Why do you want to discontinue? ASSES: MOOD disorder whether Mania/Depression Go back to mania specially when diagnosed Li if SE Asses: Psychosis Anxiety Organic Past Medical Hx Fhx Social Hx Fhx of Depression & BP1 Suicidal 7 Homicidal ideation Self care COUnselling INTRO Can you tell me more about your decision? Why? Any other reasons? These seem reasonable enough concerns & I’m glad you’re here today to talk about it Let me ask some qns How would you describe your mood today? Even if he says good..... Go through DIG FAST Grandiosity:

Ask for opposite mood Do U feel Low Please note that this is only a draft version based on several sources, including: Dr. Basel Mohasen’s lectures, Toronto Notes, Therapeutic Choices and others. Edited and organized for the sake of all attendances of the Canadian Osce Exams: NAC OSCE and MCCQE2. by: Dr. Merlyn D’Souza and Dr. Zeev Gross, Spring 2011. Page 222

MI PASS ECG 1. How were U Dsed as BP1? 2. What was done at that time? 3. Were U hospitalized? 4. Was there serious consequences? 5. Are you under reg F/U? 6. When was the last time you saw your Dr?

Li 1. Which medications are you on besides Li? 2. How much Li? 3. Is it measure d on a regular basis? 4. What was the level? 5. Any new meds/ or increase in dose? 6. How do you feel about taking Li? Any SE Have U got TSH measured? When was the last time it was measured? Do you feel cold? Inc wt/Dec conc? Drink more/Pee more? Any urine analysis? Screen for Ataxia:Any shakiness/falls/difficulty in balance? Nx/Vx/Abd pain? If TSh Inc ct with Thyroxine If Di early Stop later Ct with Thaizude GI Stop Tremor B Blocker Have you ever discontinued Li in past few yrs? TRANSITON: I know you’ve been asked this qn before, but I need to ask these qns PSYCHOSIS ANXIETY Past MH Fhx of depression/Suicide/BP1 How do u support urself financially? COUNSELLING: Compare between Mania & Depression What is your understanding about Mania? Mania is a condt that affects mood,in which people feel elevated it is one of the mood disorders it is common. Most people have depression, where people feel low & lose interest & Rx is often Talk therapy & medications. The Rx for Mania is lifelong,similar to DM,in which we can control Sx,but not cure it.There is a lot of research going on & ne day we hope to find a cure for it. If you choose to discontinue it,your chances of relapse are high upto 60% Please note that this is only a draft version based on several sources, including: Dr. Basel Mohasen’s lectures, Toronto Notes, Therapeutic Choices and others. Edited and organized for the sake of all attendances of the Canadian Osce Exams: NAC OSCE and MCCQE2. by: Dr. Merlyn D’Souza and Dr. Zeev Gross, Spring 2011. Page 223

Coming to Li levels if 1.2 upper level of (N) & we can decrease the dosea bit & see how it affects you. But you’ve to PROMISE me that at any time you spend more,sleep less etc contact me or go to ER ASAP. Pt may accept . When it comes to writing Thought block is not one of the SE of Li,give it time ,& see if it improves If S/o depression it is the other component of BP1 & I will refer you to psy.

Therapeutic Tips 



    

A key feature of bipolar disorder is recurrent nonadherence to medication; including the patient in decision-making, together with psychoeducation, promotes a strong therapeutic alliance and enhances medication adherence. Patients taking lithium need to maintain their usual salt and caffeine intake and monitor fluid intake and output, making adjustments in the event of unexpected losses due to vomiting or diarrhea. During acute manic episodes, patients may exhibit increased tolerance to lithium. Advise patients taking antipsychotics about antipsychotic-associated body temperature dysregulation and strategies to prevent heat stroke (e.g., hydration, sun protection). For lithium-associated cognitive impairment, check lithium level and thyroid function. Lowering the dose or using a slow-release formulation may improve cognitive function. Patients who experience tremor while taking lithium may benefit from elimination of dietary caffeine, lithium dose reduction or addition of a beta-blocker such as propranolol or atenolol. Patients who experience diarrhea while taking slow-release lithium preparations may fare better with immediate-release formulations,17 particularly the oral liquid citrate salt.18

Back to Content

Please note that this is only a draft version based on several sources, including: Dr. Basel Mohasen’s lectures, Toronto Notes, Therapeutic Choices and others. Edited and organized for the sake of all attendances of the Canadian Osce Exams: NAC OSCE and MCCQE2. by: Dr. Merlyn D’Souza and Dr. Zeev Gross, Spring 2011. Page 224

21/2/2011

Introduction to physical exam: “... If it is after history taking. “Thank you for the information. Now I’ll do some physical exam...hopefully towards to the end we’ll reach a working plan...” During the physical exam – talk aloud. Verbalize everything. Don’t fail to drape the patient. Doing after vitals and general inspection. Lack of empathy (Warm hands and stethoscope and try it on your arm). You warn the patient before the exam, but also before any manoeuvre. If there is painful area – don’t repeat it. If the patient having 34 physical exams, and 11 management stations (ER): Physical Exam: 1. CVS 1.1. Essential Htn for the last 30y, 65 yo for f/u 10m 1.2. 35yo dgn recently with Htn, do focused physical exam (CVS) 5m 1.3. 25yo recently HTN, relevant physical exam 5m 1.4. 60yo, Pain in calf muscles – history and physical 1.5. 35yo palpitations, history and physical. 10m 1.6. 30-40yo, Cardiac murmur. 5m. 1.7. 70yo A surgery 3d ago doesn’t pass urine 4hours – 10m (do JVP and vitals) 1.8. 50yo SOB, 3d ago had surgery, 5m focus physical exam (emboli) 1.9. Car accident 24 ago, SOB, P/E 5m (fracture, fat emboli)) 1.10. SOB for the last, surgery 3d ago, 10m 1.11. History of heart failure for the last 10y, 3d ago SOB, 10m physical exam 2. RS 2.1. Cough, for the last 3d, focused p/e 5m 2.2. Female Hx breast cancer, mastectomy, chemotherapy and radiation 5y ago, a week cough or SOB (Pulm. Fibrosis) 2.3. Coughing blood 1w, 67yo, history and p/e 10m 3. GI 3.1. Lower abdominal pain last 24hr, 22 yof, 5m focused p/e 3.2. 35 yof, came to the ER abdom. Pain 2h, 3.3. 22 yof Hx of Crohn, abdominal pain 24hr, 5m focused p/e 3.4. 30 yom abdominal pain, 5m focused 3.5. 61 yom Hx alcoholic patient vomits blood, 5m p/e 3.6. 25yof, epistaxis, bruising in skin, hematologic exam (ITP) 4. Neuro 4.1. HIV positive, headache for the last week – do cranial nerve exam (wear gloves) 4.2. 40 yo, Difficulty in vision, Hx and p/e 10m 4.3. Crooked face (Bells palsy) - hearing and than 7th nerve, 5m focused p/e 4.4. Weakness in the Rt. Or Lt. Hand – see the power, reflexes and tone, history and physical 10m 4.5. Diabetic foot – do physical exam, 5m (monofilament test – 10m exam) 4.6. Unconcious – do neurologic exam for , 5m Please note that this is only a draft version based on several sources, including: Dr. Basel Mohasen’s lectures, Toronto Notes, Therapeutic Choices and others. Edited and organized for the sake of all attendances of the Canadian Osce Exams: NAC OSCE and MCCQE2. by: Dr. Merlyn D’Souza and Dr. Zeev Gross, Spring 2011. Page 225

4.7. Shakeness in his Rt. Hand (Parkinson) – 5m 4.8. Back pain 5. M/S – all joints except elbow. 5.1. Neck (level of the lesion) 5.2. Shoulder pain 5.3. Hand 5.3.1.Laceration in the wrist 5.3.2.CTS 5.4. 35 yom – Hip pain (gonorrhoea), otherwise 5.5. Knee – Osgood Schletter, Chondromalacia patella and osteoarthritis 5.6. AP cruciate ligament, 5.7. Ankle – counsel patient. There is no fracture or rupture of ligaments. 10m 5.8. Back pain 5.8.1.Acute (3d ago) 5.8.2.Acute superimposed on chronic (fracture on metastasis) 5.8.3.Chronic back pain (young – Ankylosing spondylitis, old – spinal stenosis or osteoarthritis) Intro Vitals GI (I, A, Per, PS-PD, ST), DRE, Pelvic exam for females RS (I, Pal, Per, Aus, ST) CVS (I, Pal, A, ST), PV exam and JVP in the neck MS (I, Pal (Temp, Tenderness, Crepitus), ROM, ST) one below and one above joint. Neuro (I and orientation, CN, U and LE, Coord., Cortical sensation) InsBulk/PulpTonePowerReflexesSensation SEADS (Swelling, Erythema, Atrophy, Deformity, Scars)

Vitals: BP, HR, Temp, RR (“Based on the vital patient is stable I am going to do...) If there are no vitals you will say “I am going to start my physical exam by measuring the vitals by taking BP, pulse, temp...) If there are only 3 out of the 4 parameters – “Before I proceed I’d like to know what is the temperature...” Weight and height in pounds and inches. (5 feet is 150cm, 6 feet is 180, 5,6 is 165cm) General inspection: lying down comfortably, no signs of distress Specific inspection: SEADS for each joint (Is it OK for you to lower your gown... ) Neck – no scars, erythema, atrophy, + muscle contractions; Normal cervical and thoracic curvatures. Back – no SEADS. From side – normal cervical, thoracic and lumbar curvatures. Shoulders – both shoulders are symmetrical, clavicles deltoid and scapulae are in the same level and angle. Hand – SEADS + thenar and hypothenar muscles. Hip – I’d like to have full exposure. Hip joints are deeply seated joints – I am looking for any obvious SEADS and gluteal folds on the same level, and mentioning the lumbar curvature.

Back to Content Please note that this is only a draft version based on several sources, including: Dr. Basel Mohasen’s lectures, Toronto Notes, Therapeutic Choices and others. Edited and organized for the sake of all attendances of the Canadian Osce Exams: NAC OSCE and MCCQE2. by: Dr. Merlyn D’Souza and Dr. Zeev Gross, Spring 2011. Page 226

Knee INSPECTION: (Stand, walk and lie down). 1. Stands up: By inspection 1. (B)knees are symmetrical 2. (B) knee jts are normally aligned 3. No genu varus or valgus. 2. ask him to WALK: & look for: 1. Gait 2. popliteal fossa.(no bulge in popliteal fossa) 3. LIES DOWN:– SEADS (B) Quadriceps muscles are in the same bulk. Ankle – SEADS, no open fracture no bruises. The last thing in the ankle is the gait. Gait – do in every joint besides shoulder, arm and hand. TTC (Temp, Tenderness, Crepitus) If the patella is the same temp as the rest of the knee – there is inflam. Both patellae are the same temp. And colder as the rest of the leg. Quadriceps, suprapatellar pouch, patella (press and swing) – there is no signs consistent with chondromalacia patella, along the ...press on the collateral ligament, press to the back, when bending the knee: Up, Down, and In. Crepitus – no crepitus. Effusion – Bulging sign and milky test. Bend your knee all the way. Normal or limited flexion. Can you push against my hand and pull. ST – cruciate ligaments, medial/lateral, and anterior posterior. Medial and lateral collateral ligament – varus and valgus stress test. McMeri test for the meniscus. Examine the other knee, pulse (popliteal). Examine above and below the knee (just mention it). Patrick’s test (sacroiliac joint) and Thomas’ test (fist in the lumbar region – in case of osteoarthritis they need to do flexion by increasing their lordosis and feel less pressure).

Please note that this is only a draft version based on several sources, including: Dr. Basel Mohasen’s lectures, Toronto Notes, Therapeutic Choices and others. Edited and organized for the sake of all attendances of the Canadian Osce Exams: NAC OSCE and MCCQE2. by: Dr. Merlyn D’Souza and Dr. Zeev Gross, Spring 2011. Page 227

KNEE Intro: Good afternoon Mr XXX as I understand you’ve a pain in your (R) knee for the next few minutes I will be examining you,& if you feel any pain please let me know. I will also be reporting my findings to the examiner. Is that Okay with you? Can I proceed? Can I get the vitals please? On General examination: Pt sitting comfortably in no obvious distress Mr xxxx Can you please stand up? Can you please hold up your gown? By inspection 1. (B)knees are symmetrical 2. (B) knee jts are normally aligned 3. No genu varus or valgus. Can you please WALK: & look for: 1. Gait 2. popliteal fossa.(no bulge in popliteal fossa) Thank you, Could you please turn around walk back & lie down, I’m going to drape you LIES DOWN:– O/Inspection: 1. SEADS 2. (B) quadriceps muscles are in the same bulk I’m Going to feel your knee PALPATION: TTC (Temp, Tenderness, Crepitus) 1. Both patellae are the same temp & colder as the rest of the knee. 2. (B) Knees are symmetrical & there is no increase in temperature 3. (B)Quadriceps are normal in bulk 4. Suprapatellar pouch (N) 5. Patella (press and swing) – there is no signs consistent with chondromalacia patella, 6. Go along patellar tendon & end on Tibial tuberosity 7. No tenderness of T Tuberosity 8. press on the Medial collateral ligament, 9. press on the lateral collateral ligament 10. press to the back, for pop[liteal fossa Bend the knee: Open joint fully 11. Up for Femoral condyles 12. Down for ,Tibial condyles 13. In for the lateral & medial meniscus Please note that this is only a draft version based on several sources, including: Dr. Basel Mohasen’s lectures, Toronto Notes, Therapeutic Choices and others. Edited and organized for the sake of all attendances of the Canadian Osce Exams: NAC OSCE and MCCQE2. by: Dr. Merlyn D’Souza and Dr. Zeev Gross, Spring 2011. Page 228

Relax your knee,I’m going to move your knee & examine for

crepitus

EFFUSION:  Eliminate the suprapatellar pouch  Press on patella ------->Patellar tapping ------>Bounce = Fluid  No bounce on patellar tapping  Bulging sign  Milking Sign ROM; Can you please bend your knee all the way (N) flexion Full flexion & extension POWER: I need to examine the stability of the knee: 3 tests: 1. Medial & lateral collateral lig Varus & Valgus stress test 2. Cx ligament: Ap drawer test 3. Meniscus : Mcmurray test Examine other knee POpliteal pulse Dorsalis pedis pulse I would like to examine one Jt below & one jt above Ankle joint & back

Back to Content

Please note that this is only a draft version based on several sources, including: Dr. Basel Mohasen’s lectures, Toronto Notes, Therapeutic Choices and others. Edited and organized for the sake of all attendances of the Canadian Osce Exams: NAC OSCE and MCCQE2. by: Dr. Merlyn D’Souza and Dr. Zeev Gross, Spring 2011. Page 229

HIP JOINT Intro: Vitals G/E: Would you please stand up? Do you need help? Turn to (L) side------ go to back Ask examiner: Can I have full exposure? Can you please Roll up shirt,I’m going to look at your hip INSPECTION The hip is a deeply seated joint,however I’m looking for SEADS (B) hips are symmetrical (B) gluteal folds are same level Lumbar curvature Normal PALPATION: I’m going to feel your joint,plz inform me if you’ve pain: 1. SI jt (N) 2. Post superior Iliac spine (N) 3. Iliac crest (N) 4. Ant superior iliac spine (N) 5. Greater trochanter (N) Plz walk to the wall,do you need help? Gait (N) No limping Can you please turn & come back? When standing look for EXTENSION Trendelenbergh test Can you please lie down DRAPE I would like to continue my inspection anteriorly SEADS PALPATION: 1. Along inguinal ligament 2. Head of Femur 3. Symphysis Pubis Examiner will say (N) (Inspection & palpation done) ROM 1. Extension done when standing 2. Bend knee to abdomen as much as you can (flexion) 3. Abduction & adduction 4. Passive & active Please note that this is only a draft version based on several sources, including: Dr. Basel Mohasen’s lectures, Toronto Notes, Therapeutic Choices and others. Edited and organized for the sake of all attendances of the Canadian Osce Exams: NAC OSCE and MCCQE2. by: Dr. Merlyn D’Souza and Dr. Zeev Gross, Spring 2011. Page 230

length Discrepancy in true length Hip lesion

Patrick test Thomas test Sensory fn Knee joint & Lumbar joint

Back to Content

Please note that this is only a draft version based on several sources, including: Dr. Basel Mohasen’s lectures, Toronto Notes, Therapeutic Choices and others. Edited and organized for the sake of all attendances of the Canadian Osce Exams: NAC OSCE and MCCQE2. by: Dr. Merlyn D’Souza and Dr. Zeev Gross, Spring 2011. Page 231

SHOULDER Frozen shoulder – active and passive are limited. Rotator cuff – four muscles.  Complete tear (initiation of abduction is lost 1st 30 -60) swing the hand or tilting and doing flexion and abduction. Cannot initiate and has painful arm and dropping.(DROP ARM)  Partial tear or tendinitis or impingement with same presentation (u/s or MRI can help to differentiate between them). Painful arch – can move, but it may ease him to turn the hand in supination. The empty can test – his arms fall.  Anterior dislocation – apprehension test positive. For posterior dislocation – push the elbow backward.  Bicepts tendinitis – supination and flexion (Jargonson test). Flexing against resistance (Job’s test).  Infraspinatus and teres minor – external rotation against resistence. Internal rotation for subscapularis (lift-off test).

INTRO: Is it Ok to untie your gown & is it Okay to kep it in your lap? INSPECTION: 1. (B) shoulders are symmetrical 2. (B) Deltoids are symmetrical 3. (B) clavicles are at same angle 4. (B) Scapulae are at same level 5. No SEADS PALPATION: I’m going to feel your joint, 1. Temp (N) 2. I’m going to press your joint 3. Sternal notch NT 4. (B) Sterno clavicular joint NT 5. (B) Clavicles NT 6. Acromio clavicular jts NT I’m going to focus on (R) shoulder & then (L) shoulder Press on: 1. Acromian 2. Spine of scapula till medial aspect of scapula 3. Tip of scapula 4. Spinal process of neck 5. Insertion of Supraspinatous NT (Greater Tuberosity) 6. Glenohumeral joint NT Sulcus Sign _ve (Pull down on shoulder) CREPITUS Please note that this is only a draft version based on several sources, including: Dr. Basel Mohasen’s lectures, Toronto Notes, Therapeutic Choices and others. Edited and organized for the sake of all attendances of the Canadian Osce Exams: NAC OSCE and MCCQE2. by: Dr. Merlyn D’Souza and Dr. Zeev Gross, Spring 2011. Page 232

Relax I’m going to move your shoulder & feel the movts MOVE TO NECK & examine neck To ENSURE that shoulder pain not related to neck pain

ROM: Please put your gown back stand up & face me Would you mind copying me Full flexion & extension Push back (extension) Int rotation Cross arms--- Adduction Move to sides all the way up to the head------ Abduction Ct moving down. Hold below (No painful arc) No drop arm SPECIAL TESTS: POWER PULSE

Back to Content

Please note that this is only a draft version based on several sources, including: Dr. Basel Mohasen’s lectures, Toronto Notes, Therapeutic Choices and others. Edited and organized for the sake of all attendances of the Canadian Osce Exams: NAC OSCE and MCCQE2. by: Dr. Merlyn D’Souza and Dr. Zeev Gross, Spring 2011. Page 233

Chronic back x 6m0 INTRO: VITALS G/E Can you please stand up? CAN YOU PLZ UNTIE YOUR GOWN? INSPECTION: st

If Hx,:1 inspection of face 1. Eyes for rednes 2. Mouth for ulcers 3. Nails: No pitting/ulcers/or skin changes Look at back Curvature ---Side:Normal cervical,Thoracic & Lumbar curvatures Back: No Scoliosis SEADS PALPATION: Warm hands & tell Pt: I’m GOING TO FEEL YOUR BACK, tell me if you feel pain Feel temp Press Spinous processes individually Identify C7 Iliac crest: L 4-5 Press; Para vertebral muscles SI Jts

TIE THE GOWN BACK & Ask Pt to lie down DRAPE I’d like you to do some movts for me:

ROM 1. 2. 3. 4.

Can you touch your toes with your fingers without bending knees? Arch your back backwards without bending your knees Can you slide your arm along your thigh as low as you can? Can you cross your arms & rotate the shoulder (Fix the hip)

“Because I noticed you have restricted ROM of movements in all directions I’ll do a test called Shubert test. I will draw some lines on your back which are washable Dimples of venus – sacroiliac joints for line A 10 cm above ----- Line B 5 cm below ------Line C Try & touch toes without bending back the difference from line B it should be at least 15cm. Less than 15cm – it is restricted. The 5cm below is for control WALK to wall: Gait (N) No limping Please note that this is only a draft version based on several sources, including: Dr. Basel Mohasen’s lectures, Toronto Notes, Therapeutic Choices and others. Edited and organized for the sake of all attendances of the Canadian Osce Exams: NAC OSCE and MCCQE2. by: Dr. Merlyn D’Souza and Dr. Zeev Gross, Spring 2011. Page 234

Stand against wall

Occipital from wall test.(When there is Shubert test positive ) Stand On toes – S1, Stand on heels L5. Pitting changes in the nails, psoriatic changes.

CAN You please lie down? DRAPE SLR Patrick test Listen to his heart for Aortic Insuff. Chest expansion – measurement in max inspiration and expiration (changes should be more than 5cm).

ACUTE BACK PAIN Intro:

Always ask Pt if he prefers to lie down or stand OCD: ONSET: What were you doing at that time? Did you lift heavier than usual? Did you hear a snapping sound? Did you have to stop what you were doing? C PQRST: R: Does it move to the leg?,reach toe or thigh? Which bothers you more,The Leg or Back?

EMPATHY........... Did you try any pain killer? A &A: Lying down? Stretching? Bend/Move? U: V; ASx: 1. Weakness 2. Numbness 3. Tingling 4. Loss of balance & falls 5. Do you need to drag your foot? 6. How about Urine & Bowel symptoms: Some patients with similar condition may soil underwear 7. Numbness in buttock area? Please note that this is only a draft version based on several sources, including: Dr. Basel Mohasen’s lectures, Toronto Notes, Therapeutic Choices and others. Edited and organized for the sake of all attendances of the Canadian Osce Exams: NAC OSCE and MCCQE2. by: Dr. Merlyn D’Souza and Dr. Zeev Gross, Spring 2011. Page 235

8. H/O trauma to back? 9. Urinary: Dysuria/Flsnk pain? SOCIAL Hx: Smoke Alcohol Drugs: ......Particularly IV drug use

PE: G/E: Vitals please Can you please turn to side (so examiner can see) Can you please untie your gown? Dorso lumbar spine looks (N) curvature From Back: No scoliosis SI Joints appear (N) Tie gown

ROM: Forward flexion & extension WALK to bed & wall (Make sure that Pt does not FALL!!) Walk on heels & toes (support Pt) I’m going to raise your leg, please lie down & if it causes pain please let me know Can you please lie down? SLR SENSORY: Start with Little toe:S1 1st Web:L5 (common peroneal nerve) Medial malleolus:L4 Knee:L3 Mid thigh:L2 REFLEXES: Knee Ankle Clonus Babinski DRE Femoral stretch test End with Dorsalis pedis PULSES

Please note that this is only a draft version based on several sources, including: Dr. Basel Mohasen’s lectures, Toronto Notes, Therapeutic Choices and others. Edited and organized for the sake of all attendances of the Canadian Osce Exams: NAC OSCE and MCCQE2. by: Dr. Merlyn D’Souza and Dr. Zeev Gross, Spring 2011. Page 236

Red Flag” Symptoms/Signs in Assessment of Low Back Pain Condition Symptoms/Signs Investigations Herniated Nucleus Pulposus Positive SLR (leg pain at < 60°); MRI of lumbar spine weak dorsiflexion of ankle (L4-5) or great toe (L5-S1 or L4-5); reduced ankle reflex (L5-S1); reduced light touch in L4, L5 or S1 dermatomes of foot/leg1 Cancer

Spinal Osteomyelitis

Age > 50; previous cancer history; Positive laboratory tests unexplained weight loss; failure to (including elevated ESR, improve after 1 mo therapy2 reduced hematocrit) 2 and imaging showing erosion or blastic lesions Intravenous drug abuse; sources of Positive laboratory tests infection (e.g., skin, teeth, urinary and imaging tract, or indwelling catheter); fever; vertebral tenderness3

Spinal Fracture/Compression Fracture

Age > 50, female gender, major Positive laboratory tests trauma, pain and tenderness, and a including plain x-rays distracting painful injury;4 also consider a history of osteoporosis or corticosteroid use

Cauda Equina Syndrome

Acute urinary retention or overflow Emergency laboratory incontinence; loss of anal sphincter assessment and imaging tone/fecal incontinence; perineal numbness; change in sexual function; weakness of legs1

Please note that this is only a draft version based on several sources, including: Dr. Basel Mohasen’s lectures, Toronto Notes, Therapeutic Choices and others. Edited and organized for the sake of all attendances of the Canadian Osce Exams: NAC OSCE and MCCQE2. by: Dr. Merlyn D’Souza and Dr. Zeev Gross, Spring 2011. Page 237

Factors Adversely Affecting Prognosis of Low Back Pain Psychosocial Factors

1. 2. 3. 4. 5. 6. 7. 8. 9.

Mental Status Indicators of Significant Anxiety or Depression Duration of work absence 1. Insomnia or nightmares High levels of self-reported functional 2. Irritability disability 3. Withdrawal Self-report of extreme pain and constant 4. Panic episodes or anxiety pain in multiple body areas during the day or night History of prolonged sick-listing after 5. Persistent tearfulness previous injuries 6. Poor concentration Prior history of absenteeism 7. Inability to enjoy (anhedonia) Delays/obstacles in work re-entry 8. Poor appetite/weight loss process 9. Poor libido Patients who believe that they will 10. Thoughts that ―life is not worth never return to work living‖ Adversarial attitude toward employer Long-standing history of psychiatric distress or maladjustment

Please note that this is only a draft version based on several sources, including: Dr. Basel Mohasen’s lectures, Toronto Notes, Therapeutic Choices and others. Edited and organized for the sake of all attendances of the Canadian Osce Exams: NAC OSCE and MCCQE2. by: Dr. Merlyn D’Souza and Dr. Zeev Gross, Spring 2011. Page 238

Back to Content

Please note that this is only a draft version based on several sources, including: Dr. Basel Mohasen’s lectures, Toronto Notes, Therapeutic Choices and others. Edited and organized for the sake of all attendances of the Canadian Osce Exams: NAC OSCE and MCCQE2. by: Dr. Merlyn D’Souza and Dr. Zeev Gross, Spring 2011. Page 239

NECK EXAM INTRO: Vitals Pt stable G/E:

INSPECTION: I’d like to take a look at your back, can you please untie your gown? (N) Cxal curvature---Look from side look from back SEADS

PALPATION: I’m going to feel 1. (N) Temp 2. Press along individual spinous proceses (C1 to C7) 3. P Vertebral muscles 4. Trapezius 5. Sternocleido mastoids 6. Mastoid process 7. LN 8. Thyroid (ask the patient to swallow) ROM: I’m going to examine ROM

Copy me, 1. 2. 3. 4. 5. 6. 7.

Touch chest to chin-----> Flexion Look at ceiling -------> Extension Turn to R/L Rotation Touch shoulder to ear ---- R & L Lateral flexion Check Streno Cleido mastoid by pressing against my hand & push to back (? Not done!) Neck pain not associated with muscle spasm Can you cough? ------ “No neck pain with Valsava’s manoeuvre”

Part of my exam is to check your UppExt: Can you roll up your sleeves? INSPECTION: 1. (B) U extremities are symmetrical 2. No abnormal posture or contracture 3. Bulk is symmetrical PALPATION: See & feel deltoids, biceps, Triceps, forearm, Thenar & hypothenar muscles TONE: WRIST: No cogwheel rigidity Elbow No Pb pipe Please note that this is only a draft version based on several sources, including: Dr. Basel Mohasen’s lectures, Toronto Notes, Therapeutic Choices and others. Edited and organized for the sake of all attendances of the Canadian Osce Exams: NAC OSCE and MCCQE2. by: Dr. Merlyn D’Souza and Dr. Zeev Gross, Spring 2011. Page 240

No clasp knife rigidity SENSORY: C6 C7 C8 C4 -----REFLEXES POWER: In U/E Deltoid Biceps Fan fingers Power of thumb SPECIAL TEST: Spurling test Ask Pt to stand: Check Clonus & gait CNErve exam

Back to Content

Please note that this is only a draft version based on several sources, including: Dr. Basel Mohasen’s lectures, Toronto Notes, Therapeutic Choices and others. Edited and organized for the sake of all attendances of the Canadian Osce Exams: NAC OSCE and MCCQE2. by: Dr. Merlyn D’Souza and Dr. Zeev Gross, Spring 2011. Page 241

HAND Laceration Hx:

1. AMPLE + Tetanus 2. Mood 3. Handedness (occupation : can affect if Pianist, Speech therapist, Plastic surgeon) 4. X ray 5. 5. 6. Irrigate with NS 7. Antibiotic prophylaxis 8. NPO INTRO:: As pt has an injury,I would like to get gloves for protection Greet Pt & ask for vitals G/E; Remove bandage & describe the wound: Position: wound on palmar aspect: 3 cm in length/2mm width/depth cannot be assessed 5-10 cm proximal to wrist on Volar aspect No active bleeding/No oozing/Margins clear & not elevated (B) hands are symmetrical SEADS Colour similar I’m going to FEEL: Temp (B) hands is normal (N) Capillary refill I’m going to feel your hands to see if there is damage to the arteries (N) radial artery & ulnar SENSATIONS: Lt touch Ulnar/Median/Radius Tenderness to PALPATION: Distal radius/Styloid process/distal part of ulna & styloid process/base of thumb Press carpal bones & metacarpal bones ROM: try to do on table & not move elbow Ulnar deviation Radial deviation MPOTOR FN OF MEDIAN N: OK Sign Ulnar N :Able to hold peice of paper betn Adducted finger & resist pulling Please note that this is only a draft version based on several sources, including: Dr. Basel Mohasen’s lectures, Toronto Notes, Therapeutic Choices and others. Edited and organized for the sake of all attendances of the Canadian Osce Exams: NAC OSCE and MCCQE2. by: Dr. Merlyn D’Souza and Dr. Zeev Gross, Spring 2011. Page 242

RADIAL N: Extend thumb Thumbs up THUMB: Make a fist & fan out fingers Can you touch ........your thumb to the tip of your little finger? (flexion) Take it all the way to other side ? (extension) Point to ceiling (Abduction) Put close to your hand?(Adduction) Touch thumb to tips of fingers? (opposition) FLEXION: Can you bend your fingers one by one?----- Flexor digitorum profondus Flexor Doigitorum superficilias

Back to Content

Please note that this is only a draft version based on several sources, including: Dr. Basel Mohasen’s lectures, Toronto Notes, Therapeutic Choices and others. Edited and organized for the sake of all attendances of the Canadian Osce Exams: NAC OSCE and MCCQE2. by: Dr. Merlyn D’Souza and Dr. Zeev Gross, Spring 2011. Page 243

CTS Pain in (R) wristy x 2 wks: Hx & PE OCD PRTY UV A&A ASx CSx D/d: 1. CTS 2. Spinal stenosis/OA/Cervical disc herniation 3. TIA 4. Thoracic outlet syndrome OCD: O: C: How often? Daily? Since when daily? Before that?

At Night? D: How long each attack? What brings these attacks? What relieves it? What do you do for a living? PQRST: P: Can you show me where it is? Q; S: U; V: A & A: Movts/Medications/Repeated movts Local Sx: Swelling/Numbness/Weakness/Other hand/Leg/Bladder & bowel disturbances CSx: AETIO:

I’ve to ask you qns as to the presence of any condt that might have caused this: 1. 2. 3. 4. 5. 6. D/d:

Hx & Sx of DM: Hx & Sx of Hypothyroid Hx & Sx of Acromegaly Trauma Fall HX of RA

Please note that this is only a draft version based on several sources, including: Dr. Basel Mohasen’s lectures, Toronto Notes, Therapeutic Choices and others. Edited and organized for the sake of all attendances of the Canadian Osce Exams: NAC OSCE and MCCQE2. by: Dr. Merlyn D’Souza and Dr. Zeev Gross, Spring 2011. Page 244

Neck pain Past MH: HTN/any long term disease Social Hx;SAD Fhx: Thank You for this information,I will now proceed to the PE GE: INSPECTION: 1. (B) hands are symmetrical 2. No SEADS 3. No Bouchardfs nodes 4. No Swan neck deformity 5. (B) Thenar & Hypothenar muscles equal bulk FEEL: 1. Temp & capillary refill 2. Palpate distal part of radius of hand 3. Bulk of thenar & hypothenar muscles ROM of wrist THUMB: Power check against resistance,pu;ll up & down & pull Hook thumb ….. BICEPS: Check Power & Reflex NECK: ROM to R/o C6 SENSORY: 1. Little finger :Ulnar 2. Ring Finger:Ulnar aspect for Ulnar nerve & radialaspect to R/o median nerve 3. 2 POINT DISCRIMINATION: Only in Index finger SPECIAL TESTS: 1. Tinel’s Tap at medial aspect of wrist x3 times Ask if feels numb 2. Phalen’s sign PULSE: Radial

T Back to Content

Please note that this is only a draft version based on several sources, including: Dr. Basel Mohasen’s lectures, Toronto Notes, Therapeutic Choices and others. Edited and organized for the sake of all attendances of the Canadian Osce Exams: NAC OSCE and MCCQE2. by: Dr. Merlyn D’Souza and Dr. Zeev Gross, Spring 2011. Page 245

TREMORS PARKINSONISM INTRO: Vitals: G/E: Pt is sitting There is an obvious tremor in (R) hand (N) elbow No tremor in shoulder Ask patient to count from10 to 1 backwards: & observe the tremor....  Tremor does not disappear on mental activity but increases, which is consistent with Parkinsonism, & R/O Anxiety related tremor Please extend arms & fingers:  No fine tremors R/O Thyroid disease  No flapping tremors R/O Liver disease  Can you touch Finger to my finger & then to your nose? No intention tremor R/O Cerebellar disease There is no dysdiadokinesia Pt has a limited facial expression Limited eye blinking No drooling INSPECTION: Tremors in (R) hand, which are pill rolling & involve the (R) arm Pt does not have tremors in (L) hand, arm & shoulder NO head nodding I want to examine the TONE:  Cog wheel  Pb pipe  Clasp knife Ask pt to please stand There is difficulty in initiating movt Stooped posture Decreased arm span Festinant gait Turns in block Ask Pt to say: British Constitution (N) articulation Ask Pt to write; Micrographia I want to check for orthostatic hypotension Difficulty in rapid alternating movts: Please note that this is only a draft version based on several sources, including: Dr. Basel Mohasen’s lectures, Toronto Notes, Therapeutic Choices and others. Edited and organized for the sake of all attendances of the Canadian Osce Exams: NAC OSCE and MCCQE2. by: Dr. Merlyn D’Souza and Dr. Zeev Gross, Spring 2011. Page 246

1. 2. 3. I would like to arrange for a MMSE which can happen later.

Back to Content

Please note that this is only a draft version based on several sources, including: Dr. Basel Mohasen’s lectures, Toronto Notes, Therapeutic Choices and others. Edited and organized for the sake of all attendances of the Canadian Osce Exams: NAC OSCE and MCCQE2. by: Dr. Merlyn D’Souza and Dr. Zeev Gross, Spring 2011. Page 247

st

HIV Pt with HA/PE (Cranial nerve exam)

1 nerve. (Coffee and ammonia). I’m going to skip the first nerve. I’ll ask the patient if he has any difficulty smelling.

2nd nerve.: OPTIC NERVE (5 tests): ACUITY: Ask Pt for best vision or if he wears EYE GLASSES Hold Snellen’s chart with (R) hand & cover Lt. eye. Choose a mid-line, jump two lines below, and finally last line. COLOUR VISION:, then the other eye, change eyes, ask colour first in a reversed order and if he sees in the same intensity. Go straight to last line and ask to read backwards. VISUAL FIELDS: (DDx one eye blindness, bitemporal and homonymous hemianopia) PUPILLARY REACTION: I am going to shine the light in your eyes it might might bother you: first shine at the (R) Look at the (R). Eye, second shine in rt. Look at left side, 2 shine light in eyes & see pupillary reaction : 2-3nerve (2- afferent, 3-efferent) FUNDOSCOPY: verbalize (DM: microaneurysms, cotton wool spots, neovascularisation; Htn: flame hemorrhage, disc edema, nipping of veins)

3rd, 4th and 6th I am going to examine the nerves which cause movement of the eyes. INSPECTION: (B) eyes are symmetrical, No deviation. No head tilting (4th nerve), No ptosis (3rd nerve), No nystagmus. Tell patient to look at the tip of pen and follow with eyes and when you see double vision please tell me. Start from middle and create an H. Then go to centre and check conversion. “Normal extraoccular vision, no limitation, no nystagmus.”

5th nerve: motor and then sensory. INSPECTION: No atrophy in temporal and masseter area. I am going to examineSENSORY:. This is a piece of cotton, I’ll put in on your chest – this is how it feels. Now I am going to touch different parts in your face while your eyes are closed. Whenever you feel it touches you tell me. Then ask him if it is the same feeling. MOTOR: clench teeth and relax twice. Feel the bulk of the temporal and masseter – they should be similar bulk. Can you push your jaw against my hand? Please note that this is only a draft version based on several sources, including: Dr. Basel Mohasen’s lectures, Toronto Notes, Therapeutic Choices and others. Edited and organized for the sake of all attendances of the Canadian Osce Exams: NAC OSCE and MCCQE2. by: Dr. Merlyn D’Souza and Dr. Zeev Gross, Spring 2011. Page 248

7th is mostly motor. Sensory for the tongue (anterior 2/3). Corneal reflex efferent limb. INSPECTION: Face symmetrical, Normal nasoliable fold, No drooling, No deviation of angle of mouth. Now copy me: raise your eyebrows, frown, close your eyes and don’t let me open them, puff cheeks and don’t let me blow out, show your teeth, and whistle. I would like to check corneal reflex in the eye

8th nerve. I am going to whisper words in your ear. Repeat after me (“horse” and “house”).

9th and 10th 1. 2. 3. 4.

Normal voice, no hoarseness. Swallow for me – swallowing is normal. Say AHAA – soft palate symmetrical, uvula is central. Gag reflex I’d like to do.

11th nerve – shrug your shoulders (“Normal trapezius”). Turn your head to the right against resistant and feel the bulk of the sternocleidomastoid.

12th nerve – no atrophy/ fasciculation of the tongue no deviation of the tongue. Wiggle your tongue left and right.

Back to Content

Please note that this is only a draft version based on several sources, including: Dr. Basel Mohasen’s lectures, Toronto Notes, Therapeutic Choices and others. Edited and organized for the sake of all attendances of the Canadian Osce Exams: NAC OSCE and MCCQE2. by: Dr. Merlyn D’Souza and Dr. Zeev Gross, Spring 2011. Page 249

Unconcious Patient INTRO: Hello, Mr….DO you hear me. If you hear me open your eyes. I am Dr. … one the physicians working in the clinic. FIRST CHECK PUPILS: Pupillary reaction – pupils are round and symmetrical and reactive. Not dilated or constricted. There is no pupillary discrepancy. If one is reacting & the other not reacting – call neuro. If you hear me – can you move your eyes up and down? “There is no locked-in syndrome”. Vitals (Cushing triad absent; If the patient has fever we will verbalize it). GCS Cranial nerve: 1. 2-fundoscopy, 2. 2-3 – pupillary reflexion, 3. 3-4-6 – eye deviation, 4. 5-7 – corneal reflex, 5. 7 – inspection of face symmetry, 6. 9-10 – gag reflex

Upper extremity: Inspection (symmetrical, normal position, no movement, no contractures), tone, reflexes (biceps, triceps, brachioradialis)

Lower extremity: Inspection (symmetrical, normal position, no movement, no contractures), tone, reflexes (knee, ankle, Babinski)

Meningeal signs Neck stiffness, Kernigs, Brudinsky Babinsky

Special test: Caloric test, Dolls eyes

Back to Content

Please note that this is only a draft version based on several sources, including: Dr. Basel Mohasen’s lectures, Toronto Notes, Therapeutic Choices and others. Edited and organized for the sake of all attendances of the Canadian Osce Exams: NAC OSCE and MCCQE2. by: Dr. Merlyn D’Souza and Dr. Zeev Gross, Spring 2011. Page 250

ER Trauma Hx + Transition A B C D AMPLE Head to Toe Orders

Non-trauma ABCD History of present illness CC PQRST (Head to toe) ASx Α PMHx (Risk Fcts) Focused P/E Orders

In ER don’t be comfortable till after primary survey and IV lines. If non-trauma patient in ER – you do primary survey (shortened), more time on history and focus on CC. Case of Trauma I’d like to initiate ATLS protocol and I’d like protection to my team and myself (gown, goggles, mask, and gloves). When walk to patient ask the nurse: How is the patient doing? What was done till now? If not wearing collar – tell nurse to fix the head, tell patient not to move “we need to fix your neck collar for your neck”. Take a small history: “how do you feel right now?” (to see if he can talk). If complains of severe pain (empathy: I can see you are in pain, please bear me with me for a few minutes, as soon as I can I will give you a pain killer. At the moment I want to make sure you are stable for that reason, I am going to give some orders to the nurse, and as soon as I am done I’ll ask you more questions).

A - Airway Please open your mouth. Mouth clear, the Flip your tongue: “there are no clots, foreign bodies, broken teeth, and Patient is talking to me – that mean airways are patent.

B Please note that this is only a draft version based on several sources, including: Dr. Basel Mohasen’s lectures, Toronto Notes, Therapeutic Choices and others. Edited and organized for the sake of all attendances of the Canadian Osce Exams: NAC OSCE and MCCQE2. by: Dr. Merlyn D’Souza and Dr. Zeev Gross, Spring 2011. Page 251

Can I get the saturation?” Give oxygen. If saturation is 95% than you give oxygen – ask if the saturation improved. Oxygen + saturation is a part of B. Open the neck collar and look for trachea deviation, Jugular vein. Trachea J Veins Air Entry Heart Sounds Normal Central Bilateral S1, S2 Tension Pneum. Opposite side Increased Decreased same S1, S2 side Hemothorax Opposite side Low/Normal Decreased same S1, S2 side Cardiac Central Increase Bilateral Muffled Tamponade Pneumothorax – large bore needle in 2nd intercostals, midclavicular line, upper margin of the third rib. Hemothorax – insert chest tube in 5th intercostals space mid-axillary line. Ask nurse “how much blood” (If > 1.5litre – ask for thoracic surgeon, also if greater than 800cc in 4 hours). If less – monitor. Cardiac tamponade – ask for thoracic surgeon. Pericardiocentesis – needle in mid-xyphoid 45 degrees towards the tip of the scapula and look for blood. Continuous ECG.

C Vitals (every 10 minutes), and blood orders. “I’d like to get the vitals. Comment if hypotensive tachycardia. I’d like to start 2 IV lines 16G in both anti-cubital fossa. 2 litre bolus Ringer lactate in one side, and from the other take blood. If no improvement after 2 litres – give another bolus. If deterioration in vitals – give blood. Finger prick glucose; BLOOD for:  CBC, Lytes, Group, Cross match,  INR, PTT, LFT, BUN, Cr,  Toxoc screen, Alcohol level. If unable to get the IV line – insert intraosseous line (IO).  Order 6 units of blood: 2 O positive for male or negative for female in reproductive age and add 4 units of cross matched blood (pRBC).  Continuous ECG±cardiac enzymes (troponine, cpk-mb), Ask the change in vitals again & results of blood glucose. Ask if 2litres were given. If stable – OK. If UNSTABLE:– look for source of bleeding. Start with ABDOMEN: Please note that this is only a draft version based on several sources, including: Dr. Basel Mohasen’s lectures, Toronto Notes, Therapeutic Choices and others. Edited and organized for the sake of all attendances of the Canadian Osce Exams: NAC OSCE and MCCQE2. by: Dr. Merlyn D’Souza and Dr. Zeev Gross, Spring 2011. Page 252

INSPECTION: listen, and palpation. If bruising – ask for surgical consult stat. If not available ask for FAST. If technician is not available – then DPL (Diagnostic Peritoneal Lavage). Then go for the PELVIS:– I am going to press on hips to see if there is any pain. If complain of pain tell “I suspect pelvic fracture” Wrap sheet and call ortho stat. Look at the LOWER EXTREMITY: No internal/external rotation, feel there is any pain, difference in the legs. If you suspect fracture ask for Thomas splint and check the pulse before and after. Log roll – check for spinal process and DRE.

D D1- Deficit – Gross Neurological: Shine light to both eyes “Both pupils normal size reacting to light. Can you squeeze my finger, wiggle your toes. Touch his sides of both upper and lower limbs – can you feel my touch. “Patient is grossly neurologically intact.” If unconscious – check papillary reaction and assess GCS.

D2 – universal antidote Thiamine, Glucose,& Naloxone. D3 – specific antidote.

AMPLE Allergy Medication PMHx Last meal, Last tetanus shot, LMP (if female) Event – describe the event (Rear end, T bone, Head on); Were you driver/passenger/alone? Have you had any head trauma? Do you remember anything before or after the event? Do you have nausea/vomiting/headache. Head-Toe examination Orders

Back to Content

Please note that this is only a draft version based on several sources, including: Dr. Basel Mohasen’s lectures, Toronto Notes, Therapeutic Choices and others. Edited and organized for the sake of all attendances of the Canadian Osce Exams: NAC OSCE and MCCQE2. by: Dr. Merlyn D’Souza and Dr. Zeev Gross, Spring 2011. Page 253

Hypertension/Secondary As I understand you came here today because you were diagnosed with increased blood pressure. I’ll do a physical exam on you. Can I get the VITALS:please? 1. Patient have (B)systolic & diastolic blood pressure raised. 2. Patient does not have tachycardia, r/o pheochromocytoma and thyrotoxicosis. 3. Patient does not have bradycardia – r/o hypothyroidism. 4. I’d like to compare BP in upper and lower extremity to r/o coarctation of aorta. 5. I’d like to r/o orthostatic hypotension for pheochromocytoma. 6. Check orientation: Time, Place, and Person

On general examination: Patient sitting comfortably  No sign of truncal obesity  No cervical fat pad.  Face is symmetrical  No moon like face EYES:  Normal eye brow,  No puffiness around the eyes  No exophthalmus,  Please Follow my finger – there is no lid-lag or lid retraction.  Sclera for anemia or pallor.  No xanthelasma or arcus senilis  On fundoscopy there are no signs of Htn.  No loss of visual fields (acromegally). NOSE:  Nose OK (septal perforation in cocaine abuse). HANDS:  Symmetrical,  skin normal not dry or moist, no sign of drug abuse (needle puncture).  Normal capillary refill,  No clubbing  No nicotine staining.  Please stretch your hands – no fine tremor.  Pulse – regular, normal volume and contour.  Compare both pulses. (When lies down – take radio-femoral delay.)  Abduct shoulders to check proximal weakness for Cushings. NECK:  feel thyroid, swallow,  ask patient to lie down, put bed at 45 degrees and ask for JVP.

 Check for carotid bruit (first listen than palpate) Please note that this is only a draft version based on several sources, including: Dr. Basel Mohasen’s lectures, Toronto Notes, Therapeutic Choices and others. Edited and organized for the sake of all attendances of the Canadian Osce Exams: NAC OSCE and MCCQE2. by: Dr. Merlyn D’Souza and Dr. Zeev Gross, Spring 2011. Page 254

BACK:  Listen between scapula for collateral circulation and bruit (COA)  Base of lungs for creps and heart failure.  Press on sacrum and ankle for edema. CHEST: (lies down, please lower your gown)  chest is symmetrical.  No obvious pulsation.  I am going to feel.  Feel for apex beat, fine and identified PMI position and size, not enlarged not displaced, not sustained.  No parasternal heave.  Listen to mitral area – normal S1, S2  Move to bell and lie on side: no S3 and S4 ABDOMEN:  abdomen non-distended, symmetrical, no pulsation, no striae, no caffe au lait, no obvious masses.  I am going to listen to the abdomen. 2 inches above umbilicus is the aortic bruit, renal is 2 inches on the same level, and the iliac are 2 inches below on 45 degree below.  Tap, feel dullness in renal area for masses. No supra-renal masses.  Femoral-radial delay,  No peripheral edema. Neuro: Kneel on chair and do ankle reflex a Quick neuro.

Back to Content

Please note that this is only a draft version based on several sources, including: Dr. Basel Mohasen’s lectures, Toronto Notes, Therapeutic Choices and others. Edited and organized for the sake of all attendances of the Canadian Osce Exams: NAC OSCE and MCCQE2. by: Dr. Merlyn D’Souza and Dr. Zeev Gross, Spring 2011. Page 255

P/E of CVS INTRO: VITALS: (Thank you for the Vitals) Both Sys and Dia BP are elevated, HR is normal. Orientation: time, place, person G/ E:no obvious obesity HEAD: Eyes - ±pallor/arcus senalis, no xanthelasma Mouth: no dehydration FUNDOSCOPIC: exam HAND:Temp/Capillary refill/Clubbing/Nicotine stain Pulse: regular/normal volume and contour NECK: at 45o look for JVP, listen to carotids one by one, then palpate carotids CHEST: ask to lower gown INSPECTION: Sit and look for pulsation PALPATION: PMI Feel apex/thrills/heaves AUSCULTATION: Aortic/tricuspid/MV, lay patient on left side, no S4 Sit up and lean forward, breathe out and hold it – listen if there is aortic regurgitation (?) Listen to base of lung Press on sacrum for edema.

Ask patient to lie down on bed ABDOMEN: listen to bruit (aortic, renal and iliac) LOWER EXTREMITY: temp, capillary refill, dorsalis pedis and peripheral edema Three places you look for orientation:  Volume status  Malignant hypertension  Hypoxia and SOB

Back to Content

Please note that this is only a draft version based on several sources, including: Dr. Basel Mohasen’s lectures, Toronto Notes, Therapeutic Choices and others. Edited and organized for the sake of all attendances of the Canadian Osce Exams: NAC OSCE and MCCQE2. by: Dr. Merlyn D’Souza and Dr. Zeev Gross, Spring 2011. Page 256

Volume Status 79 Hip replacement 3d ago, nurse asked to come and see, not passed urine for four hours. Do Volume status exam. INTRO: VITALS: (and mention that BP should be done twice – while lying and sitting) After measuring BP in one position, there are two minutes before you measure the second position, meanwhile you do (the cuff of the BP should be on the same level of the heart). Width of cuff is equal to 40% of circumference of arm. 1. ORIENTATION:Time, Place, and Person. 2. Listen to the base of the lung. 3. Look for sacral edema. 4. Look for sclera for pallor. 5. Mouth: open and look for dehydration. Flip tongue for central cyanosis. 6. Look for hands, skin (moist and dry). 7. Capillary refill – should be less than 2s. Measure HR again and BP. If there is no increase in pulse more than 20bpm and no decrease in SBP more than 20 or DBP more than 10 – there is no orthostatic hypotension. If one of them is positive – Orthostatic hypotension. Put patient at 45o to do JVP. Press on base to see if JV disappears. Measure. Take deep breath and hold – Kussmaul Sign absent. Untie the shirt and do hepato-jugular reflex. INSPECTION – S3 and S4 and all cardiac exam. ABDOMEN: percussion at suprapubic to see if bladder is full. Pedal edema, than look at examiner and ask for: input-output chart & weight charts. (If there is cathter:) I’d like to make sure that the catheter is not kinking.

Back to Content

Please note that this is only a draft version based on several sources, including: Dr. Basel Mohasen’s lectures, Toronto Notes, Therapeutic Choices and others. Edited and organized for the sake of all attendances of the Canadian Osce Exams: NAC OSCE and MCCQE2. by: Dr. Merlyn D’Souza and Dr. Zeev Gross, Spring 2011. Page 257

PVD Pain in calf for three months. Vitals (“patient is stable”), if patient is wearing socks ask him to remove them. INSPECTION: (B) Feet: SEADS+3:  Normal hair distribution  Skin non-tight and shiny  No hypertrophy of nails PALPATION: (I will feel your feet):  Peripheral temperature  Capillary refill  Pulse (both sides): DP, PT, Pop. & Femoral. Drape the patient and listen to his abdomen for bruits. Feet – LIGHT TOUCH SENSATION: Bergers test – raise legs for two minutes – any you feel tingling/numbness tell me. After two minutes tell: “no pallor, pain, numbness, tingling.” Sit up and dangle his feet – “No rubour on depandance.” I would like to do ANKLE BRACHIAL INDEX

Back to Content

Please note that this is only a draft version based on several sources, including: Dr. Basel Mohasen’s lectures, Toronto Notes, Therapeutic Choices and others. Edited and organized for the sake of all attendances of the Canadian Osce Exams: NAC OSCE and MCCQE2. by: Dr. Merlyn D’Souza and Dr. Zeev Gross, Spring 2011. Page 258

Diabetic Foot Intro (As I understand you are here today cause you have DM for 2 y and ulcers in your Rt leg. I have to do a P/E) Vitals – stable. Drape and remove socks. INSPECTION: Look at sole of foot. Ulcer – 3cm in diameter, round, margin not elevated, no active bleeding or oozing, located at base of 1st metatarsal. I am going to look for other ulcers at the base of the toes (Between medial and lateral maleolus.) Check SEADS + 3. PALPATION: Temp and capillary refill. Shift to NEUROLOGICAL EXAMINATION: LIGHT TOUCH SENSATION in glove and stocking manner. Start with big toe and go to level – and than up and down until finding the right level. Light touch absent or decreased at a level to distal point. For example – above wrist. And then say to the patient: “Thank you and open eyes.” POSITION SENSE: close eyes and move the big toe up and down five to six times. “Thank you. Open your eyes.” VIBRATION SENSE: tuning fork – put on sternum to show how it feels, then put it on the 1st interphalangeal joint. If doesn’t feel – vibration sense absent. Start with first joint, and second joint (you check vibration also for (1) medial maleolus, (2) tibial tuberosity, anterior superior iliac spine, sternum, chin, and forhead). (1) and (2) are for spinal injury. ANKLE REFLEX: MONOFILAMENT TEST: press on sole or foot. Feel or no-feel. Increase the pressure & bend the monofilament. “He has lost light touch and pressure.” But if feels when pressure, say: “pressure present but light touch gone.” You check the same way in 9 points on the sole. PULSES: DP, TP Auscultation (?) and ABI (Ankle-Brachial Index)

Acute Abdomen, Physical exam Intro General inspection: the patient is lying comfortably and I see no signs of distress. Can I get the vitals please? The patient is stable, normal temperature, BP and HR. You are going to face – can I take a look at your eyes: there is no jaundice, no sign of anemia. Please open your mouth – there is no sign of dehydration and obvious ulcer in the mouth. Upper extremity: capillary refill is normal. Please note that this is only a draft version based on several sources, including: Dr. Basel Mohasen’s lectures, Toronto Notes, Therapeutic Choices and others. Edited and organized for the sake of all attendances of the Canadian Osce Exams: NAC OSCE and MCCQE2. by: Dr. Merlyn D’Souza and Dr. Zeev Gross, Spring 2011. Page 259

Abdomen (drape appropriately): please move the cover – by inspection: the abdomen is not distended, umbilical inverted, abdominal moves with breathing, no scars, no bruises. Ask patient to look aside and cough twice (once look at his face to see for cough tenderness and then for abdominal bulging). Now I am going to listen (warm the sthetoscope): “Normal bowel sounds, no bruits – aorta, renal, and iliac.” Percussion: I am going to tap – show me where it pains. First tap away from painful area, than tap over the 9 areas – the painful area last. Pulpation: I am going to feel – no tender in epigastric/Rt. And Lt. Hypochondral/Rt. And Lt. Iliac regions/ Umbilical / Suprapubic. Deep pulpation: I am going to apply more pressure – there is no guarding in deep pulpation, there are no obvious masses. Now I am going to feel your kidneys – there is no enlargement of your kidneys. Now I am going to do some special tests. Murphy sign Rebound tenderness. McBurny sign. Rovsing sign. Psoas sign. Obturator sign. “Please sit up. Now I am going to tap your back. There is no tenderness on costo-vertebral angle. Now listening again to the base of the lungs. There is no crepitus at the base of the lungs.” “I’d like to finish my exam by doing pelvic exam, vaginal exam for bleeding or discharge or bimanual examinations. Looking for any cervical motion tenderness, and adnexal masses.” “In DRE looking for any bleeding or haemorrhoids.” Acute on chronic abdomen (like Crohn’s Dis.) Add to the above: General inspection: moon faces, truncal obesity, redness in sclera, nails – pitting and clubbing, no skin rushes, no striae, no erythema nodosum on legs, sacroiliac joints look normal.

Back to Content

Please note that this is only a draft version based on several sources, including: Dr. Basel Mohasen’s lectures, Toronto Notes, Therapeutic Choices and others. Edited and organized for the sake of all attendances of the Canadian Osce Exams: NAC OSCE and MCCQE2. by: Dr. Merlyn D’Souza and Dr. Zeev Gross, Spring 2011. Page 260

Acute Abdomen, Physical exam Intro General inspection: the patient is lying comfortably and I see no signs of distress. Can I get the vitals please? The patient is stable, normal temperature, BP, RR and HR. You are going to face – can I take a look at your eyes: there is no jaundice, no sign of anemia. Please open your mouth – there is no sign of dehydration and obvious ulcer in the mouth. Upper extremity: capillary refill is normal. Abdomen (drape appropriately): please move the cover – by inspection: the abdomen is not distended, umbilical inverted, abdominal moves with breathing, no scars, no bruises. Ask patient to look aside and cough twice (once look at his face to see for cough tenderness and then for abdominal bulging). Now I am going to listen (warm the sthetoscope): “Normal bowel sounds, no bruits – aorta, renal, and iliac.” Percussion: I am going to tap – show me where it pains. First tap away from painful area, than tap over the 9 areas – the painful area last. Pulpation: I am going to feel – no tender in epigastric/Rt. And Lt. Hypochondral/Rt. And Lt. Iliac regions/ Umbilical / Suprapubic. Deep palpation: I am going to apply more pressure – there is no guarding in deep palpation, there are no obvious masses. Now I am going to feel your kidneys – there is no enlargement of your kidneys. Now I am going to do some special tests. Murphy sign Rebound tenderness. McBurny sign. Rovsing sign. Psoas sign. Obturator sign. “Please sit up. Now I am going to tap your back. There is no tenderness on costo-vertebral angle. Now listening again to the base of the lungs. There is no crepitus at the base of the lungs.” “I’d like to finish my exam by doing pelvic exam, vaginal exam for bleeding or discharge or bimanual examinations. Looking for any cervical motion tenderness, and adnexal masses.” “In DRE looking for any bleeding or haemorrhoids.” Acute on chronic abdomen (like Crohn’s Dis.) Add to the above: General inspection: moon faces, truncal obesity, redness in sclera, nails – pitting and clubbing, no skin rushes, no striae, no erythema nodosum on legs, sacroiliac joints look normal.

Back to Content Please note that this is only a draft version based on several sources, including: Dr. Basel Mohasen’s lectures, Toronto Notes, Therapeutic Choices and others. Edited and organized for the sake of all attendances of the Canadian Osce Exams: NAC OSCE and MCCQE2. by: Dr. Merlyn D’Souza and Dr. Zeev Gross, Spring 2011. Page 261

Hematemesis “Because it is hematemesis I’d like to initiate a ATLS protocol for me and my team, please can I can get gloves, goggles, masks, and gowns.” Intro How do you feel right now? I want to make sure you are stable and therefore I’ll give some orders to the nurse. Once you are stable I’ll ask you some questions. ABCD Vitals OCD + COCA How did it start? Forceful and retching? Did you vomit once or more? How much? Dark /bright blood? Any clots? Any smell? IMPACT±PAIN -PAIN  Liver +PAIN  GIT If No Pain: Hx: Any Hx of liver disease? Any screening for liver disease? Any bruising in body? Increase in abdominal size lately? Alcohol: how long? How much? Hx of PUD Heartburn Any nausea When was your last bowel movement? Colour? Any tarry stool/fresh blood? Any Hx of bleeding disorder? Any NSAIDs (Aspirin) – how much? How long? Why? Who prescribed? Any blood thinner? CSx (Ask for weight loss) Long term disease Physical exam: Vitals If suspected liver disease (no pain): Sclera – no yellow discoloration, pallor Enlargement of parotid glands Please note that this is only a draft version based on several sources, including: Dr. Basel Mohasen’s lectures, Toronto Notes, Therapeutic Choices and others. Edited and organized for the sake of all attendances of the Canadian Osce Exams: NAC OSCE and MCCQE2. by: Dr. Merlyn D’Souza and Dr. Zeev Gross, Spring 2011. Page 262

Mouth: Fetor hepaticus, mouth is clear no bleeding no clots Hands: no clubbing, capillary refill, no atrophy of thenar or hypothenar, no palmar erythema, no dupytren’s contraction, no flapping tremor Chest: no spider nevi, no gynecomastia, normal chest hair, no bruising Abdomen: not distanded, umbilicus normal, caput medusa, collateral veins, no bruising. Auscultation: bowel sounds normal, bruits (aorta, renal, iliac), no hepatic rub, hum, or bruit; No splenic rub, hum Tapping: four taps – general percussion, percussion for liver (upper and lower margin), spleen (Castle sign), shifting dullness DRE Testicular atrophy Peripheral edema Epigastric tenderness Gastroenterologist consult and admit to ICU Endoscopy and IV PPI

Back to Content

Please note that this is only a draft version based on several sources, including: Dr. Basel Mohasen’s lectures, Toronto Notes, Therapeutic Choices and others. Edited and organized for the sake of all attendances of the Canadian Osce Exams: NAC OSCE and MCCQE2. by: Dr. Merlyn D’Souza and Dr. Zeev Gross, Spring 2011. Page 263

Acute Abdomen – management Abdominal pain 24 hr with vomiting and diarrhea, BP 90/60, Pulse 140 Diagnosis: acute pancreatitis Intro: “As I understand... please bear with me, as your BP is low I’ll give orders to the nurse, and as soon as you become stable, I’ll give you something to relieve your pain.” A B – Vitals, Oxygen saturation C – because he is hypotensive and tachycardic you give IV fluids; Take blood to: (add amylase to the other blood work) D – Gross neurological exam Hx What happened? Pain: OCD, PQRST, AA Vomiting: how much, how many times, amount, forceful, blood/coffee ground IMPACT RF (Alcohol, Gall stones, Hypertriglyceridemia, DM, Viral infection, Medications) Recent trauma Alcohol: how much, when was last time, last drink (was it more than normal?) Hx of gall bladder disease Recent flu DM Medications CSx Hx of HTn (R/O Aortic dissection) Chest pain Cough, phlegm Flank pain Liver disease PMHx FHx SHx Vitals 2min stabilize, 4min Hx P/E Look for liver disease: sclera, tongue, and hands Abdomen: Drape No Cullens and Great Turner signs. Look for cough tenderness. Auscultate bowel sounds: no aortic/renal bruit. Feel or tap abdomen Please note that this is only a draft version based on several sources, including: Dr. Basel Mohasen’s lectures, Toronto Notes, Therapeutic Choices and others. Edited and organized for the sake of all attendances of the Canadian Osce Exams: NAC OSCE and MCCQE2. by: Dr. Merlyn D’Souza and Dr. Zeev Gross, Spring 2011. Page 264

Groing exam DRE Orders: Meperidine, NPO, NG Tube, Admit to ICU, Foley catheter, Input-output chart, Imaging: AXR, Abdominal U/S and CT, surgical consult

Back to Content

Please note that this is only a draft version based on several sources, including: Dr. Basel Mohasen’s lectures, Toronto Notes, Therapeutic Choices and others. Edited and organized for the sake of all attendances of the Canadian Osce Exams: NAC OSCE and MCCQE2. by: Dr. Merlyn D’Souza and Dr. Zeev Gross, Spring 2011. Page 265

MI – Management Hx OCD PQRST (if it is suspected to be ACS - stop at R and start primary survey) Primary Survey (If patient talks – Airway preserved, Take Oxygen saturation and start Oxygen Stat – 4L/m through nasal prongs) VITALS: Auscultation:  Air entry (N)  S1 & S2 (N) IV lines : NaCl 50ml/hr to keep line open, from the other side take blood for: Troponin, CK-MB, Cr, BUN, Lytes, CBC, INR, PTT, LFT, Toxic., Alcohol, Lipids; and finger prick for Glucose) ECG 12 leads & continous monitoring Portable X-ray (r/o dissection)

Ask about Allergy for Aspirin and Viagra (if negative)[12hrs for Viagra & 36 hrs for Cialis] Give ASA chewable (325mg)

Non-ST elevation Nitro x3 (S.L) Morphine Continue now with: PQRST AA&A How do you feel now?

Ask Hx: CVS GI (especially peptic ulcer) CSx RS DVT

ST Elevation: do not go for DDx Nitro (IV Nitro is C/I in IWMI) Morphine (5mg if ALMI, and 1mg if PWMI) VITALS: (again) Please note that this is only a draft version based on several sources, including: Dr. Basel Mohasen’s lectures, Toronto Notes, Therapeutic Choices and others. Edited and organized for the sake of all attendances of the Canadian Osce Exams: NAC OSCE and MCCQE2. by: Dr. Merlyn D’Souza and Dr. Zeev Gross, Spring 2011. Page 266

R/O Contra Indications for Thrombolytics:   

Peptic ulcer & Recent surgery, Pericarditis, Aortic dissection, Brain tumor, & Stroke Start Thrombolytics:

Tpa  Ask for heparin protocol  Start B Blocker RISK FACTORS:  HTN  DM  FH  Coccaine Nitro (2nd dose) O/E:  JVP  Listen to heart  Base of lung  Compare BP in both Upper extremities to r/o coarctation of Ao CXR Once there is no Aortic Dissection  Thrombolytics (should be clear to r/o: Peptic ulcer, recent surgery, pericarditis, aortic dissection, brain tumor, and stroke) Based on ECG – counselling

Counseling Based on your ECG it is most likely you are having an heart attack. If stable – BP and HR are stable, but it is a serious condition, however it is treatable. Heart attack means that greater than one blood vessel supporting your heart is blocked by a clot that has to be reimoved. The medications are called clot busters. Based on ECG and no sign of pericarditis or signs of aortic dissection you are a good candidate for treatment. It is an effective medication, needs consent. 1% chance of stroke and we can start heparin.

Back to Content

Please note that this is only a draft version based on several sources, including: Dr. Basel Mohasen’s lectures, Toronto Notes, Therapeutic Choices and others. Edited and organized for the sake of all attendances of the Canadian Osce Exams: NAC OSCE and MCCQE2. by: Dr. Merlyn D’Souza and Dr. Zeev Gross, Spring 2011. Page 267

Respiratory System – P/E     

  

General inspection: comfort, colour, pursed lips, flare nose, intercostal retractions, auxiliary muscles Eyes, nose (perforated septum), mouth (ulcers, thrush in HIV, central cyanosis, moist tongue) Hands: peripheral cyanosis, clubbing, capillary refill Cervix: trachea, lymph nodes Chest: inspection (symmetry, expansion, intercostals retractions); Palpations for any pains, estimating chest expansion Tactile phremitus (“99”) Tappings (including sides): dullness/tympanic, diaphragmatic excursion Auscultation (including sides): vesicular sounds Vocal phremitus: “E” Whispering pectoriloqui: “1,2,3” Heart: pulses and auscultation (r/o AF and Rheumatic disease). Other lymph nodes: axial, femoral, popliteal Lower leg: no signs of Caposi sarcoma, DVT (Homan sign, measuring calf in case of tenderness or suspicious calf swelling).

Please note that this is only a draft version based on several sources, including: Dr. Basel Mohasen’s lectures, Toronto Notes, Therapeutic Choices and others. Edited and organized for the sake of all attendances of the Canadian Osce Exams: NAC OSCE and MCCQE2. by: Dr. Merlyn D’Souza and Dr. Zeev Gross, Spring 2011. Page 268

Diabetic Daughter 2y, Counsel Either she is not doing well in school as she is not seeing well due to vision problems Not playing well, as she is tired DKA Is it regular f/u or something special you wanted to discuss?

When was the last f/u? How was she diagnosed? What happen then? What were the symptoms? Any pain / vomiting? Are you feeling eating/drinking/peeing more? Any weight loss or blurred vision? From the last f/u till now have you had DKA? How about before? Have you had low blood sugar?

Talk with the father: Which medication does she take? How does she take? When was the last time? Do you take insulin or somebody else gave it to you? Do you take it all the time? DO you skip dose? Does she need any help to take insulin? DO you measure blood sugar regularly? When was the last time? Do you record them in the machine? (The glucometer should be used by only one patient). There is a blood work called ―Hemoglobin A1C‖ it is done every three month – did you do it? Did you start new medication? How about your diet? DO you have your log book? What do you eat? Have you ever seen by a dietician? PMHx FHx Counseling A lot of people have diabetes and she is not the only one. What’s your understanding of diabetes? Whenever we eat food contains sugar it is absorbed in our stomach and goes to the blood and from there to different parts of our body. Sugar act in our body like a fuel, in order for our body to use this energy it needs insulin. Patients having diabetes have not enough insulin. Sugar will be built up in your blood. The body tries to get rid or it, by peeing extra sugar – this will lead to thirsty and tiredness. Please note that this is only a draft version based on several sources, including: Dr. Basel Mohasen’s lectures, Toronto Notes, Therapeutic Choices and others. Edited and organized for the sake of all attendances of the Canadian Osce Exams: NAC OSCE and MCCQE2. by: Dr. Merlyn D’Souza and Dr. Zeev Gross, Spring 2011. Page 269

This can be avoided by controlling the blood sugar. If you control your blood sugar you’ll be able to play again. If not controlled – may end in DKA, hypoglycaemia and serious consequences. Always be aware of hypoglycaemic symptoms: loss of conscious, sweating, heart racing, hungry. Since you might lose conscious it is important to carry MedAlert Caed or Bracelet which will clarify your situation.

Back to Content

Please note that this is only a draft version based on several sources, including: Dr. Basel Mohasen’s lectures, Toronto Notes, Therapeutic Choices and others. Edited and organized for the sake of all attendances of the Canadian Osce Exams: NAC OSCE and MCCQE2. by: Dr. Merlyn D’Souza and Dr. Zeev Gross, Spring 2011. Page 270

Medical Error, Wrong blood transfused When there is a mistake, always there is a kind of unintentional medical error. (to the nurse) when informed about wrong blood – ask: ―did you stop the blood?‖ say: ―Well done!‖ If she asks not to tell the patient...ask her what her believe she may lose her job, and it is too early to determine who is responsible. Errors take place in medical practice. We don’t know what exactly happened. We will stabilize patient and ensure he’s fine and later deal with this issue.

Remove blood unit and keep cannula (to the patient)INTRO: I am the doctor in charge, and it looks like it was an unintentional medical error took place. We need to make sure you are stable. We don’t know who is responsible, there are at least 15 steps and in each step could have been an error. We will fill an incident report and as soon as we get result we will inform you. You can sue, it is your right at the moment it is my priority to stabilize you. I will start PRIMARY SURVEY:, ABCD A – Open your mouth (check for anaphylaxis, no swelling in mouth, ask for any itchiness, or difficulty breathing), Oxygen saturation. Normal air entry. Normal S1, S2 VITALS: Pleaese . Remove blood unit and keep cannula C: Start new IV line. Once new line, don’t give fluids if stable. Send blood: CBC, Lytes, INR, PTT, LFT, Cr, BUN, FDP, Haptoglobulin, Direct coombs test; Urinalysis: hemoglobulinuria Unit to be sent to blood bank for cross matching. Ask nurse to call the blood bank and keep original blood.

D D1 – I’d like to shine a light in your eyes. Pupils are round, active, and symmetrical. Squeeze my finger, wriggle...wriggle... D2 – (if febrile) give tylenol Please prepare for me :  Benadryl (Diphenhydramine) 50mg.  Steroids (Hydrocortisone) and  Epinephrine SECONDARY SURVEY: Hx (two parts:) Please note that this is only a draft version based on several sources, including: Dr. Basel Mohasen’s lectures, Toronto Notes, Therapeutic Choices and others. Edited and organized for the sake of all attendances of the Canadian Osce Exams: NAC OSCE and MCCQE2. by: Dr. Merlyn D’Souza and Dr. Zeev Gross, Spring 2011. Page 271

1. Condition (how is he feeling now) 2. ―Why blood was given?‖ CONDITION: Check out for Anaphylactic shock: Do you feel warm? Chills? Itchiness? Tinglings? Diffculty breathing? Wheezing? Swelling in lips / fingers? Hives? Before transfusion did you have fever? Check for Haemolytic reaction – any back or flank pain? P/E – no oozing at IV line Then press on flank and back – no pain for haemolytic reaction. Is it the first time? WHY did you receive blood? If received blood before – was there any complications? Any long term diseases? COUNSELLING: Mr. X what do you know about blood transfusion? It is a life saving measure, and a lot of measures are taken to make sure it is safe. However, like any other medication with blood transfusion there could be side effects, and these side effects could be serious. The most common side effect is:  Febrile reaction (3%), usually it is self limited and can happen again. Next time you receive blood we will give you tylenol.  Anaphylactic reaction. It is a severe allergic reaction, and it is very serious and we cannot predict it. However, we have good measures to deal with it, and your symptoms make it less likely that you have had an anaphylactic reaction. The third reaction is more serious and called  HEMOLYTIC reaction. Usually happens when patients receive blood belonging to another blood group. The fact that this blood is same as your blood group, and the symptoms are not consistent with haemolytic anemia make it less likely that this is not the case here. The blood is sent to the blood bank and once results are back we will get final confirmation, we will able to reassure you.

Back to Content

Please note that this is only a draft version based on several sources, including: Dr. Basel Mohasen’s lectures, Toronto Notes, Therapeutic Choices and others. Edited and organized for the sake of all attendances of the Canadian Osce Exams: NAC OSCE and MCCQE2. by: Dr. Merlyn D’Souza and Dr. Zeev Gross, Spring 2011. Page 272

Son has anaphylactic shock, is stable now. Next few minutes I’ll talk with you and hopefully will come to a good plan. Yawning – give empathy.

Hx (Short) It happened at home you should take history. If not – don’t take history.  Itchiness,  Swelling,  Hives.  Was he able to talk, wheezing, chest tightness,  Lost his consciousness,  Turned blue? Start immediately with Epinephrine. What have you done at the event? What did they do? Any other children at home with anaphylactic shock?

Management Based on the Hx your child has anaphylactic shock. Explain: a kind of severe allergic or hypersensitivity, from birth or develop later. Usually people get allergic to foods, medications, or chemicals. Any questions? At certain stage the immune system starts to interact with some elements of the peanut which are called antigens. From now on when your son will be exposed to the same antigens it will lead to release of some chemicals which will affect his skin, widening blood vessels which will become leaky and different parts of your body will become swollen. When not enough blood will reach the brain it will lose conscious, difficulty breathing. The concern we have is that it might happen again. It is common.

Plan: the best treatment is prevention. After that I need to go and talk with your child.  You have to check the ingredients of any food you buy – make sure it is peanut free.  IF there are other children at home they must be informed as well.  In case that your child was exposed to peanuts by mistake, you should use EpiPen – this is a special pen, has a cap at the top, which is needed to be activated by removing the cap, press it against his thigh for ten seconds. This increases the blood pressure for about 20 min, in that time you should seek help.  Your son should carry with him two pens – one at home and one on his bag.  he should carry Med Alert. In case your child become unconscious  I will refer him to allergist specialist.  Aspirin, stress test, and imaging...  Some children will outgrow it.

Back to Content

Please note that this is only a draft version based on several sources, including: Dr. Basel Mohasen’s lectures, Toronto Notes, Therapeutic Choices and others. Edited and organized for the sake of all attendances of the Canadian Osce Exams: NAC OSCE and MCCQE2. by: Dr. Merlyn D’Souza and Dr. Zeev Gross, Spring 2011. Page 273

Marijuana Counselling (Mother comes in to see you as she has discovered Marijuana in her son’s belongings) INTRO: As I understand you’re here because you’re concerned about your son. What is his name? What is your concern? How much did you find? Did you ask him about it?  WHAT MAKES YOU BELEIVE IT IS MJ ? Is he using it? Or Is he carrying it? Is it the first time you’ve found it?  Did you notice any CHANGES in his behaviour? Is he excited? Laughing out of nowhere? Is he preoccupied? Does he stare at a wall? Does he talk to himself? Is he aggressive? Any problems with the law? Any fights? Any criminal records? Is he more isolated?  How is his MEMORY? Is he more forgetful/lose his stuff? Does he take more time to react? Does he spend more time in his room? How much time do you spend with him? How much time is he out of the home? How much time does he spend with his friends? Do you know any of his friends? What kind of activity are they involved in?  Does he have a lot of MONEY? Does he ask for money? Do you believe he steals money? Do you think he smokes/or drinks alcohol?  How would you describe his MOOD? Is he depressed? Is he still interested in his hobbies? Does he worry a lot? Does he have excessive fears & avoid situations? Do you have concerns that he may harm himself or anyone else? EDUCATION: How is he doing in school? Have his grades dropped? DIET: How is his general health? Please note that this is only a draft version based on several sources, including: Dr. Basel Mohasen’s lectures, Toronto Notes, Therapeutic Choices and others. Edited and organized for the sake of all attendances of the Canadian Osce Exams: NAC OSCE and MCCQE2. by: Dr. Merlyn D’Souza and Dr. Zeev Gross, Spring 2011. Page 274

Have you ever seen a psychiatrist? Fhx: SAD

COUNSELLING: Based on what you’ve told me.There are no changes in his health & behaviour (assumed that there were no changes in behaviour as per mum) When it comes to Marijuana it is a commonly used drug by teenagers, sometimes only once for experiment. When we talk about Substance Abuse & drugs we talk about different categories. Marijuana is a SOFT DRUG,others like: Coccaine,Heroin& Amphetmanies are HARD DRUGS Let us talk about Marijuana first. It is from the Cannabis family & affects the brain by feeling happy, excited & enhances experience.Sometimes with prolonged use or in high doses can cause side effects including apathy. It interferes with memory,& can interfere with his studies & function & fine motor skills & may not be able to operate machinery It impairs judgement & he might take risks. Can cause Lung cancer In some teens,in high doses unmasks schizophrenia & cause psychosis Interferes with sexual function & can cause infertility & weight gain By itself marijuana is not strongly addictive & hence he can stop it at any time with help.One of the concerns of Marijuana though is it acts as a bridge to Hard drugs which are addictive i.e you’ve to increase the dose to have the same effect,which is called “TOLERANCE”,& then one cannot stop the drug as it causes withdrawal . It is a crime to use,hold hard drugs.People can lose their jobs. If injected increases risk of HIV,Hepa B & C

PLAN If you like,bring your son here I can talk to him. It is better to be a confidante to him. Try to be close to him, someone he can trust & can talk to.Try to make sure who’re his friends,& make sure you know what he is doing.Keep him busy with activities. If there are any druh prevention programmes in your community or his school,get him to attend them & gets the knowledge.

In case of the resident who was asked to backup his supervisor orthopaed 7. I am competent – to emphasize 8. Short term – we don’t have time so we need to see her urgently 9. Long term – solve the situations that it wouldn’t occur again

Back to Content

Please note that this is only a draft version based on several sources, including: Dr. Basel Mohasen’s lectures, Toronto Notes, Therapeutic Choices and others. Edited and organized for the sake of all attendances of the Canadian Osce Exams: NAC OSCE and MCCQE2. by: Dr. Merlyn D’Souza and Dr. Zeev Gross, Spring 2011. Page 275

Pregnant 35 YO has concern about breast feeding. Tell me more about your concern. Is it: General information you like to discuss or some specific concerns? If worried about pain: Ask:  Have you ever been pregnant before?  Have you ever breast fed before?  What makes you believe it is painful?  In addition to that any other concern? Mostly my concern is... It is a good concern,& I’m gald you came in today. How is your pregnancy? When was your last F/U? When is your due date? Let us talk about the pain..... Breast feeding is a natural physiological process & usually it does not cause any pain. However, sometimes it might cause some discomfort. If it pains there should be some reason for that. Most of those causes are treatable Most commonly – cracks and fissures. They are caused because of not proper care of the nipple. You have to make sure that they are moist, clean them, and don’t use soap. To learn appropriate technique it might take some time. I can send you to some classes that might guide you. Retracted nipple and inflammation of the breast (mastitis) you can still continue to breast. Localized condition, like abscess, we still recommend to continue to feed breast from the other side. Before we proceed further I’d like to make sure if you are a good candidate for breast feeding.  Do you have any long term diseases,  Do you take any medication or radiation,  Do you smoke or taking any drugs,  Have you been screened for TB or HIV.  Do you plan chemotherapy or radiation therapy? Based on what you’ve told me,you’re a good candidate for Breast feeding COMPARE BETWEEN BREAST MILK & FORMULA The reason we recommend breast feeding is that we cannot match it with formulas. The first 24 hours secretion is “ Colostrum” ,it is a special kind of milk& has a lot of antibiotics, immunoglobulin & essential amino acids, which are essential for your baby which will give him protection. With time the milk becomes more mature and suits the needs of your baby. It has the right amount of carbohydrate & fat. The quality of the fat is better. It has more whey relative to casein. The iron is less than cow milk but is more available (50% as to 20%). Less load on the kidneys There are other benefits to your baby and you – there is emotional connection which is important to both of you. Babies breast fed have  Less chances of having allergies  Less chance of having diarrhea  Less chance of being obese  Less ear infection, Please note that this is only a draft version based on several sources, including: Dr. Basel Mohasen’s lectures, Toronto Notes, Therapeutic Choices and others. Edited and organized for the sake of all attendances of the Canadian Osce Exams: NAC OSCE and MCCQE2. by: Dr. Merlyn D’Souza and Dr. Zeev Gross, Spring 2011. Page 276

 Some studies even suggest that they might have higher IQ. There are some benefits for you (the mom) It helps to lessen the postpartum bleedings (because the oxytocin) the uterus go back to it’s size Helps you to regain the figure you’d prior to pregnancy. It is clean, available, always at the right temperature, even though you don’t pay for it – It is one of the most important things you can give your child. If chooses to breast feed: I will send you to clinic who will teach you. At the beginning the breast feeding is on demand & with time it regulates ,& you need to feed every 3 – 4 hours & at least 10 min in each breast. Monitor weight gain to ensure that the baby is adequately fed. Occasionally the baby may be jaundiced & sometimes stool may be loose. If you choose to breast feed you’ve to be careful whenever you take medications or alcohol. You can go back to work, after the Maternity Leave (ask for how long) Breast feeding can be continued. You can use some special pumps. Even if there is engorgement you can use the pump. Make sure it is always clean. Breast milk can be stored at 6hr in room temp, 24 hr in fridge, and 6m in freezer. Don’t put it in the microwave for heating. Breast feeding is not reliable method of contraception. Recommend the minipill or barrier method.

Back to Content

Please note that this is only a draft version based on several sources, including: Dr. Basel Mohasen’s lectures, Toronto Notes, Therapeutic Choices and others. Edited and organized for the sake of all attendances of the Canadian Osce Exams: NAC OSCE and MCCQE2. by: Dr. Merlyn D’Souza and Dr. Zeev Gross, Spring 2011. Page 277

Febrile Seizure A child brought to the ER because of febrile seizure.Next 10m counsel him.  EVENT  TRIGGER r/o meningitis.  EDUCATE & what to do next time INTRO: As I understand you’re here because your child had a seizure 20 minutes ago,& my colleagues are looking after him & he is stable. During the next few minutes I’ll ask you few questions, and after that I’ll go with you to see him. EVENT:  Describe the event.  What happened?  Did you see him? (Started to shake. All over his body? Bite his tongue / rolling up his eyes / wet himself).  Did he fall from a height?  How long did it last?  Did he stop seizing by himself or did he need medical intervention?  How did he regain consciousness? After the seizure does he have any neurologic deficits:  Was he drowsy  Did not recognize you  Able to move his arms & legs Is it the 1st time? Or happened before If first time: Ask about fever? (if it started a week ago – did you seek medical assistance? Any ear discharge? Did they give you any treatment? Did you give it to him or no?) Why! Some studies show you can treat OM without antibiotics. If reason medication not given was because parent was busy.....You should look for the reason not to give the antibiotics (negligence?).

Was he playful,eating,drowsy, Is he having any vomiting? Skin rash? Coughing & phlegm SOB, Wheezing Foul smelling urine & painful peeing Head to toe... If you find nothing – ask when he got his last shot? (up to 72 hours he can have fever).  R/O meningitis, pneumonia.  Any family history of febrile seizures, epilepsy BINDE (especially immunization to R/o Measles)

COUNSELLING: Most likely on what you’ve told me, your child has condition called febrile seizure (FS).Do you know what it is? It is a special condition in children that might happen from 6m to 60m. We don’t know exactly why – we believe it is a sudden change in the temp & as the brain is not developed fully thes e changes might lead to the seizure. This condition might happen again. The best treatment is: PREVENTION Therefore from now whenever your child has a temperature Please note that this is only a draft version based on several sources, including: Dr. Basel Mohasen’s lectures, Toronto Notes, Therapeutic Choices and others. Edited and organized for the sake of all attendances of the Canadian Osce Exams: NAC OSCE and MCCQE2. by: Dr. Merlyn D’Souza and Dr. Zeev Gross, Spring 2011. Page 278

Seek medical attention. Give Tylenol and sponge bath to decrease his temp. Most of the children will outgrow this condition by the 6th year. Chances of epilepsy later in life are higher In FHx of epilepsy,it is a risk fct for development of epilepsy. They don’t recommend Diazepam because it might make him drowsy. I will give you brochures If it stopped less than 5m or more than 5m including neurological symptoms seek ER immediately. Brochure.

Back to Content

Please note that this is only a draft version based on several sources, including: Dr. Basel Mohasen’s lectures, Toronto Notes, Therapeutic Choices and others. Edited and organized for the sake of all attendances of the Canadian Osce Exams: NAC OSCE and MCCQE2. by: Dr. Merlyn D’Souza and Dr. Zeev Gross, Spring 2011. Page 279

PHONE CASE: “This is Dr. ... (immediately should introduce yourself). I am the Dr. In charge in the ER.As I understand, you’re calling as your child has swallowed some medication. I know that you’re stressed & it is a difficult time for you. I need your phone number now & it is important, as if we get disconnected I will call you back. What is your address? How far away from the hospital are you? Try & stay calm. Your son needs you, I am going to give you some instructions and you need to follow them. FIRST STEP: Is your son is alert or not? Is he conscious? Can he talk to you? Can he recognize you? (If he doesn’t – do you know how to do CPR and start with that.) He’s crying? What is his colour? Pink? Hold him and try to calm & soothe him. Try to hold him and check his mouth,if there are medications there,remove them. Is he breathing? We’ll send the ambulance for you. When did it happen? How long was he alone? Which medications did he take? Whose medications did he take? Do you have the container? (Don’t go to the next room to bring them,when the paramedics arrive then you can go & get the container). Do you know what condition your father have (was it vitamins, sleeping pills, or any other?) How much the amount? Don’t use any ipecac? Do not induce vomiting. Is it happened before? What is the weight of the child? BINDE (was it full date, did he needed special attention after term, and does he have any special conditions). Weight for two reasons – antidote and estimate neglect. Are his shots up to date? Are there other children at home? Have you visited the Er frequently? Post encounter Q: what are the first four steps you do when he arrives? ABC, Monitor vitals, IV line, NG, Foley as needed, Blood works – CBC, Lytes, BUN, Cr, Osmolality, Coagul, LFTs, Tox screen – blood and urine). List three risk factors forneglect for this child. What is the antidote for betablocker (glucagon) and for CaChannel is (Calcium gluconate). CAS & Poisoning centre. Please note that this is only a draft version based on several sources, including: Dr. Basel Mohasen’s lectures, Toronto Notes, Therapeutic Choices and others. Edited and organized for the sake of all attendances of the Canadian Osce Exams: NAC OSCE and MCCQE2. by: Dr. Merlyn D’Souza and Dr. Zeev Gross, Spring 2011. Page 280

Second scenario Phone case: Febrile Seizure This is Dr. ... (immediately should introduce yourself). I am the Dr. In charge in the ER.As I understand, you’re calling as your childis having a seizure. I know that you’re stressed & it is a difficult time for you. I need your phone number now & it is important, as if we get disconnected I will call you back. What is your address? How far away from the hospital are you? Try & stay calm. Your son needs you, I am going to give you some instructions and you need to follow them. While he is seizing just put him on the side, and not start any CPR. Is he seizing right now? Try to put him on the floor on the left side (the right bronchus is shorter than the lt.). Observe him. What is his colour? Is he still shaking? You send the ambulance. Can you tap on his shoulder? If he is not responding – can you do CPR? Can you feel his pulse? If stopped seizing...... Good Is he alert? Does he respond? Can he talk to you? Can he move his legs? EVENT: Can you describe what happened? OCD Fever +/Does your child have fever? Did he have Hx/Nx/Vx/Skin rash/Neck stiffness Any long term disease? Did you seek medical attention? What prevented you from giving the medication? Is it the same time or happened before. If it is the second time – more than 15m he needs intervention.

Back to Content

Please note that this is only a draft version based on several sources, including: Dr. Basel Mohasen’s lectures, Toronto Notes, Therapeutic Choices and others. Edited and organized for the sake of all attendances of the Canadian Osce Exams: NAC OSCE and MCCQE2. by: Dr. Merlyn D’Souza and Dr. Zeev Gross, Spring 2011. Page 281

Post Concussion. Hx & PE INTRO: As I understand you had a head injury 3 days ago when playing hockey. Start with EVENT: Before & After If LOC ask How long? Do u remember what happened? What was done?Were you hospitalized? Was a CT Scan done? HOW DO YOU FEEL TODAY? Full neuro assessment Headache: OCD PQRST U V A&A Vomiting,bending,Nausea Balance,vision falls weakness numbness Difficulty finding words Past Med Hx: HEADDS PE: Vitals CN Power Reflexes Sensory GAIT check Tendem gait Conclusion: I know you’re eager to play hockey Since you’ve headache you’re not ready to play again as you still have active Sx. If you start to play again your tolerance for injury is lower & if you are injured again,your tolerance is lower & recovery time is longer & there might be serious consequences. Why don’t you wait till full recovery time I will refer you to a PT & with gradual step up exercises you can get back to your game: One week with warming up, after that stationary activity, after that skiing, than drilling without contact (seven steps of rehab.).

Back to Content

Please note that this is only a draft version based on several sources, including: Dr. Basel Mohasen’s lectures, Toronto Notes, Therapeutic Choices and others. Edited and organized for the sake of all attendances of the Canadian Osce Exams: NAC OSCE and MCCQE2. by: Dr. Merlyn D’Souza and Dr. Zeev Gross, Spring 2011. Page 282

Osgood Schlatter 2 scenarios (Osgood schlatter and Post-concussion) Decision will based whether the child can tolerate pain or not? #1 About to see the father of 14yom with Osgood Schlatter. Make sure that the child best interest are preserved. What was done to diagnose the child? OCD PQRST compare to the other knee, is the first time or not, was any trauma. What is the child wish? (Don’t go for HEADDSSS since it is the father). Counseling What is your understanding of OS. Let me explain to you what is the mechanism for OS. Avoid him from playing, especially jumping. But he can continue with ice presses and pain killers. The rule is that he can continue up to his limit of his pain.

Back to Content

Please note that this is only a draft version based on several sources, including: Dr. Basel Mohasen’s lectures, Toronto Notes, Therapeutic Choices and others. Edited and organized for the sake of all attendances of the Canadian Osce Exams: NAC OSCE and MCCQE2. by: Dr. Merlyn D’Souza and Dr. Zeev Gross, Spring 2011. Page 283

IMMUNIZATION (Newcomer come to Canada from Ukraine, concerned about immunization) May need interpreter: Ask: Do you understand? Can I talk slower? Do you need interpreter? INTRO: As I understand you’re here as you’re concerned about vaccines & my understanding is that you’re new to Canada, Welcome to Canada! What is your concern?  Pt: My neighbour told me vaccines are not safe Dr: what do you mean? Pt: Concerned about autism & vaccine Dr: This is a reasonable enough concern,& I’m glad you came here. There is a misinformation about between vaccines & autism.The origin of this misunderstanding is due to a study done in the UK & the author of that study found a connection between autism & vaccines. Because vaccines are lifesaving & important for our children’s protection,further studies were done,also in other countries,& then it was definitely proved that there is no connection between autism & vaccines.The only connection was coincidence between time of the vaccine & time when symptoms of autism were picked up by parents.  Another common concern is that mercury was used as a preservative for MMR vaccine. It is no longer used now. When we find out why this study gave such a result it was found out that there was a bias in sample & thus led to the wrong conclusion.  Another common Qn pt may ask: These diseases do not exist in Canada, so why give my child the vaccine if there is no disease here. The world is getting smaller & even though we do not have these diseases in Canada, because we have the vaccines, it does exist around the world & people travel.So we do not want your son to get affected whenever there is an outbreak somewhere in the world. Hx: Let me ask some questions about your son:  How old is he?  Has he received any vaccines so far?  Were there any side effects?  Any reason why he was not vaccinated?  Does he have any congenital medical condition?  Does he have any allergies (egg??)  Any neurological history? Inform that baby is a good candidate As I told you vaccines are life saving, before vaccines many children died from measles, rubella. The reason being children are not fully protected & may get the infection & die. Once vaccinated, children get the immunity Any Qns? HOW: Please note that this is only a draft version based on several sources, including: Dr. Basel Mohasen’s lectures, Toronto Notes, Therapeutic Choices and others. Edited and organized for the sake of all attendances of the Canadian Osce Exams: NAC OSCE and MCCQE2. by: Dr. Merlyn D’Souza and Dr. Zeev Gross, Spring 2011. Page 284

We take different bugs like bacteria, viruses or products of these bugs & process them so that it does not harm the body,& inject it into our bodies by needles. Our body reacts by forming elements that fight these antigens, so later in life when your son is exposed to the real factor, these antibodies will protect him. Some of these antibodies will last forever; some will need booster doses.Because there are a lot of disease we need to minimize the number of injections & we’ve to give greater than one needle for vaccination. There is a combination vaccine e.g: PEDISIL = DPT + HiB + Polio This is given as a single shot at 2,4,6 & 18 Mo. We will give you a schedule to remind you each time you’ve to come to the clinic Concerning the SIDE Effects, the benefits clearly outweigh the S/E,however: A febrile reaction can develop & you can give Tylenol if this occurs Pain & swelling at injection site Some children can have prolonged crying Others may become floppy In still rare conditions can have a seizure Very rarely,can gt an anaphylactic reaction Since you’re a newcomer & not got your insuarence there are some organizations that will help you out.

Back to Content

Please note that this is only a draft version based on several sources, including: Dr. Basel Mohasen’s lectures, Toronto Notes, Therapeutic Choices and others. Edited and organized for the sake of all attendances of the Canadian Osce Exams: NAC OSCE and MCCQE2. by: Dr. Merlyn D’Souza and Dr. Zeev Gross, Spring 2011. Page 285

IUGR INTRO: As I understand you’ve just given birth to a baby,& my colleagues are looking after him,& I’m here to talk to you. How do you feel right now? Have you seen your baby? Did anyone tell you about your baby? If at this point mother voices a concern that she saw her baby covered with green stuff. Your baby was covered with “Meconium” one of the substances in the fluid surrounding your baby.It is normal for the baby when under stress during delivery. I’ve been told that your baby has a condition called “IUGR”,& I need to ask questions as to why it happened Qns about Pregnancy: Smoked/Alcohol/Drugs Qns about Delivery: Term or preterm MGOS: O:Previous pregnancy/abortions/miscarriage, & if yes how many? G:If Hx of Cancer or chemotherapy Any congenital disease in her or husband’s family or Consanguinity If she asks whether her mistake: Don’t reproach her – it is NOT her mistake. It is a multi-factorial condition. Can be due to various causes, some genetic, pregnancy, related to baby Because safe levels of smoking, drugs & alcohol not known, We always recommend not to smoke or drink for future pregnancies.

Back to Content

Please note that this is only a draft version based on several sources, including: Dr. Basel Mohasen’s lectures, Toronto Notes, Therapeutic Choices and others. Edited and organized for the sake of all attendances of the Canadian Osce Exams: NAC OSCE and MCCQE2. by: Dr. Merlyn D’Souza and Dr. Zeev Gross, Spring 2011. Page 286

CHILD ABUSE:    

# Femur # LE & (B) limbs Spiral # # post ribs 1.EVENT 2.1st time or prior 3.BINDE: 4.Past MH for osteoporosis imperfecta 4. COUNSELLING INTRO: As I understand you’re here as your child had a #.My colleagues are looking after the child, who is stable now. EVENT: How?......Describe what happened….if fall from couch: How high is the couch? When? …If time log……Why bring the child now??? If at night? Did he sleep or was he crying? Were you there? Did you see it? Any LOC? Is it the 1st TIME or has it happened before? If before? How many times? Type of #? Did you come to the same hospital or to a different one? Any other children at home? BINDE: Planned pregnancy Term pregnancy Any cong anomalies During pregnancy: SAD Immunizations up to date or not Weight today Development: Is he a difficult child? Environment: PARENT-CHILD RELATIONSHIP: Stress at home Who is primary caregiver or who feeds the child? How do the parents punish the child? Financial problems SAD Any Psy Hx in either parent PAST MH: Here specifically ask about Osteogenesis Imperfecta COUNSELLING: I can see that you’re going through a difficult time. Sometimes it is challenging to work & care for a child. From the history you gave me about the injury is not enough to explain such an injury. Children at

Please note that this is only a draft version based on several sources, including: Dr. Basel Mohasen’s lectures, Toronto Notes, Therapeutic Choices and others. Edited and organized for the sake of all attendances of the Canadian Osce Exams: NAC OSCE and MCCQE2. by: Dr. Merlyn D’Souza and Dr. Zeev Gross, Spring 2011. Page 287

this age have very flexible bones which are difficult to break by jumping off the couch. I’m sure you share my concerns with me about the safety of your child & in this situation we contact the CAS. The CAS will come & ask questions & talk to you & your partner: If does not accept & says will take my child……. Ask what makes you think like that? Do you have any experience about these matters? It is not neccassarily,they will asses the situation & if the family is considered safe… If pleads etc: Tell I’ve a legal responsibility to report to the CAS

Back to Content

Please note that this is only a draft version based on several sources, including: Dr. Basel Mohasen’s lectures, Toronto Notes, Therapeutic Choices and others. Edited and organized for the sake of all attendances of the Canadian Osce Exams: NAC OSCE and MCCQE2. by: Dr. Merlyn D’Souza and Dr. Zeev Gross, Spring 2011. Page 288

SUICIDE ATTEMPT 16/F suicide attempt ASA overdose Sex: AGe Depression Previous attempts Ethanol use Rational thinking loss Suicide in family Organized plan No support (here put HEADSS) Serious illness

INTRO: As I understand,you’re here because you overdosed on Aspirin last night & I’ve been told that you’ve been seen by my colleagues.My understanding is that you’re stable now,& I can talk to you. HOW DO YOU FEEL RIGHT NOW? (if she is playing around with phone etc ask her to disconnect & speak to you) Can you tell me more about what happened? Pt: I went home & took Aspirin Dr; why? Pt: I was frustrated Dr: Why? I can see that you’re busy with your phone,is it important? Can you stop for a few minutes? I’d Like you to know that whatever you tell me is confidential here Give confidientiality Dr: Why are you angry? Pt:I made a car accident When? Were you alone,or with someone else? Were you driver or passenger? Was anyone else hurt? How Much aspirin? Did you talke aspirin alongwith any other medication? Did you lose consciousness? Who called for help? Did You seek help ort someone else did? Was it IMPULSIVE or PLANNED? Did you leave a note? Have you recently been giving away your belongings to others? Is it the First time? Any suicidal attempt in the family? Please note that this is only a draft version based on several sources, including: Dr. Basel Mohasen’s lectures, Toronto Notes, Therapeutic Choices and others. Edited and organized for the sake of all attendances of the Canadian Osce Exams: NAC OSCE and MCCQE2. by: Dr. Merlyn D’Souza and Dr. Zeev Gross, Spring 2011. Page 289

Do you usually take alcohol,or take drugs? R What did you think about ending your life? R/O Psychosis: Sometimes when people want to end their lives they see a vision or hear voices,did you experience any of such? How is your mood, for the LAST 2 WEEKS (last 48 hrs make no difference) HEADDSS Do you have a driver’s liscence? What made you take the car? It seems an important trip, where did you go? After you leave the hospital, what do you plan to do? Finish SAD PERSONS If parents separted ask about the other parent If score 99%. Help us to r/o other conditions is very safe. Like neural tube defects Risk of abortion is very low < 5%. At the age of 35 the risk of having a miscarriage due to amniocentesis complication is less than the risk of having a child with a congenital anomaly. As always in medicine, we’ve to find a balance between risk & benefit. If pt wants an earlier test. Another option, not commonly done is Chorio venous Sampling Here risk of abortion is 2%, & thus higher than amniocentesis. The results are back in 48 hrs,however the chances of having a high false negative is 3% when baby is still affected Also there is a risk of limb injury Genetic abnormalities: cystic fibrosis, thalasemia, etc. For that reason you should be referred to a genetic counselling.

Please note that this is only a draft version based on several sources, including: Dr. Basel Mohasen’s lectures, Toronto Notes, Therapeutic Choices and others. Edited and organized for the sake of all attendances of the Canadian Osce Exams: NAC OSCE and MCCQE2. by: Dr. Merlyn D’Souza and Dr. Zeev Gross, Spring 2011. Page 296

From Mayo Clinic: Amniocentesis is a procedure in which amniotic fluid is removed from the uterus for testing or treatment. Amniotic fluid is the fluid that surrounds and protects a baby during pregnancy. This fluid contains fetal cells and various chemicals produced by the baby. With genetic amniocentesis, a sample of amniotic fluid is tested for certain abnormalities — such as Down syndrome and spina bifida. With maturity amniocentesis, a sample of amniotic fluid is tested to determine whether the baby's lungs are mature enough for birth. Occasionally, amniocentesis is used to evaluate a baby for infection or other illness. Rarely, amniocentesis is used to decrease the volume of amniotic fluid. Although amniocentesis can provide valuable information about your baby's health, the decision to pursue invasive diagnostic testing is serious. It's important to understand the risks of amniocentesis — and be prepared for the results. Before amniocentesis, you can eat and drink as usual. Your bladder must be full before the procedure, however, so drink plenty of fluids before your appointment. Your health care provider may ask you to sign a consent form before the procedure begins. You may want to ask someone to accompany you to the appointment for emotional support or to drive you home afterward.

During the procedure First, your health care provider will use ultrasound to determine the baby's exact location in your uterus. You'll lie on your back on an exam table and expose your abdomen. Your health care provider will apply a special gel to your abdomen and then use a small device known as an ultrasound transducer to show your baby's position on a monitor. Next, your health care provider will clean your abdomen with an antiseptic. Generally, anesthetic isn't used. Most women report only mild discomfort during the procedure. Guided by ultrasound, your health care provider will insert a thin, hollow needle through your abdominal wall and into the uterus. A small amount of amniotic fluid will be withdrawn into a syringe, and the needle will be removed. The specific amount of amniotic fluid withdrawn depends on the number of weeks the pregnancy has progressed. Please note that this is only a draft version based on several sources, including: Dr. Basel Mohasen’s lectures, Toronto Notes, Therapeutic Choices and others. Edited and organized for the sake of all attendances of the Canadian Osce Exams: NAC OSCE and MCCQE2. by: Dr. Merlyn D’Souza and Dr. Zeev Gross, Spring 2011. Page 297

You'll need to lie still while the needle is inserted and the amniotic fluid is withdrawn. You may notice a stinging sensation when the needle enters your skin, and you may feel cramping when the needle enters your uterus. The entire procedure usually takes about an hour, although most of that time is devoted to the ultrasound exam. In most cases, the fluid sample is obtained in less than two minutes. The small amount of amniotic fluid that's removed will be replaced naturally. After the procedure After the amniocentesis, your health care provider may use ultrasound to monitor your baby's heart rate. You may experience cramping or a small amount of vaginal bleeding immediately after the amniocentesis. Your health care provider may suggest resting after the procedure. You may want to ask someone to drive you home. You'll likely be able to resume normal activities the next day. Meanwhile, the sample of amniotic fluid will be analyzed in a lab. For genetic amniocentesis, some results may be available within a few days. Other results may take one to two weeks. Results of maturity amniocentesis are often available within hours. If you develop a fever after amniocentesis or if vaginal bleeding, loss of vaginal fluid or uterine cramping lasts more than few hours, contact your health care provider.

Back to Content

Please note that this is only a draft version based on several sources, including: Dr. Basel Mohasen’s lectures, Toronto Notes, Therapeutic Choices and others. Edited and organized for the sake of all attendances of the Canadian Osce Exams: NAC OSCE and MCCQE2. by: Dr. Merlyn D’Souza and Dr. Zeev Gross, Spring 2011. Page 298

39 YOF high grade squamous endometrial ca., ASK HER ABOUT RISK FACTORS In Lab work you always ask Why? And is it the first time? SPIKE Explain Local symptoms CSx MGOS PMHx Plan (colposcopy) INTRO: ...because it is the first time I want to ask you some questions so as to get a better understanding of your results. Why? Is it the first time? When was it done? Any reason prevented you from doing it? What was your result at that time? If done long ago? Some people want to know in ―Are you the kind of person who prefers to know all the details about what is going

on?‖ ―How much information would you like me to give you about your diagnosis and treatment?‖ ―Would you like me to give you details of what is going on or would you prefer that I just tell you about treatments I am proposing?‖ If not anxious: What do you know about Pap smear? Yes, we look for changes in the cervix including cervical cancer What do you know about Ca Cx? It is a common cancer & we pick it up with Pap’s smear & if detected early, outlook is good What are your expectations of today’s visit? The results are back & ―I wish I had better news for you but unfortunately it shows you have some changes in the pap smears, & these changes are called ― HGSIL‖.& these changes if Ca or not are not detected by PAPs smear. We need to do further assessment to determine whether it is Ca or not.

Please note that this is only a draft version based on several sources, including: Dr. Basel Mohasen’s lectures, Toronto Notes, Therapeutic Choices and others. Edited and organized for the sake of all attendances of the Canadian Osce Exams: NAC OSCE and MCCQE2. by: Dr. Merlyn D’Souza and Dr. Zeev Gross, Spring 2011. Page 299

Let me ask you some questions to see if you have some symptoms related to it: Local, Meta, Constitutional LOCAL: Vaginal bleeding/Discharge/Ulcers/Blisters/Warts? Pain with intercourse Bleeding with intercourse Lumps, bumps in groin area? Fever,wt loss,Back pain? MGOS M:At which age you had your first period, G: Any Gyn surgery Contraception? O: Have you been pregnant? How many times? At which age was your first pregnancy? S: At what age were you sexually active? How many partners did you have? With whom do you live? How long have you been in this relationship? Before this relationship,How many partners did you have? STDs, Smoke, How do you support yourself financially Past Medical Hx Family Hx ―I have bad news. The colposcopy result came back and consistent with cervical cancer. We need to take further steps & I will refer you to a gynaecologist. If you want future babies they will use local options & do something called a Cone biopsy If you do not want any more children the uterus & cervix will be removed & the prognosis is excellent

Back to Content

Please note that this is only a draft version based on several sources, including: Dr. Basel Mohasen’s lectures, Toronto Notes, Therapeutic Choices and others. Edited and organized for the sake of all attendances of the Canadian Osce Exams: NAC OSCE and MCCQE2. by: Dr. Merlyn D’Souza and Dr. Zeev Gross, Spring 2011. Page 300

Allergic Rhinitis, Counsel Intro OCD (seasonal: caused by pollens from trees. Summer, spring, early autumn – usually last several weeks, disappears and recurs following year at the same time; Perennial: occurs intermittently for years with no pattern or may be constantly present); P: is the nasal congestion is only in one side (allergic rhinitis) or varies from side to side (vasomotor rhinitis) COCA-B (should be clear rhinorrhea, under microscope it contains increased eosinophils); Watery/mucoid: allergic, viral, vasomotor, CSF leak (halo sign) Mucopurulent: Bacterial, foreign body Serosanguinous: Neoplasia Bloody: Trauma, neoplasia, bleeding disorder, hypertension/vascular disease ΑA: Allergic rhinitis (hay fever): most common inhaled allergans - house dust, wool, feathers, foods, tobacco, hair, mold; most common ingested allergans – wheat, eggs, milk, nuts; Vasomotor rhinitis: caused by – temperature change, alcohol, dust, smoke, stress, anxiety, neurosis, hypothyroidism, pregnancy, menopause, Drugs: parasympathomimetic drugs and estrogens (OCPs, HRTs); Beware of rhinitis medicamentosa: reactive vasodilation due to prolonged use (>5 days) of nasal drops and sprays (Dristan, Otrivin) ΑSx: Itching eyes with tearing, frontal headache and pressure, hypothyroid symptoms, change in menstruation (pregnancy/menopause); MOAPS (especially – anxiety, neurosis, and drugs); Complications: signs of sinusitis (pain in the face, post nasal drips, fever, severe headaches, teeth pain, PMHx of sinusitis); Ask for diagnosis of nasal polyps or obstruction in breathing through the nose when there is no sign of allergy; Ear pain (especially serous otitis media) SHx:

Counsel: From the Hx I’ve just taken it is most likely that you suffer from a condition called: allergic rhinitis/vasomotor rhinitis. This condition is very common and is caused by exposure to irritants in the environment which are called alergans. These alergans trigger the immune system to release substances which cause the congestion in your nose. Finding and eliminating the appropriate trigger/s can prevent this condition. For that reason I am sending you to do some allergy testing. Meanwhile I can recommend several options to alleviate your symptoms. For allergic rhinitis:  Nasal irrigation with saline  Spray, nasal drops, or tablets with antihistamines (e.g. diphenhydramine, fexofenadine)  Oral decongestants (e.g. pseudoephedrine, phenylpropanolamine)  I wouldn’t recommend to use topical decongestants since they may lead to a condition called “rhitinitis medicamentosa” which may increase and deteriorate your condition. In case of necessity – you may use a topical decongestant up to five days.  There are many other medications that might help in case of serious condition – like steroids (fluticasone), or for prevention (disodium cromoglycate), also ipratropium bromide. If very severe oral steroids may be used.  Desentization by allergen immunotherapy is also an option in some cases. For vasomotor rhinitis: Please note that this is only a draft version based on several sources, including: Dr. Basel Mohasen’s lectures, Toronto Notes, Therapeutic Choices and others. Edited and organized for the sake of all attendances of the Canadian Osce Exams: NAC OSCE and MCCQE2. by: Dr. Merlyn D’Souza and Dr. Zeev Gross, Spring 2011. Page 301

   

Some relief can be achieved by exercise (increased sympathetic tone) Drugs that called parasympathetic blockers (e.g. Atrovent nasal spray) In serious conditions – steroids (e.g. beclomethasone, fluticasone) There are also some invasive procedures that might be used in stubborn cases. Surgery (which is often with limited lasting benefit), electrocautery and cryosurgery which use hot or cold instruments to affect the lining or your nose. Overall this condition can be annoying but it is not dangerous and there are many ways to treat it. However, in most cases it is repeated and the benefit of each treatment should be well balance against its risks.

Back to Content

Please note that this is only a draft version based on several sources, including: Dr. Basel Mohasen’s lectures, Toronto Notes, Therapeutic Choices and others. Edited and organized for the sake of all attendances of the Canadian Osce Exams: NAC OSCE and MCCQE2. by: Dr. Merlyn D’Souza and Dr. Zeev Gross, Spring 2011. Page 302

Enuresis Intro, ask about the concern Ask about the name and age of the child OCD - Analyze the problem: Since when is he wetting his bed? Is it primary or secondary? Does the child lose control on his bladder during day or only at night? When did the child control his bladder and toilet? How does the child feel about it? How do his caregivers feel about it? R/O organic causes (red flags): DM: Drinking too much, going more often to pee, feeling tired, lost weight DI: Hx of meningitis, encephalitis (brain infection), head trauma Seizure UTI: Dysuria, odd smell or colour of urine Neuro: Bowel dysfunction, leg weakness or numbness, trauma or surgery to back Stress: Any stress or problem or new event Others: Sickle cell disease, pinworms, constipation, and the most common cause for diurnal dieresis is micturition deferral PMHx – including medications (diuretics) and allergies FHx BINDE (briefly – because the child is 8-9 y.o) How was the pregnancy (any problem) How was the delivery (NVD vs. C/S) Term or pre-term Are his regular shots up to date? How is his nutrition (does he eat well balanced diet)? How is his school performance? Who is the primary care giver? Who else live with them at home? Is he the only child? Counsel  Explain what is happening – say it is m/p regression of his development because of the current stresses in his life  It is caused by maturational lag in bladder control while asleep. It is self limiting and you need to give the child some time and he will adapt very well to the changes. About 20% of the children resolve spontaneously each year.  The prevalence of this problem: 10% of 6 y.o, 3% of 12 y.o, 1% of 18 y.o  Treatment by changing life style: limiting nighttime fluids and voiding prior to sleep, engaging child using rewards, bladder retention exercises, scheduled toileting  You can try a method called ―conditioning‖: ―wet‖ alarm wakes child upon voiding – this method has 70% success rate  As last resort you can try even medication: DDAVP by nasal spray or oral tablets, but there is high relapse rate and it is costly. Other medical options: oxybutynin (Ditropan), imipramine (Tofranil) – the latter is rarely used since it is lethal in overdose and has cholinergic side effects.

Please note that this is only a draft version based on several sources, including: Dr. Basel Mohasen’s lectures, Toronto Notes, Therapeutic Choices and others. Edited and organized for the sake of all attendances of the Canadian Osce Exams: NAC OSCE and MCCQE2. by: Dr. Merlyn D’Souza and Dr. Zeev Gross, Spring 2011. Page 303

Important Drugs to Remember Enuresis Desmopresin 0.2-0.6mg at bed time

Torticulosis Treatment: Diphenhydramine 50mg

Warfarin counselling Enoxaperin 20mg OD (low risk) Enoxaperin 40mg OD (high risk)

Want to stop Li Normal level 0.5-1.2

Breast feeding counseling

Please note that this is only a draft version based on several sources, including: Dr. Basel Mohasen’s lectures, Toronto Notes, Therapeutic Choices and others. Edited and organized for the sake of all attendances of the Canadian Osce Exams: NAC OSCE and MCCQE2. by: Dr. Merlyn D’Souza and Dr. Zeev Gross, Spring 2011. Page 304

View more...

Comments

Copyright ©2017 KUPDF Inc.
SUPPORT KUPDF